Está en la página 1de 179

Machine Translated by Google

872 Apéndice  A

det  A  =  (a11  +  ka21)a22  −  a21(a12  +  ka22)
=  (a11a22  −  a21a12)  +  k(a21a22  −  a21a22)
=  det  A  +  k  ∙  0
=  det  A

De  manera  similar,  si  B  es  la  matriz  obtenida  al  sumar  k  por  el  renglón  1  al  renglón  2,  
entonces  det  B  =  det  A.

EJEMPLO  A.5 Evalúe  el  det  A  con  el  teorema  A.5,  donde
1  2  −3
un  = 4  −5  6
−7  8  −9

SOLUCIÓN:  
Mantenga  el  1  en  la  columna  1  para  hacer  que  las  otras  dos  entradas  en  la  columna  1  (4  
y  −7)  sean  0.  Para  ello,  suma  −4  veces  el  renglón  1  al  renglón  2  y  7  veces  el  renglón  1  al  
renglón  3:
1  2  −3
det  A  = 0  −13  18
0  22  −30

Expandiendo  el  det  A  por  la  columna  1,  vemos  que
det  A  =  1[(−13)  ∙  (−30)  −  22  ∙  18]=−6

Las  propiedades  anteriores  utilizadas  simultáneamente  pueden  acelerar  la  evaluación  de  
los  determinantes,  como  muestra  el  siguiente  ejemplo.
EJEMPLO  A.6 Evalúe  el  det  A  si

−31  2  −1
3  0  −5  6
un  =
        
24  0  8         

−12  0  3

SOLUCIÓN:  Usa  
los  teoremas  A.4  y  A.5  para  evaluar  el  det  A.  Factoriza  2  de  la  fila  3:

−31  2  −1
3  0  −5  6
det  A  =  2
12  0  4
−12  0  3
Machine Translated by Google

A.2  Determinantes 873

Ahora  use  el  1  en  la  columna  1  para  hacer  que  las  entradas  restantes  en  la  columna  
sean  ceros.  Para  ello,  suma  3  veces  la  fila  3  a  la  fila  1  y  −3  veces  la  fila  3  a  la  fila  2;  
agregue  la  fila  3  a  la  fila  4:

072  −11  0  −6  −5  
−6  det  A  =  2  120  
4  040  7

Expanda  este  determinante  por  la  columna  1:

7  2  −11
det  A  =  2 −6  −5  −6
40  7

Expande  el  nuevo  determinante  por  la  fila  3:

2  −11   7  2  −6  
det  A  =  2  4
−5  −6
−  0  +  7 −5

=  2[4(−12  −  55)  −  0  +  7(−35  +  12)]
=  −858

Ejercicios  A.2

Evalúa  cada  determinante.
2  0   −3  2  5   a  −bba
1. 2. 3.
0  3 −7
100  020   350  670   23  5  14  
4. 003 5. 240 6. 7  6  9  15

ab  1  bc   1  +  x  1  1   abc  bca  


7. 1  ca  1 8. 1  +  y  1    +  
11   1  1 9. taxi
z

111  abc   ab  0  0  cd  0   −1011  0  1  −1  −1  


10 a2  b2  c2 11
0  0  0  ef  0  0  
12
1  −2  3  −4  −2  4  
g −6  8

Demuestra  cada  uno.

13.  Sean  A  y  B  dos  matrices  de  2  ×  2.  Entonces  |AB|=|A|∙|B|.
14.  Sea  A  cualquier  matriz  cuadrada  de  orden  n.  Entonces  |kA|  =  kn|A|.
Machine Translated by Google

874 Apéndice  A

15.  Sea  A  una  matriz  n  ×  n  tal  que  AAT  =  In.  Entonces  |A|=±1.

16.  Sea  A  =  (aij)n×n  donde  aij  =  0  para  j>i.  (Tal  matriz  se  llama  matriz  triangular  
superior.)  Entonces  |A|  =  a11a22 ...an.

Resuelve  cada  ecuación  para  f(x).
f(x)  1  x  f(a)   f(x)  1  x  x2  f(a)  1  
17 1  a  f(b)  1  b =  0 a  a2  f(b)  1  b  b2  
18 =  0
f(c)  1  c  c2

A.3  Funciones  exponenciales  y  logarítmicas*

Las  funciones  exponenciales  y  sus  parientes  más  cercanos,  las  funciones  logarítmicas,  forman  
una  de  las  clases  de  funciones  más  importantes.  Tienen  una  amplia  gama  de  excelentes  
aplicaciones  para  las  ciencias  de  la  computación,  las  ciencias  de  la  vida,  las  ciencias  de  la  
administración,  las  ciencias  físicas  y  las  ciencias  sociales.
Por  ejemplo,  una  función  exponencial  ocurre  en  las  ciencias  naturales:  E.  coli  es  
una  bacteria  unicelular  microscópica  que  se  multiplica  por  la  mitad  en  dos  bacterias  
aproximadamente  cada  20  minutos  en  condiciones  ideales.  Supongamos  que  
inicialmente  hay  una  bacteria  y  20  minutos  es  la  unidad  de  tiempo.  Entonces  habrá  
dos  bacterias  en  una  unidad  de  tiempo,  cuatro  en  dos  unidades,  ocho  en  tres  unidades,  
y  así  sucesivamente.  En  general,  habrá  f(x)=2x  bacterias  en  x  unidades  de  tiempo,  
suponiendo  que  ninguna  muera  durante  el  período.  Esta  función  pertenece  a  la  gran  
clase  de  funciones  exponenciales,  definidas  a  continuación.

Funcion  exponencial
Sea  a  un  número  real  positivo  diferente  de  1  y  x  un  número  real  arbitrario.  Entonces  
la  función  f(x)  =  ax  se  llama  función  exponencial  con  base  a.

Si  a  =  1  en  esta  definición,  la  función  se  convierte  en  la  función  constante  f(x)  =  1,  
que  tiene  poco  interés  práctico.  Como  resultado,  se  excluye  de  la  definición.

La  Figura  A.1  muestra  las  gráficas  de  varias  funciones  exponenciales  con  diferentes  
bases  a,  donde  a  >  1.  Crecen  más  rápido  y  más  abruptamente  a  medida  que  a  aumenta.
Podemos  usar  tales  funciones  exponenciales  para  describir  el  crecimiento  de  las  poblaciones;  
en  consecuencia,  sus  gráficas  se  denominan  curvas  de  crecimiento.
La  Figura  A.2  muestra  algunas  gráficas  exponenciales  y  =  ax,  donde  0  <a<  1.
Cada  uno  se  puede  obtener  volteando  el  gráfico  correspondiente  y  =  ax,  donde  a  >  1,  
sobre  el  eje  y.

Basado  en  T.  Koshy,  College  Algebra  and  Trigonometry  with  Applications,  McGraw­Hill,  
Nueva  York,  págs.  204–241.
Machine Translated by Google

A.3  Funciones  exponenciales  y  logarítmicas 875

Figura  A.1   y

Curvas  de  crecimiento  y  =   y  =  10x
ax,  a  >  1.
y  =  7x

y  =  3x

X
o

Figura  A.2 y
Curvas  de  caída  y  =  ax,  0   y  =  10−x
<a<  1.
y  =  7−x

y  =  3−x

X
o

Aquí  los  valores  de  y  disminuyen  a  medida  que  x  aumenta.  Estos  gráficos  se  utilizan  para  estudiar
la  desintegración  de  sustancias  radiactivas  y  se  denominan  curvas  de  desintegración.

Las  propiedades  importantes  de  la  función  exponencial  f(x)  =  ax  se  enumeran  a  continuación:

•  Dom(f)  =  (−∞,  ∞);  rango(f)  =  (0,  ∞).

•  La  intersección  con  el  eje  y  de  la  gráfica  es  1.

•  La  función  exponencial  es  biyectiva.  •  Si  a  >  1,  

entonces  f  es  una  función  creciente;  es  decir,  si  x1  >  x2  entonces  f(x1)  >  f(x2).  Si  0  <a<  1,  
entonces  f  es  una  función  decreciente;  es  decir,  si  x1  >  x2  entonces  f(x1)  <  f(x2).

•  El  eje  x  es  una  asíntota  horizontal  para  la  gráfica.  •  Las  gráficas  y  

=  ax  y  y  =  a−x  son  reflexiones  entre  sí  sobre
el  eje  y.

Los  siguientes  dos  ejemplos  exploran  algunas  aplicaciones  de  la  función  exponencial.
Machine Translated by Google

876 Apéndice  A

EJEMPLO  A.7 Considere  la  población  mundial  (en  miles  de  millones)  dada  por  la  Tabla  A.1.  Podemos  
usarlos  para  dibujar  una  curva  suave  como  en  la  Figura  A.3.  El  gráfico  muestra  que  la  
población  mundial  está  creciendo  exponencialmente.

Cuadro  A.1
Año  t 1 1000  1650  1750  1800  1850  1900  1950  1970  1990  2000

Población  0.25?  0,3  0,5  0,7  P(t) 0.9 1.1 1.6 2.5 3.6 5.2 6.1

Figura  A.3
y
El  crecimiento  
6
exponencial  de  la  
población  mundial. 5

t
1650 1750  1850 1950  2000

Años

EJEMPLO  A.8     Suponga  que  un  tazón  de  caldo  nutritivo  contiene  1,000,000  de  bacterias  viables  por  milímetro.  
La  Tabla  A.2  muestra  el  número  de  bacterias  viables  por  milímetro  en  el  caldo  después  de  
agregar  un  agente  para  matarlas.  La  población  en  el  tiempo  t  es  solo  una  décima  parte  de  
la  del  tiempo  t  −  10.  Sea  N(t)  el  número  de  bacterias  viables  en  el  tiempo  t.  Entonces  N(t)  =  
106−0.1t . Si  t  puede  tomar  todos  los  
valores  posibles  (al  menos  teóricamente),  entonces  la  gráfica  de  N(t)  se  parece  a  la  curva  
de  decaimiento  de  la  figura  A.4.

Tabla  A.2
Tiempo  t  (en  minutos) 0 10 20 30 40 50  60

Número  de  bacterias  N(t) 106 105 104 103 102 10 1

Una  función  exponencial  extremadamente  útil  es  f(x)  =  ex,  donde  e  es  el  número  
irracional  2.718281828... .  (La  letra  e  para  la  base  fue  elegida  en

Basado  en  RM  Thrall  et  al.  (ed.),  “Extermination  of  Bacteria,”  Some  Mathematical  Models  in  
Biology,  The  University  of  Michigan  Press,  Ann  Arbor,  MI,  1967,  pp.  PE3.1–PE3.2.
Machine Translated by Google

A.3  Funciones  exponenciales  y  logarítmicas 877

Figura  A.4 Nuevo  Testamento)

106

t
o Hora

honor  de  Euler.)  Se  establece  en  cálculo  que

1 norte

e  =  limn→∞  1  + norte

Las  gráficas  de  las  funciones  f(x)  =  ex  y  g(x)  =  e−x  aparecen  en  la  figura  A.5.

Figura  A.5 y

X
y  =  mi
y  =  e−x

X
o

logaritmos
Los  logaritmos  fueron  inventados  por  el  matemático  escocés  John  Napier  
(1550­1617)  para  facilitar  los  cálculos  numéricos.  Publicó  un  resumen  de  sus  
resultados  en  1614.

función  logarítmica
En  la  función  exponencial  y  =  ax,  donde  a  >  0  ya  =  1,  el  exponente  x  es  el  logaritmo  
de  y  en  base  a;  se  denota  por  log  a  ( y)  o
Machine Translated by Google

878 Apéndice  A

simplemente  registre  una  y.  Por  lo  tanto,

loga  y  =  x  si  y  solo  si  y  =  ax

La  función  g(x)  =  loga  x  es  la  función  logarítmica  con  base  a.
Con  la  base  a  siempre  positiva,  y  =  ax  es  positivo  para  todos  los  valores  de  x.
Como  resultado,  x  =  loga  y  se  define  solo  si  y  es  positivo.  En  otras  palabras,  solo  
se  definen  logaritmos  de  números  positivos.  Así,  el  dominio  de  la  función  logarítmica  
es  (0,∞),  y  el  rango  es  (−∞,∞).
Dado  que  la  función  exponencial  es  biyectiva  con  dominio  (−∞,∞)  y  rango  (0,∞),  
tiene  una  función  inversa.  El  siguiente  teorema  muestra  que  la  inversa  es  de  hecho  
la  función  logarítmica.

TEOREMA  A.6  La  función  logarítmica  g(x)  =  loga  x  es  la  inversa  de  la  función  exponencial  f(x)  =  ax.

PRUEBA:
Por  definición  alog  a X =  x  para  cada  x  >  0,  y  que  log  a(ax)  =  x  para  cada  x.
Después

(f  ◦  g)(x)  =  f( g(x))  =  f( loga  x)  =  aloga  x =  x
y

(g  ◦  f)(x)  =  g(f(x))  =  g(ax)  =  loga(ax)  =  x

Entonces  la  función  logarítmica  g  es  la  inversa  de  la  función  exponencial  f.

De  acuerdo  con  este  teorema,  podemos  obtener  la  gráfica  de  y  =  loga  x  por
reflejando  la  de  y  =  ax  sobre  la  línea  y  =  x,  como  se  muestra  en  la  Figura  A.6.

y y
Figura  A.6 y  =  hacha y  =  hacha

y  =  x y  =  x

y  =  logax
1 1
X X

o 1 o 1

a  >  1 0  <  un  <  1 y  =  logax


Machine Translated by Google

A.3  Funciones  exponenciales  y  logarítmicas 879

A  continuación  se  enumeran  las  propiedades  importantes  de  la  función  logarítmica  
g(x)  =  loga  x.

•  Dom( g)  =  (0,∞);  rango( g)  =  (−∞,∞).

•  La  intersección  x  de  la  gráfica  logarítmica  es  1.  •  La  

función  logarítmica  es  biyectiva.  •  Si  a  >  1,  g  es  una  

función  creciente;  si  0  <a<  1,  g  es  decreciente
función.

•  El  eje  y  es  una  asíntota  vertical.

•  Las  funciones  f(x)  =  ax  y  g(x)  =  loga  x  son  inversas  entre  sí;  sus  gráficos  son  
imágenes  especulares  entre  sí  sobre  la  línea  y  =  x.

•  Las  gráficas  de  y  =  log  a  x  y  y  =  log1/  ax  son  reflejos  entre  sí  sobre  el  eje  x.

Bases  de  uso  frecuente  Se  
utilizan  con  frecuencia  tres  bases  de  logaritmos,  10,  e  y  2.  Los  logaritmos  en  base  10  
se  denominan  logaritmos  comunes  o  logaritmos  de  Briggs,  en  honor  al  matemático  
inglés  Henry  Briggs  (1561­1631).  Su  base  generalmente  se  omite:

registro  x  =  registro10  x

Los  logaritmos  en  base  e  son  logaritmos  naturales  y  se  abrevian  como  ln:

ln  x  =  loge  x

Los  logaritmos  en  base  2,  que  a  menudo  usamos  en  informática,  aparecen  como  lg:

lg  x  =  log2  x

Las  propiedades  fundamentales  de  los  logaritmos  se  dan  en  el  siguiente  teorema.
Podemos  probar  cada  uno  usando  la  definición  de  un  logaritmo.
TEOREMA  A.7 Sean  a,  b,  x  e  y  números  reales  positivos  cualesquiera  tales  que  a  =  1,  b  =  1  y  n  
cualquier  número  real.  Entonces:  (1)  aloga  x  (3)  log  a  y  (5)  log  a  1  =  0  (7)  log  a  (ax)  =  x  
=  xxx  =  loga  b
loga  x  (9)  logb   (2)  log  a(xy)  =  log  a  x  +  log  a  y  (4)  
=  log  a  x  −  log  a  y log  a(xn)  =  n  log  a  (6)  log  Xaa  =  1  
(8)  log  ax  =  log  a  y  si  y  solo  si  x  =  y
Machine Translated by Google

880 Apéndice  A

Los  siguientes  tres  ejemplos  ilustran  algunas  funciones  de  micrófono  logarítmicas  y  
exponenciales  útiles.

EJEMPLO  A.9  El  pH  de  una  solución,  que  mide  su  acidez  o  alcalinidad,  se  define  como  el  logaritmo  negativo  de  
la  concentración  de  hidrógeno  x  en  moles  por  litro  de  solución;  es  decir,  pH  =  −log  x.  A  
25◦C,  el  pH  del  agua  pura  es  7;  si  el  pH  de  una  solución  cae  por  debajo  de  7,  la  solución  es  
ácida  y  por  encima  de  7  es  alcalina.  Calcule  el  pH  de  una  solución  donde  la  concentración  
de  iones  de  hidrógeno  es  6.  7  ×  10−4.

SOLUCIÓN:
pH  de  la  solución  =  −  log  (6.  7  ×  10−4)  =  −  log  

6.  7  −  log  (10−4)

=  −  logaritmo  6.  7  +  4

≈  −0.  8261  +  4
=  3.  1739

EJEMPLO  A.10  La  población  mundial  (en  miles  de  millones)  en  cualquier  momento  t  (en  años)  está  dada  por  p(t)  
, población  
=  6.1e0.0167t  
donde  p(0)  
denota  
duplica   la  
el   población  
nivel   en  2000.  A  la  tasa  dada,  ¿cuándo  ¿La  
de  2000?

SOLUCIÓN:  
Población  en  2000  =  p(0)  =  6,1  mil  millones.  Necesitamos  el  valor  de  t  cuando  p(t)  =  12.2  
(Figura  A.7).  Después

6.  1e0.0167t  =  12.  2

Dividir  por  6.1:
e0.0167t  =  2

Después

ln  e0.0167t  =  ln  2

0.  0167t  =  ln  2

en  2
t  =
0.  0167
≈  42  años

Figura  A.7
12.2
t  = ?
2000 6.1
Machine Translated by Google

A.3  Funciones  exponenciales  y  logarítmicas 881

EJEMPLO  A.11  La  vida  media  de  una  sustancia  radiactiva  es  el  tiempo  necesario  para  que  se  desintegre  la  
mitad  de  esa  sustancia.  La  cantidad  de  11C,  un  isótopo  de  carbono,  presente  en  un  tiempo  
futuro  t  (en  meses)  está  dada  por  A(t)  =  100e−0.0338t . encuentra  la  mitad
vida  de  la  materia.

SOLUCIÓN:  
Cantidad  original  =  A(0)  =  100  gramos.  La  vida  media  es  el  valor  de  t  cuando  A(t)  =  2A(0)  
1
=  50  (Figura  A.8).  Después

100e−0.0338t  =  50

Dividir  por  100:
1
e−0.0338t  =  
2
Después

1  ln  e−0.0338t  =  ln  2

−0.  0338t  =  −  ln  2
en  2
t  =  
0.0338
≈  20.  5  meses

Figura  A.8
100

t  = ?

50

Ejercicios  A.3

Demuestre  cada  uno,  donde  f  denota  la  función  exponencial.  f(x)  

1.  f(x  +  y)  =  f(x)  ∙  f( y) 2.  
f(x  −  y)  =  f( y)
3.  f  es  inyectiva.

4.  La  función  logarítmica  g(x)  =  log  a  x  es  inyectiva.
Machine Translated by Google

882 Apéndice  A

Demuestre,  por  contradicción,  que  cada  uno  es  un  número  irracional.

5.  log  2   6.  registro  3

7.  La  concentración  de  iones  de  hidrógeno  en  una  solución  es  3.76  ×  10−8.
Calcule  el  pH  de  la  solución.

8.  El  pH  de  una  solución  es  5.3575.  Calcule  su  concentración  de  iones  de  hidrógeno.

La  intensidad  del  sonido  que  el  oído  humano  puede  escuchar  se  mide  en  decibelios,  
llamado  así  por  Alexander  Graham  Bell  (1847–1922),  el  científico  estadounidense  que  
inventó  el  teléfono.  El  número  de  decibelios  en  un  sonido  de  intensidad  I  viene  dado  por

yo
B  =  10  registro
yo0

donde  I0  es  la  intensidad  estándar  de  10−12  vatios  por  metro  cuadrado  (W/m2).
Encuentra  el  número  de  decibelios  en:

9.  Umbral  de  audición  (10−12  W/m2)
10.  Conversación  normal  (10−6  W/m2)

11.  Tráfico  en  la  calle  (10−5  W/m2)

12.  Dos  sonidos  de  intensidad  I1  e  I2  tienen  decibelios  B1  y  B2,  respectivamente.
I1 .
Demuestre  que  B1  −  B2  =  10  log
I2
13.  La  magnitud  de  una  estrella  es  una  medida  de  su  brillo:  cuanto  más  brillante  es  la  
estrella,  menor  es  la  magnitud.  Si  b1  y  b2  denotan  las  magnitudes  de  dos  estrellas,  
B1  y  B2  sus  respectivos  brillos,
B1  
b1  −  b2  =  −2,512  Iniciar  sesión .  Las  estrellas  más  brillantes,  Aldebaran  y  Sirius,  
B2  tienen  magnitudes  1  y  −1,6,  respectivamente.  
¿Cuánto  más  brillante  es  Sirio  que  Aldebarán?

La  magnitud  absoluta  M  de  una  estrella,  su  magnitud  aparente  m  y  la  distancia  estelar  d  
(en  parsecs)  de  la  tierra  están  relacionadas  por  la  fórmula  m  =  M  +  5  log  0.1d.

14.  Calcula  la  distancia  estelar  de  Aldebarán  si  su  magnitud  absoluta  es
−0,2  y  su  magnitud  aparente  0,86.
15.  La  diferencia  m  −  M  es  el  módulo  de  distancia  de  la  estrella.  Él
el  módulo  de  distancia  de  Sirio  es  −2,86.  Encuentre  su  distancia  estelar.

16.  Con  Rubik's  Cube,  un  juego  popular,  el  objetivo  es  convertir  sus  caras  en  una  
configuración  particular.  El  número  mínimo  de  vueltas  requeridas  de
3  ×  1019
alguna  posición  al  estado  deseado  es  1  +  log154. .  Calcular
18
eso.  (C.  Kluepfel,  1982)
Machine Translated by Google

A.4  Generación  de  permutaciones  y  combinaciones 883

A.4  Generación  de  permutaciones  y  combinaciones

Las  secciones  6.2  a  6.5  presentaron  varios  tipos  de  permutaciones  y  combinaciones,  
así  como  fórmulas  para  calcular  el  número  de  r­permutaciones  y  r­combinaciones  de  un  
conjunto  finito  S.  En  esta  sección,  estudiaremos  algoritmos  que  las  enumeran.

Generación  de  permutaciones
Por  conveniencia,  elegimos  S  =  {1,  2, ... ,  n}.  Presentaremos  un  algoritmo  para  generar  
las  diversas  permutaciones  de  elementos  del  conjunto.  El  algoritmo  se  basa  en  el  
ordenamiento  lexicográfico,  el  mismo  tipo  que  se  usa  para  ordenar  las  palabras  en  un  
diccionario.

Orden  lexicográfico
En  el  ordenamiento  lexicográfico,  una  permutación  A  =  a1a2 ...an  es  menor  que  (o  
precede)  a  una  permutación  B  =  b1b2 ...bn  si:
• a1  <  b1  o

• ai  =  bi  para  1  ≤  yo  ≤  k  −  1,  y  ak  <  bk.

Este  es  precisamente  el  orden  que  usamos  para  ordenar  alfabéticamente  las  
palabras  (de  la  misma  longitud)  en  el  diccionario.  Por  ejemplo,  la  palabra  calcular  
precede  a  la  palabra  permutar  y  la  palabra  patrimonio  precede  a  la  palabra  estimar.
EJEMPLO  A.12 Considere  la  permutación  a1a2a3a4  =  2134  y  b1b2b3b4  =  2143  del  conjunto  {1,  2,  3,  
4}.  Coinciden  en  las  dos  primeras  posiciones:  a1  =  b1  y  a2  =  b2.  Pero  difieren  en  las  
terceras  posiciones:  a3  =  b3.  Dado  que  a3  <  b3,  la  permutación  2134  precede  a  la  
permutación  2143  en  el  ordenamiento  lexicográfico.

¡Recuerda  que  hay  4!  =  24  permutaciones  del  conjunto  {1,  2,  3,  4}.  Se  enumeran  en  
la  Figura  A.9  en  orden  lexicográfico  por  columnas,  comenzando  con  1234.  (¿Puede  
encontrar  un  patrón  para  enumerarlos?)

Figura  A.9 1234  1423  2314  3124  3412  4213  1243  1432  
2341  3142  3421  4231  1324  2134  2413  3214  
4123  4312  1342  2143  2431  3241  4132  4321

Supongamos  que  tenemos  un  algoritmo  para  generar  la  siguiente  permutación  más  
grande  b1b2 ...  bn  a  partir  de  una  permutación  dada  a1a2 ...  an.  Luego,  comenzando  
con  la  permutación  123 ...  n,  podemos  invocarla  para  generar  las  n  −  1  permutaciones  
restantes.  A  continuación  se  describe  el  procedimiento  para  generar  la  siguiente  
permutación  mayor  a  partir  de  una  permutación  dada  en  orden  lexicográfico.
Machine Translated by Google

884 Apéndice  A

Si  an−1  <  an,  cámbielos  para  obtener  la  nueva  permutación  a1a2 ...an−2  anan−1.  
Entonces  la  permutación  a1a2 ...an−2an−1an  precede  a  la  permutación  
a1a2 ...an−2anan−1  en  el  ordenamiento  lexicográfico.
Por  ejemplo,  considere  la  permutación  2143;  sucede  a  la  permutación  2134  en  el  
listado  de  la  Figura  A.9.
Por  otro  lado,  supongamos  que  an−1  >  an.  (Entonces,  cambiándolos  no  podemos  
obtener  una  permutación  más  grande).  Así  que  miramos  an−2.  Si  an−2  <  an−1,  
encuentre  el  menor  de  los  elementos  an−1  y  an  que  sea  mayor  que  an−2;  colóquelo  
en  la  posición  n  −  2;  y  ahora  organice  an−2  y  los  elementos  restantes  en  orden  
creciente.
Por  ejemplo,  considere  la  permutación  a1a2a3a4  =  2341  en  la  figura  A.9.
Aquí  a3  >  a4,  pero  a2  <  a3;  el  menor  de  a3  y  a4  que  es  mayor  que  a2  =  3  es  a3  =  4;  
así  que  mantén  4  en  la  posición  2  para  obtener  24__;  ahora  organice  los  elementos  
restantes  a2  =  3  y  a4  =  1  en  orden  ascendente  para  producir  la  siguiente  permutación  
más  grande  2413  (vea  la  Figura  A.9).  Si  an−2  >  an−1,  necesitamos  ver  los  elementos  
an−3  a  an.

En  términos  más  generales,  todo  lo  que  tenemos  que  hacer  es  lo  siguiente.  De  
derecha  a  izquierda,  encuentra  el  primer  par  de  elementos  ai,  ai+1  tal  que  ai  <  
... >ai+1,  
elementos  ai+1,  ai+1     ai+2  d>onde  
  ai+2, ... ,  
>  an.  
aLn,  
uego  
digamos,  
encuentra  
aj,  que  
el  e
ms  
ás  
mayor  
pequeño  
que  a
di.  
e  los  
Coloque  aj  en  la  posición  i.  Ahora  ordene  los  elementos  ai,  ai+1, ... ,  aj−1,  aj,  
aj+1, ... ,  an  en  orden  creciente  en  las  posiciones  i  +  1  a  n.

El  siguiente  ejemplo  ilustra  este  procedimiento.
EJEMPLO  A.13  Encuentre  la  siguiente  permutación  mayor  que  a1a2a3a4  =  3421.
SOLUCIÓN:  El  
primer  par  ai,  ai+1  de  derecha  a  izquierda  para  el  cual  ai  <  ai+1  es  el  par  a1,  a2.  Así  
que  encuentra  el  más  pequeño  de  los  elementos  a2  =  4,  a3  =  2  y  a4  =  1  que  sea  
mayor  que  a1  =  3.  Claramente,  es  a2  =  4.  Así  que  colocamos  4  en  la  posición  1.  
Ahora  colocamos  los  elementos  3,  2  y  1  en  orden  creciente:  123.  En  consecuencia,  la  
siguiente  permutación  más  grande  es  4123.  (Vea  la  Figura  A.9.)

La  discusión  anterior  se  puede  traducir  a  un  algoritmo  para  encontrar  la  siguiente  
permutación  más  grande  que  sigue  a  una  permutación  dada  a1a2 ...  an.  Se  presenta  
en  el  Algoritmo  A.1.
Algoritmo  siguiente  permutación  (a1a2...an)
(*  Este  algoritmo  encuentra  la  permutación  más  grande  que  la  
permutación  dada  a1a2...an.  Suponga  que  no  es  la  permutación  
más  grande  n(n  ­  1)...321.  *)
0.  Inicio  (*algoritmo*)
(*  De  derecha  a  izquierda,  encuentre  el  primer  par  ai,  ai+1  para
que  ai  <  ai+1.  *)  1.  yo  ←  
n­1
Machine Translated by Google

A.4  Generación  de  permutaciones  y  combinaciones 885

2.  Mientras  ai  >  ai+1  do  (*  seguir  buscando  a  la  izquierda  *)  3.  i  ←  i­1  (*  
Cuando  
ai+1.  
salgamos  
Hallar  lda  
el  
mbenor  
ucle,  
dhe  
abremos  
los  elementos  
encontrado  
ai+1  au  tna  
ravés  
i  para  
de  laa  
n,  
que  
digamos,  
ai  <  
ak,  que  es  mayor  que  ai.Como  ai  +1  >  ai+2  > ...  >  an,  explore  de  
derecha  a  izquierda  para  encontrar  el  elemento  ak  que  es  mayor  que  ai.  
*)  4.  k  ←  n  5.  Mientras  ak  <  ai  do  (*  continúa  escaneando  hacia  la  
izquierda.  *)  6.  k  ←  k­1

(*  ak  es  el  más  pequeño  de  los  elementos  ai+1  hasta  an  que  es  
mayor  que  ai.  Intercambiar  ai  y  ak.  *)  7.  intercambiar  ai  y  ak  8.  
ordenar  los  elementos  en  las  posicionesi+1  hasta  n  en

orden  ascendente.
9.  Fin  (*algoritmo*)

Algoritmo  A.1

EJEMPLO  A.14  Utilizando  el  Algoritmo  A.1,  enumere  en  orden  lexicográfico  todas  las  permutaciones  de  
elementos  del  conjunto  {1,  2,  3}.
SOLUCIÓN:  
Comenzamos  con  la  permutación  básica  a1a2a3  =  123.  Dado  que  a2  <  a3  y  el  menor  de  
los  elementos  a3  que  es  mayor  que  a2  es  a3,  intercambie  2  y  3  para  obtener  la  siguiente  
permutación  mayor  132.  Dado  que  1  <  3  y  el  menor  de  3  y  2  que  es  mayor  que  1  es  2,  
intercambiamos  1  y  2,  y  ordenamos  los  elementos  3  y  1  para  obtener  la  permutación  213.  
Continuando  así,  obtenemos  las  permutaciones  restantes  en  orden  lexicográfico:  123,  
132,  213,  231,  312,  321.  (Vea  también  el  diagrama  de  árbol  en  la  Figura  A.10).

Figura  A.10 permutaciones

2 3 > 123
1
3 2 > 132

1 3 > 213
2
3 1 > 231

1 2 > 312
3
2 1 > 321

Generando  Combinaciones
¿Cómo  podemos  generar  las  r­combinaciones  del  conjunto  {1,  2, ... ,  n},  donde  0  <r<  n?  
Primero  enumeramos  las  diversas  combinaciones  en  orden  numérico  creciente.
Machine Translated by Google

886 Apéndice  A

Por  ejemplo,  hay  diez  combinaciones  de  3  del  conjunto  S  =  {1,  2,  3,  4,  5};  en  orden  lexicográfico  
son:

123,  124,  125,  134,  135,  145,  234,  235,  245,  345

¿Cómo  podemos  encontrar  la  siguiente  combinación  de  3  más  grande  de  una  combinación  de  
3  dada?  Por  ejemplo,  considere  la  combinación  de  3  134  del  conjunto  S.
El  último  elemento  (desde  la  derecha)  es  menor  que  5  (=  n).  Entonces  lo  incrementamos  en  1  
para  obtener  la  siguiente  combinación  más  grande  135.

EJEMPLO  A.15  Usando  el  conjunto  S  =  {1,  2,  3,  4,  5},  encuentre  la  combinación  que  sigue  a  a1a2a3  =  135  en  orden  
lexicográfico.

SOLUCIÓN:  Note  
que  a3  =  5,  el  mayor  en  S.  Así  que  mire  a2;  a2  <  5;  así  que  incrementa  a2  en  1  para  obtener  4.  
Reemplaza  a3  en  1  más  que  el  actual  a2:  a3  ←  a2+1  =  4+1  =  5.
Por  lo  tanto,  la  siguiente  combinación  más  grande  es  145.

EJEMPLO  A.16  Encuentre  la  combinación  que  sigue  en  orden  lexicográfico  a  la  combinación  de  3
145  del  conjunto  {1,  2,  3,  4,  5}.

SOLUCIÓN:  Sea  
a1a2a3  =  145.  Claramente,  a3  =  5  y  a2  =  4.  Como  a2  <  5,  si  lo  incrementamos  en  1  para  obtener  
4  +  1  =  5,  el  nuevo  a3  debería  ser  5  +  1;  desafortunadamente,  no  existe  en  el  conjunto.  Entonces  
vaya  a  a1  =  1,  que  es  menor  que  5.  (Observe  que  a3  =  5  −  3  +  3;  a2  =  4  =  5  −  3  +  2;  pero  a1  =  1  
=  5  −  3  +  1.  Por  lo  tanto,  a1  es  el  primer  elemento  por  la  derecha  tal  que  ai  =  n  −  r  +  i.)  Aumentar  
a1  en  1 :  a1  ←  a1  +1;  entonces  a1  ←  2.  Ahora  asigne  a2  =  a1  +1  =  3  y  a3  =  a1  +2  =  4.

La  combinación  resultante  es  234.

Así,  para  encontrar  la  combinación  que  sigue  a  la  r­combinación  a1a2 ...ar,  procedemos  de  la  
siguiente  manera.  De  derecha  a  izquierda  encontramos  el  primer  elemento  ai  tal  que  ai  =  n  −  
r  +  i.  Incrementar  ai  en  1 :  ai  ←  ai  +  1.  (Usando  el  nuevo  ai)  asignar  los  valores  ai  +  1,  ai  +  
2, ...  a  ai+1,  ai+2, ... ,  ar,  respectivamente;  es  decir,  aj  ←  ai  +  j  −  i,  donde  i  +  1  ≤  j  ≤  r.

EJEMPLO  A.17  Encuentre  la  combinación  que  sigue  en  el  ordenamiento  lexicográfico  de  3­
combinación  245  del  conjunto  {1,  2,  3,  4,  5}.

SOLUCIÓN:  Aquí  
n  =  5,  r  =  3  y  n  −  r  =  2.  Sea  a1a2a3  =  245.  De  derecha  a  izquierda,  encuentre  el  primer  ai  tal  que  
ai  =  n  −  r  +  i  =  2  +  i.  Claramente,  a3  =  5  =  2  +  3,  a2  =  4  =  2+2;  pero  a1  =  2  =  2+1.  Por  lo  tanto,  el  
primero  de  tales  ai  es  a1.  Actualice  a1  como  a1  +  1:  a1  ←  2  +  1  =  3.  Ahora  asigne  el  valor  a1  +  j  −  
1  a  aj  para  las  posiciones  restantes  j,  es  decir,  2  y  3:

Cuando  j  =  2,  a2  =  a1  +  2  −  1  =  3  +  1  =  4
Machine Translated by Google

A.4  Generación  de  permutaciones  y  combinaciones 887

Cuando  j  =  3,  a3  =  a1  +  3  −  1  =  3  +  2  =  5

La  combinación  resultante  es  345.

Estas  discusiones  nos  llevan  al  Algoritmo  A.2.
Combinación  siguiente  del  algoritmo  (a1a2...ar)
(*  Este  algoritmo  encuentra  la  combinación  que  sigue  a  la  combinación  r  
a1a2...ar  del  conjunto  {1,2,...,n}.  Suponga  que  la  combinación  dada  no  
es  la  combinación  más  grande  (n  ­  r  +  1). ..(n  ­  1)n.*)

0.  Comenzar  (*  combinación  siguiente  *)
(*  Encuentra  el  primer  ai  de  derecha  a  izquierda  para  el  cual  ai  
=  n  ­  r  +  i.  *)  1.  i  ←  r  2.  while  ai  =  n  ­  r  +  i  do  (*continúa  
escaneando*)  3.  i  ←  i­  1  4.  ai  ←  ai  +  1  (*  actualiza  ai  *)  (*  actualiza  los  
valores  de  ai+1,  ai+2,...,ar  *)  5.  para  j  =  i  +  1  to  r  do  6.  aj  ←  ai  +ji  7.  Fin  
(*siguiente­combinación*)

Algoritmo  A.2

EJEMPLO  A.18  Usando  el  algoritmo  A.2,  encuentre  la  combinación  que  sigue  en  orden  lexicográfico  a  la  
combinación  de  4  1345  del  conjunto  {1,  2,  3,  4,  5}.
SOLUCIÓN:  
Aquí  n  =  5,  r  =  4  y  n  −  r  =  1.  Sea  a1a2a3a4  =  1345.  Comenzando  con  a4,  calcule  ai  y  n  −  
r  +  i  hasta  que  ai  =  n  −  r  +  i:

a4  =  5  =  1  +  4 a3  =  4  =  1  +  3
a2  =  3  =  1  +  2 a1  =  1  =  1  +  1

Por  lo  tanto,  i  =  1.  En  consecuencia,  incrementa  el  valor  de  a1  en  1:

a1  ←  a1  +  1  =  2.

Actualice  los  valores  de  a2,  a3  y  a4:  a2  ←  a1  +  1  =  3,  a3  ←  a1  +  2  =  4,  a4  
←  a1  +  3  =  5.  Por  lo  tanto,  la  combinación  resultante  es  2345.

Ejercicios  A.4

Encuentre  la  siguiente  permutación  más  grande  que  cada  permutación  en  el  orden  
lexicográfico,  si  existe.

1.  1432 2.  2341 3.  4132 4.  4321

5.  21345 6.  21354 7.  21543 8.  35421


Machine Translated by Google

888 Apéndice  A

Encuentre  las  siguientes  tres  permutaciones  que  siguen  a  cada  permutación  en  orden  
gráfico  léxico.
9.  213 10.  1324 11.  2314 12.  2413

13.  3412 14.  13245 15.  23514 16.  45213

Usando  el  algoritmo  de  la  siguiente  permutación,  enumere  todas  las  permutaciones  de  
elementos  de  cada  conjunto.

17.  {1,  2} 18.  {1,  2,  3}

Encuentre  la  próxima  combinación  r  mayor  que  cada  combinación  del  conjunto  
correspondiente,  si  es  posible.

19.  124,  {1,  2,  3,  4} 20.  134,  {1,  2,  3,  4}

21.  1245,  {1,  2,  3,  4,  5} 22.  234,  {1,  2,  3,  4}

23.  2456,  {1,  2,  3,  4,  5,  6} 24.  3456,  {1,  2,  3,  4,  5,  6}

Con  el  algoritmo  de  la  siguiente  combinación,  genere  todas  las  combinaciones  r  del  
conjunto  {1,  2,  3, ... ,  n}  para  cada  par  de  valores  de  r  y  n.

25.  r  =  2,  n  =  3  26.  r  =  1,  n  =  3  27.  r  =  2,  n  =  4  28.  r  =  3,  n  =  4

29.  Recuerda  que  un  byte  es  una  palabra  de  8  bits.  Hay  256  bytes  y  se  pueden  organizar  
en  orden  lexicográfico:  00000000,  00000001,  00000010, ... ,  11111110,  11111111.  
Escriba  un  algoritmo  para  encontrar  el  byte  que  sigue  a  un  byte  dado  a1a2 ...  a8  en  
orden  lexicográfico.

30.  Una  palabra  ternaria  es  una  palabra  sobre  el  alfabeto  {0,  1,  2}.  Escribe  un  algoritmo  
para  encontrar  la  palabra  ternaria  que  sigue  a  la  palabra  ternaria  a1a2a3a4  en  orden  
lexicográfico.  Por  ejemplo,  la  palabra  ternaria  que  sigue  a  0222  es  1000.

Usando  el  algoritmo  de  la  siguiente  combinación,  encuentre  todos  los  subconjuntos  de  cada  conjunto.

31.  {1} 32.  {1,  2} 33.  {1,  2,  3} 34.  {1,  2,  3,  4}

A.5  El  teorema  multinomial

El  teorema  del  binomio  nos  permite  expandir  las  potencias  del  binomio  x  +  y.
Se  puede  reformular  de  una  manera  aparentemente  diferente  de  la  siguiente  manera:

norte norte

norte ¡norte!

(x  +  y)
norte
= xn−r  año  = xn−r  yr  
r r!(n  −  r)!
r=0 r=0
¡norte!

= xi  yj  
i,j≥0   i!j!
i+j=n
Machine Translated by Google

A.5  El  teorema  multinomial 889

¡norte!

El  coeficiente  binomial  se  puede  interpretar  como  el  número  de  formas  de  i!j!

dividir  un  conjunto  de  tamaño  n  en  dos  subconjuntos  disjuntos  de  tamaños  i  y  j.  Esta  sección  
generaliza  esta  idea:  Nos  gustaría  encontrar  la  expansión  de  (x1  +  x2  +  ∙∙∙+  xk)  n.

Teorema  del  trinomio
Para  ello,  consideremos  el  caso  especial  (x  +  y  +  z)  n.  Claramente,

(x  +  y  +  z) norte
=  (x  +  y  +  z)(x  +  y  +  z)∙∙∙(x  +  y  +  z) (A.1)

a  n  factores.  Por  lo  tanto,  cada  término  en  la  expansión  de  (x  +  y  +  z)  n  es  de  la  forma  Cxi  yj  zk ,  
donde  i,  j  y  k  son  números  enteros  no  negativos,  i  +  j  +  k  =  n,  y  C  es  el  coeficiente  a  ser  determinado.  
Así,  los  diversos  términos  se  obtienen  asignando  valores  a  i,  j  y  k,  de  modo  que  sean  números  
enteros  no  negativos  y  su  suma  sea  n.

El  coeficiente  C  es  el  número  de  formas  en  que  se  pueden  seleccionar  i  x,  j  y  y  k  z  de  los  n  
factores  en  la  RHS  de  la  Ecuación  (A.1).  Los  i  x  pueden  ser
norte

seleccionado  en maneras.  Ahora  quedan  n  −  i  factores.  Entonces  se  pueden  seleccionar  los  j  y
i
norte  
en maneras.  Ahora  quedan  n−i−j  factores.  Por  lo  tanto,  los  k  z  pueden  ser
­  ij
norte  ­  yo  ­  
seleccionado  en maneras.  Así,  por  el  principio  de  multiplicación,
jk

norte norte   norte  ­  yo  ­  


C  =
i ­  ij jk

¡norte! (n  −  i)!  j! (n  ­  yo  ­  j)!  k!


∙ ∙
=
yo!(n  −  yo)! (n  −  i  −  j)! (n  −  yo  −  j  −  k)!
¡norte!
= , ya  que  n  =  i  +  j  +  k.
i!j!k!0!
¡norte!

=
i!j!k!

Así  tenemos  el  siguiente  teorema.

TEOREMA  A.8  (Teorema  del  Trinomio)  Sean  x,  y,  yz  números  reales  cualesquiera,  yn  cualquier  número  entero.  Después

¡norte!

(x  +  y  +  z) norte
= xi  yj   (A.2)
zk  i!j!k!
i,j,k≥0  
i+j+k=n

EJEMPLO  A.19  Usando  el  teorema  del  trinomio,  desarrolle  (x  +  y  +  z)  2.
Machine Translated by Google

890 Apéndice  A

SOLUCIÓN:
Por  el  teorema  del  trinomio,

2 = 2!
(x  +  y  +  z) xi  yj  
i,j,k≥0   zk  i!j!k!
i+j+k=2

Para  encontrar  los  diversos  términos  en  el  desarrollo,  listemos  las  posibles  combinaciones  
de  valores  de  i,  j  y  k,  y  los  coeficientes  y  términos  correspondientes  en  una  tabla,  como  en  
la  Tabla  A.3.

Tabla  A.3 2!
jk xiyjzk Término
i!j!k!  
2!
200 =  1 x2 x2
2!0!0!
2!
020 =  1 y2 y2
0!2!0!
2!
002 =  1 z2 z2
0!0!2!
2!
110 =  2 xy 2xy
1!1!0!
2!
011 =  2 yz 2yz
0!1!1!
2!
101 =  2 zx 2zx
1!0!1!

Sumando  los  términos  de  la  última  columna,
2
(x  +  y  +  z) =  x2  +  y2  +  z2  +  2xy  +  2yz  +  2zx

EJEMPLO  A.20  Encuentre  el  coeficiente  de  x2y3z4  en  la  expansión  de  (x  +  y  +  z)  9.

SOLUCIÓN:  
Aquí  i  =  2,  j  =  3,  k  =  4  y  n  =  9.  Por  lo  tanto,  el  coeficiente  requerido  está  dado  por

¡norte! 9!
= =  1260
i!j!k! 2!3!4!

La  demostración  utilizada  para  desarrollar  el  teorema  del  trinomio  se  puede  generalizar  
a  k  variables  x1,  x2, ... ,  xk  de  forma  obvia.  El  teorema  resultante,  llamado  teorema  
multinomial,  se  da  a  continuación.

TEOREMA  A.9  (El  Teorema  Multinomial)  Sean  x1,  x2, ... ,  xk  cualquier  k  variable  real  y  n  cualquier  entero  no  
negativo.  Después

¡norte! i1 i2 yo
= X
1 X 2 ...X
norte

(x1  +  x2  +∙∙∙+  xk) k
i1!i2! ...  ik!

donde  i1,  i2, ... ,ik  ≥  0  e  i1  +  i2  +∙∙∙+  ik  =  n.
Machine Translated by Google

A.5  El  teorema  multinomial 891

EJEMPLO  A.21  Usando  el  teorema  multinomial,  encuentre  el  coeficiente  de  x1x2  2x3  3x4  4x5  5  en

la  expansión  de  (x1  +  x2  +  x3  +  x4  +  x5)  15.

SOLUCIÓN:

¡15!
Coeficiente  requerido  =  1!2!3!4!
5!

=  37.837.800

¡norte!

el  coeficiente en  el  teorema  multinomial,  llamado
i1!i2! ...  ik!
norte

coeficiente  multinomial,  se  denota  por
i1,i2, ... ,ik .  Por  ejemplo,

5!
= =  20  3!
5  3,  0,  1,  1 0!1!1!

norte

Así  como  el  coeficiente  binomial denota  el  número  de  formas  en  que  un  conjunto  de
i,  j  
tamaño  n  se  puede  dividir  en  dos  subconjuntos  disjuntos  de  tamaños  i  y  j  =  n  −  i,  el  coeficiente  
multinomial  también  se  puede  interpretar  de  manera  similar,  como  se  establece  en  el  siguiente  teorema.

TEOREMA  A.10  El  número  de  formas  de  dividir  un  conjunto  S  de  tamaño  n  en  k  subconjuntos  ordenados  mutuamente  disjuntos  
S1,  S2, ... ,  Sk  de  tamaños  i1,  i2, ... ,ik,  respectivamente,  viene  dado  por  el  multinomio  coeficiente

norte

i1,i2, ... ,ik

donde  i1,  i2, ...ik  ≥  0  e  i1  +  i2  +∙∙∙+  ik  =  n.

PRUEBA:
norte

Los  elementos  i1  del  subconjunto  S1  se  pueden  seleccionar  en maneras.  esto  deja
i1  
n  −  i1  elementos  en  S  −  S1.  Por  lo  tanto,  los  elementos  i2  de  S2  se  pueden  seleccionar
norte  ­  
en maneras.  De  manera  similar,  los  elementos  i3  de  S3  se  pueden  seleccionar  en
i1  i2  
norte  ­  i1  ­  i2  
maneras,  y  así  sucesivamente.  Los  elementos  ik  de  Sk  se  pueden  elegir  en
i3  norte  
­  i1  ­  i2  ­  ik ... −  ik−1
maneras.  En  consecuencia,  por  el  principio  de  multiplicación

principio,  el  número  de  formas  de  elegir  los  subconjuntos  mutuamente  disjuntos  S1,  S2, ... ,  Sk  es

norte
norte  ­   norte  ­  i1  ­  i2   ∙∙∙ norte  −  i1  −  i2  −∙∙∙−  ik−1  ik
i1 i1  i2 i3
Machine Translated by Google

892 Apéndice  A

¡norte!
∙ (n  −  i1)!   ∙∙∙ (n  −  i1  −∙∙∙ik−1)  ik!(n  
=
i1!(n  −  i1)! i2!(n  −  i1  −  i2)! −  i1  −∙∙∙−  ik)!
¡norte!

= (Nota :  n  =  i1  +  i2  +∙∙∙+  ik)
i1!i2! ...  ik!

=
norte

i1,i2, ... ,ik

Esto  concluye  la  prueba.

EJEMPLO  A.22  Encuentre  el  número  de  maneras  de  dividir  un  conjunto  de  tamaño  cinco  en  tres  mutuamente
subconjuntos  ordenados  disjuntos  de  tamaños  2,  1  y  2.

SOLUCIÓN:  Por  
el  Teorema  A.10,  el  número  de  formas  de  dividir  el  conjunto  en  la  forma  deseada  viene  dado  
por  el  coeficiente  multinomial

5!
=
5  2,  1,  2 2!1!2!

=  30

EJEMPLO  A.23  Encuentre  el  número  de  formas  de  dividir  el  conjunto  S  =  {a,  b,  c}  en  tres  subconjuntos  S1,  S2  y  S3  
ordenados  mutuamente  disjuntos  de  tamaños  1,  1  y  1,  respectivamente,  y  anótelos.

SOLUCIÓN:
3!
Número  de  posibilidades  = = =  6  
3  1,  1,  1 1!1!1!

Se  enumeran  en  la  Tabla  A.4.

Tabla  A.4
S1 S2 S3
{a}   {b}   {c}  
{a}   {c}   {b}  
{b}   {a}   {c}  
{b}   {c}   {a}  
{c}   {a}   {b}  
{c} {b} {a}

Una  observación  importante:  La  división  {{a},  {b},  {c}}  es  diferente  de
la  división  {{a},  {c},  {b}}.  Sin  embargo,  forman  la  misma  partición.

Ejercicios  A.5

Evalúa  cada  coeficiente  multinomial.

1. 2. 3. 4.
5  2,  2,  1 6  1,  2,  3 7  2,  3,  2 8  0,  3,  5
Machine Translated by Google

A.5  El  teorema  multinomial 893

10   10   10  


5. 6. 7. 8.
8  2,  1,  3,  2 1,  2,  3,  4 3,  1,  4,  2 1,  4,  4,  1

Encuentre  el  coeficiente  de  cada  uno.

5
9.  x2yz2  en  el  desarrollo  de  (x  +  y  +  z)  10.  xy2z3  
6
en  el  desarrollo  de  (x  +  y  +  z)  11.  yz  en  el  desarrollo  
2
de  (x  +  y  −  z)  12.  xy2  en  el  desarrollo  de  ( x  −  y  −  z)  
3
13.  xy2z5  en  el  desarrollo  de  (x  +  y  +  z)  14.  xy3z3  
8
en  el  desarrollo  de  (x  −  y  +  2z)
8

Ampliar  cada  uno.

15.  (x  +  y  −  z)2 2  16.  (x  ­  y  ­  z) 3  17.  (x  −  y  +  z)

3  18.  (x  ­  y  ­  z) 3  19.  (x  +  2y  +  z) 3  20.  (x  −  2y  +  2z)

Considere  las  diversas  formas  de  dividir  el  conjunto  S  =  {a,  b,  c,  d}  en  tres  subconjuntos  
ordenados  mutuamente  disjuntos  S1,  S2  y  S3  con  tamaños  1,  1  y  2,  respectivamente.

21.  Encuentra  el  número  de  posibilidades.

22.  Haz  una  lista  de  las  posibilidades.

Encuentre  el  número  de  formas  de  dividir  un  conjunto  de  tamaño  n  en  k  subconjuntos  ordenados  
mutuamente  disjuntos  de  tamaños  i1,  i2, ... ,ik  en  cada  caso.

23.  n  =  10,  tres  subconjuntos  con  tamaños  2,  3  y  5.  24.  n  =  

10,  cuatro  subconjuntos  con  tamaños  2,  2,  3  y  3.  25.  n  =  12,  

seis  subconjuntos  con  tamaños  1,  1,  2,  2,  3  y  3.

26.  n  =  15,  cuatro  subconjuntos  con  tamaños  1,  1,  5  y  8.
norte

27.  Encuentra  la  suma  de  los  coeficientes  multinomiales
i1,i2, ... ,ik .

Por  el  teorema  A.10  se  sigue  que  el  número  de  permutaciones  de  n  elementos  de  los  cuales  i1  
son  iguales,  i2  son  iguales, ...  e  ik  son  iguales  está  dado  por  el
norte

coeficiente  multinomial
i1,i2, ... ,ik .  Usando  este  hecho,  calcule  cada  uno.

28.  La  cantidad  de  formas  de  codificar  las  letras  de  la  palabra  ABRA
CADABRA.

29.  La  cantidad  de  formas  de  codificar  las  letras  de  la  palabra
TINTINNABULACIÓN.

30.  El  número  de  palabras  binarias  de  longitud  10  y  que  contienen  exactamente  tres
1  y  siete  0.
Machine Translated by Google

894 Apéndice  A

31.  El  número  de  bytes  que  contienen  exactamente  cinco  ceros.

32.  El  número  de  palabras  ternarias  de  longitud  12  y  que  contengan  tres  ceros,
cuatro  1  y  cinco  2.

Encuentra  el  número  de  términos  en  la  expansión  de  cada  uno.  
3 10
33.  (x  +  y  +  z)  35.  (w  +  x  +  y  +  z) 34.  (x  +  y  +  z)  36.  
12 15
(2w  −  3x  +  4y  −  4z)
Encuentre  el  coeficiente  de  cada  uno.
7
37.  x2y2z3  en  el  desarrollo  de  (x  +  y  +  z)  38.  xy3z  
5
en  el  desarrollo  de  (x  +  2y  −  z)

Usando  el  teorema  multinomial,  expanda  cada  uno.

3  39.  (x  −  2y  +  z) 4  40.  (x  +  y  −  z)

Encuentra  el  número  de  términos  en  la  expansión  de  cada  uno.

6  41.  (x  +  y  −  z) 8  42.  (x  ­  y  ­  z)

9  43.  (2x  +  3y  −  4z) 10  44.  (w  +  x  −  y  −  z)

Halla  el  número  de  formas  de  dividir  un  conjunto  de  tamaño  15  en  cada  una.

45.  Cuatro  subconjuntos  mutuamente  disjuntos  con  tamaños  7,  2,  3  y  3.

46.  Cinco  subconjuntos  mutuamente  disjuntos  con  tamaños  3,  6,  2,  1  y  3.

A.6  El  Alfabeto  Griego

A α Alfa   norte v nu
B β   Beta ξ xi  
γ   gamma   O o omicron  
δ delta   π pi  rho  
mi épsilon   PAG
ρ sigma  tau
Z ζ zeta σ
H η   eta T τ
θ theta  upsilon  iota  phi  kappa  chi  λ  vlambdaψ  psi  omega
yo τ φ
k k X x

METRO
m mu ω
Machine Translated by Google

A.7  Sitios  web 895

A.7  Sitios  web

Los  siguientes  sitios  web  proporcionan  información  valiosa  para  una  mayor  exploración  
y  enriquecimiento.  El  estado  de  un  sitio  web  puede  cambiar  con  el  tiempo,  por  lo  que  es  
posible  que  no  exista  cuando  lo  busque;  si  no  es  así,  utilice  un  motor  de  búsqueda  para  
localizar  un  sitio  Web  similar.

1.  Wilhelm  Ackermann
www­gap.dcs.st­and.ac.uk/ history/Mathematicians/  
Ackermann.html  2.  John  Backus  www.digitalcentury.com/
encyclo/update/backus.html  www­gap.dcs.st­and.ac .uk/ history/
Mathematicians/Backus.html  www.acm.org/awards/
turing_citations/backus.html  www.cs.nyu.edu/cs/faculty/shashaoutofmind/
backus.html  3.  George  Boole  www­gap.dcs .st­and.ac.uk/ history/
Mathematicians/Boole.html  www.digitalcentury.com/encyclo/update/boole.html  
homepages.enterprise.net/rogerp/george/boole.html  4.  Georg  Cantor  www­
groups .dcs.st­and.ac.uk/ history/Mathematicians/Cantor.html  www.treasure­
troves.com/bios/CantorGeorg.html  www.aug.edu/dvskel/JohnsonSU97.html  5.  
Arthur  Cayley  www­gap .dcs.st­and.ac.uk/ history/Mathematicians/Cayley.html  
scienceworld.wolfram.com/biography/Cayley.html  www.stetson.edu/ efriedma/
periodictable/html/C.html  www.optisyn.  com/presentations/Cayley.html  
www.math.ukans.edu/ engheta/bio/cayley.html  www.geometry.net/Biographer/
Cayley.html  6.  Edsger  Dijkstra  www.digidome.nl/edgser_wybe_dijkstra.html  
www.ac  m.org/classics/oct95/  www.cs.utexas/users/EWD/obituary.html  news.com.com/
2100­1001­949023.html

7.  Leonard  Euler
www.maths.tcd.ie/pub/HistMath/People/Euler/RouseBall/  
RB_Euler.html  www.shu.edu/html/teaching/math/reals/history/
euler.html
8.  Pierre  de  Fermat
www.maths.tcd.ie/pub/HistMath/People/Fermat/RouseBall/  
RB_Fermat.html  9.  Karl  F.  Gauss

www­gap.dcs.st­and.ac.uk/ history/Mathematicians/Gauss.html  
www.english.upenn.edu/ jlynch/FrankenDemo/People/gauss.html  
scienceworld.wolfram.com/biography/Gauss.  html  www.indiana.edu/ intell/
gauss.html  www.brown.edu/Students/OHJC/hm4/gauss.html
Machine Translated by Google

896 Apéndice  A

10.  Christian  Goldbach
www­gap.dcs.st­and.ac.uk/ history/Mathematicians/Goldbach.html  
www.geocities.com/Heartland/Hills/7972/math­goldbach.html  
www.wikipedia.org/wiki/Christian_Goldbach  más.  maths.org/issue11/
news/Goldbach  11.  Sir  William  R.  Hamilton

www.chembio.uoguelph.ca/educmat/chm386/rudiment/tourclas/  
hamilton.html  scienceworld.wolfram.com/biography/
HamiltonWilliamRowan.html  www.iaste.com/hall_of_fame/hamilton.html

12.  Mauricio  Karnaugh
www.informatik.uni­trier.de/ ley/db/indices/a­tree/k/Karnaugh:  
Maurice.html  www.maxmon.com/library.html  www­cse.stanford.edu/
classes/cs103a/h9BooleanAlgebra .pdf  13.  Alfred  B.  Kempe  www­
gap.dcs.st­and.ac.uk/ history/Mathematicians/Kempe.html  
mappa.mundi.net/locus_014  www.uwinnipeg.ca/ ooellerm/guthrie/
FourColor.  html  www.mathsyear2000.org/explorer/morphing/
13usedownload.shtml  14.  Stephen  C.  Kleene  www.library.wisc.edu/
libraries/Math/kleen.html  www­groups.dcs.st­and.ac.uk/   history/
Mathematicians/Kleene.html  www.dcs.ed.ac.uk/homes/als/lics/
newsletters/19.html  www.student.math.uwaterloo.ca/ cs462/Hall/
kleene.html  15.  Donald  E. .Knuth

sunburn.stanford.edu/ knuth  
www­cs­staff.stanford.edu/ knuth/index.html  
www.digitalcentury.com/encyclo/update/knuth.html  
laurel.actlab.utexas.edu/ cynbe/muq/  muf3_20.html
16.  Kazimierz  Kuratowski
www­groups.dcs.st­and.ac.uk/ history/Mathematiticains/  
Kuratowski.html  www.stetson.edu/ efriedma/periodictable/
html/Kr.html  17.  Gabriel  Lame  www­groups.dcs.st­  
and.ac.uk/ history/Mathematicians/Lame.html  www.bath.ac.uk/
ma0dmp/Lamelife.html

18.  Edmundo  Landau
www.ma.huji.ac.il/ landau/landuniv.html  
www­groups.dcs.st­and.ac.uk/ history/Mathematicians/Landau.html  
www.ard.huji.ac.il/publications/  25años/chap13.html
19.  Pierre­Simon  Laplace
www­groups.dcs.st­and.ac.uk/ history/Mathematicians/Laplace.html  
www.maths.tcd.ie/pub/HistMath/People/Laplace/RouseBall/  
RB_Laplace.html  www.  stetson.edu/ efriedma/periodictable/html/
La.html
Machine Translated by Google

A.7  Sitios  web 897

20.  Gottfried  W.  Leibniz  
www.maths.tcd.ie/pub/HistMath/People/Leibniz/RouseBall/  
RB_Leibniz.html  21.  Jan  Lukasiewicz  www­groups.dcs.st­and.ac.uk/
history/Mathematiticains/  Lukawiewicz.html  www.hpmuseum.org/rpn.html  
www.wikipedia.com/wiki/Jan_Lukawiewicz  22.  George  H.  Mealy  
www.informatik.uni­trier.de/ ley/db/indices/a­tree/m/  Mealy:  
George_H=.html  23.  Marin  Mersenne  www­groups.dcs.st­and.ac.uk/
history/Mathematiticains/  Mersenne.html  www2.andrews.edu/ calkins/
math/biograph/biomerse.html  24.  Blaise  Pascalmembers.aol.com/KatherenaE/
private/Philo/Pascal/pascal.html  www.maths.tcd.ie/pub/HistMath/People/
Pascal/RouseBall/  RB_Pascal.html  www.cs.washington.edu/homes/  
jbaer/classes/blaise/blaise.html  www.math.sfu.ca/histmath/Europe/
17thCenturyAD/Balise.html  www­groups.dcs.st­and.ac.uk/ history/
Mathematiticains/  Pascal.html  25.  Bertrand  Russell

www.mcmaster.ca/russdocs/russell.html  
plato.stanford.edu/entries/russell  
desktop12.cis.macmaster.cal/ bertrand  26.  
Claude  E.  Shannon  www­groups.dcs.st­and.ac.uk/  
history/Mathematiticains/  Shannon.html  www.bell­labs.com/news/
2001/february/26/1.html  www.digitalcentury.com/encyclo/update/
shannon.html  27.  Alan  M.  Turing  www.turing.  org.uk/turing  ei.cs.vt.edu/
history/Turing.html  www­groups.dcs.st­and.ac.uk/ history/
Mathematiticains/  Turing.html  28.  Alexandre­Theophile  Vandermonde  www­
groups .dcs.st­and.ac.uk/ history/Mathematiticains/  Vandermonde.html  
29.  John  Venn  www­groups.dcs.st­and.  ac.uk/ history/Mathematicians/
Venn.html  userwww.sfsu.edu/ rsauzier/Venn.html  30.  Álgebra  
booleana  educ.queensu.ca/ compsci/units/BoolLogic/titlepage.html  
www.bit.umkc.  edu/vu/course/cs281/lectures/boolean­algebra/boolean­
algebra.html
Machine Translated by Google

898 Apéndice  A

ever.phys.ualberta.ca/ gingrich/phys395/notes/node121.html  
www.maxmon.com/1847ad.html  www.yale.edu/ynhti/curriculum/
units/1989/7/89.07.07.x.html  32.  Máquinas  de  estados  finitos

www.cs.brown.edu/people/jes/book/BOOK/node10.html  
classwww.gsfc.nasa.gov/CAGESite/pages/cage_gpf_fsm.html  
www.cs.arizona.edu/classes/cs352/summer02/fsa.  html  
www.c3.lanl.gov/mega­math/workbk/machine/mabkgd.html  
www.beigarath.demon.co.uk/java/fsme.htmlmembers.aol.com/
asakharov/fsm.html  33.  Conjuntos  borrosos  
www.answermath.com/fuzzymath.html  sun16.cecs.missouri.edu/
index.html  news:comp.ai.fuzzy  34.  Funciones  generadoras  
www.cs.wpi.edu/ cs504/s00m/notes/ln/1999/  class06/
class06.html  www.cs.wpi.edu/ cs504/s00m/classes/class05/
Class05.html  msl.cs.uiuc.edu/ lavalle/cs576/projects/wmchan  
www.eco.rug.nl/gauss /GAUSS00/mhonarc.db  
www.maths.surrey.ac.uk/personal/st/d.fisher/MS103/MS105­5.mws  
35.  Teoría  de  grafos  www.utm.edu/departments/math/graph  
www.c3.  lanl.gov/mega­math/workbk/graph/graph.html  www­
groups.dcs.st­and.ac.uk/ history/Mathematicians/Erdos.html  
www.nada.kth.se/ viggo/problemlist/  compendium.html  www.shodor.org/
interactive/lessions/frac1.html  www.scism.sbu.ac.uk/law/Section3/
chapter3/s3c3int.html  w  ww.cs.cmu.edu/ cburch/survey/recurse  
www.nd.edu/ cholak/computability/computability.html  36.  Paradojas  
de  Hilbert  www.wordsmith.demon.co.uk/paradoxes  eluzions.com/
Puzzles/Logic /Paradoxes.shtml  www.c3.lanl.gov/mega­math/workbk/
infinity/infinity.html  www.cs.tpu.ee/ jaagup/uk/fmm/math/1.2.5.3.html  
37.  Los  cuatro  Problema  de  color  www.math.gatech.edu/ thomas/
FC/fourcolor.html  www.cs.uidaho.edu/ casey931/mega­math/gloss/
math/4ct.html  www­groups.dcs.st­and.  ac.uk/ history/HistTopics/
The_four_colour  _theorem.html
Machine Translated by Google

Referencias

1.  AV  Aho  et  al.,  Compiladores:  principios,  técnicas  y  herramientas,  Addison
Wesley,  Lectura,  MA  1986.
2.  VS  Alagar,  Fundamentos  de  computación:  teoría  y  práctica,  Prentice­Hall,
Acantilados  de  Englewood,  Nueva  Jersey,  1989.

3.  K.  Appel  y  W.  Haken,  "Cada  gráfico  plano  tiene  4  colores",  Boletín  de  la  Sociedad  Matemática  
Estadounidense,  vol.  82  (septiembre  de  1976),  págs.  711–712.
4.  RG  Archibald,  Introducción  a  la  teoría  de  los  números,  3.ª  edición,  Wiley,  Nueva  York,  1972.

5.  V.  Bain,  "Un  algoritmo  para  dibujar  el  cubo  n",  The  College  Mathematics
Revista,  vol.  29  (septiembre  de  1998),  págs.  320–322.
6.  C.  Baltus,  “Una  mesa  de  la  verdad  en  la  isla  de  los  que  dicen  la  verdad  y  los  mentirosos”,
Profesor  de  Matemáticas,  vol.  94  (diciembre  de  2001),  págs.  730–732.
7.  WJ  Barneir,  “Finite­State  Machines  as  Recognizers”,  The  UMAP  Journal,  7:3  (1986),  págs.  209–
232.
8.  W.  Barnier  y  JB  Chan,  Matemática  discreta  con  aplicaciones,  West,
San  Pablo,  MN,  1989.
9.  B.  Barwell,  Solución  al  problema  702,  J.  Matemáticas  recreativas,  vol.  12:1  (1979–1980),  pág.  67–
68.
10.  B.  Barwell,  Solución  al  problema  1046,  J.  Matemáticas  recreativas,  vol.  15:1
(1981–1982),  págs.  70–72.
11.  R.  Bellman  et  al.,  Algorithms,  Graphs,  and  Computers,  Academic  Press,  Nueva  York,  1970.

12.  M.  Bellmore  y  GL  Nemhauser,  “El  problema  del  viajante  de  comercio”,
Investigación  de  operaciones,  vol.  16  (1968),  págs.  538–558.
13.  SJ  Bezsuska,  Solution  to  Problem  791,  J.  Recreational  Mathematics,  12:4  (1979–1980),  pág.  311.

14.  B.  Bissinger,  “Ask  Marilyn”,  Parade  Magazine  (25  de  abril  de  1993),  pág.  15.
15.  ML  Bittinger,  Lógica  y  prueba,  Addison­Wesley,  Reading,  MA  1972.
16.  WG  Brown,  "Nota  histórica  sobre  un  problema  combinatorio  recurrente",  The  American  Mathematical  
Monthly,  vol.  72  (noviembre  de  1965),  págs.  973–977.

899
Machine Translated by Google

900 Referencias

17.  AV  Boyd  y  MJ  Glencorss,  "Disecting  a  Circle  by  Chords  through  n  Points",  Mathematics  Teacher,  vol.  
84  (abril  de  1991),  págs.  318  y  319.
18.  J.  Burling  et  al.,  "Uso  de  gráficos  para  resolver  el  problema  del  semáforo",  FAIM
Módulo,  COMAP,  Inc.,  Lexington,  MA,  1989.
19.  D.  Burns,  Problema  596,  J.  Matemáticas  recreativas,  vol.  10:1  (1977–1978),
pag.  52.

20.  WH  Bussey,  “Origen  de  la  inducción  matemática”,  The  American  Mathematical  Monthly,  vol.  24  (mayo  
de  1917),  págs.  199–207.
21.  Problemas  de  calendario,  profesor  de  matemáticas,  vol.  83  (octubre  de  1990),  pág.  550.
22.  Problemas  de  calendario,  profesor  de  matemáticas,  vol.  85  (diciembre  de  1992),  pág.  736.
23.  Problemas  de  calendario,  profesor  de  matemáticas,  vol.  79  (abril  de  1986),  pág.  274.
24.  Problemas  de  calendario,  profesor  de  matemáticas,  vol.  79  (noviembre  de  1986),  pág.  627.
25.  DM  Campbell,  “La  computación  de  los  números  catalanes”,  Matemáticas
revista,  vol.  57  (septiembre  de  1984),  págs.  195–208.
26.  L.  Carlitz,  Solución  al  problema  B­180,  The  Fibonacci  Quarterly,  vol.  8:5  (diciembre
1970),  págs.  547–548.
27.  M.  Caudill,  "Uso  de  redes  neuronales:  decisiones  difusas",  AI  Expert,  vol.  5  (abril  de  1990),  págs.  59  a  
64.
28.  M.  Charosh,  Problema  1160,  J.  Matemáticas  recreativas,  vol.  15:1  (1983–
1984),  pág.  58.
29.  EF  Codd,  "Un  modelo  relacional  de  datos  para  grandes  bancos  de  datos  compartidos",
Comunicaciones  de  la  ACM,  vol.  13  (1970),  págs.  377–387.
30.  F.  Cohen  y  JL  Selfridge,  "No  todos  los  números  enteros  son  la  suma  o  la  diferencia  de  dos  potencias  
principales",  Matemáticas  de  computación,  vol.  29  (1975),  pág.  79.
31.  E.  Comfort,  Solución  al  Problema  596,  J.  Matemáticas  Recreativas,  vol.  11:1
(1978–1979),  pág.  66.
32.  JW  Cortada,  Diccionario  histórico  de  procesamiento  de  datos:  biografías,  Greenwood  Press,  Nueva  
York,  1987.
33.  M.  Coughlin  y  C.  Kerwin,  "La  inducción  matemática  y  el  problema  de  los  puntos  de  Pascal",  Profesor  
de  matemáticas,  vol.  78  (mayo  de  1985),  págs.  376–380.
34.  T.  Crilly,  "Un  matemático  victoriano",  The  Mathematical  Gazette,  vol.  79
(julio  de  1995),  págs.  259–262.
35.  P.  Cull  y  EF  Ecklund,  Jr.,  "Torres  de  Hanoi  y  análisis  de  algoritmos",
El  mensual  matemático  estadounidense,  vol.  92  (junio­julio  de  1985),  págs.  407­420.
36.  N.  Deo,  Teoría  de  grafos  con  aplicaciones  a  la  ingeniería  y  la  informática,
Prentice  Hall,  Englewood  Cliffs,  Nueva  Jersey,  1974.
37.  RC  Drake,  Problema  B­180,  The  Fibonacci  Quarterly,  vol.  8:1  (febrero  de  1970),
pag.  106.

38.  LR  Duffy,  "Los  números  de  Duffinian",  J.  Matemáticas  recreativas,  vol.  12:2
(1979–1980),  págs.  112–115.
39.  R.  Euler,  Problema  1551,  J.  Matemáticas  recreativas,  vol.  19:2,  1987,  pág.  151.
40.  H.  Eves,  Problema  E579,  The  American  Mathematical  Monthly,  vol.  50  (junio–
julio  de  1943),  pág.  386.
41.  H.  Eves,  Introducción  a  la  historia  de  las  matemáticas,  3.ª  edición,  Holt,
Rinehart  y  Winston,  Nueva  York,  1969.
Machine Translated by Google

Referencias 901

42.  A.  Filz,  Problema  1046,  J.  Matemáticas  recreativas,  vol.  14:1  (1981–1982),
pag.  64.

43.  T.  Fletcher,  Problema  602,  J.  Matemáticas  recreativas,  vol.  10:1  (1978–1979),
pag.  52.

44.  HG  Forder,  "Algunos  problemas  de  combinatoria",  The  Mathematical  Gazette,
vol.  45  (1961),  págs.  199–201.
45.  AJ  Friedland,  Puzzles  in  Math  &  Logic,  Dover,  Nueva  York,  1970.
46.  JA  Gallian  y  S.  Winters,  "Aritmética  modular  en  el  mercado",  The  American  Mathematical  Monthly,  
vol.  95  (junio–julio  de  1988),  págs.  548–551.
47.  M.  Gardner,  "Juegos  matemáticos",  Scientific  American,  vol.  219  (septiembre
1968),  págs.  218–230.
48.  M.  Gardner,  "Números  catalanes:  una  secuencia  entera  que  se  materializa  en  lugares  
inesperados",  Scientific  American,  vol.  234  (junio  de  1976),  págs.  120–125.
49.  M.  Gardner,  Mathematical  Circus,  Knopf,  Nueva  York,  NY,  1979.
50.  M.  Gardner,  Mathematical  Puzzles  and  Diversions,  Universidad  de  Chicago
Prensa,  Chicago,  IL,  1987.
51.  M.  Gardner,  “Ask  Marilyn”,  Parade  Magazine  (18  de  abril  de  1993),  pág.  12
52.  SW  Golomb,  "Emparejamientos  y  agrupaciones",  Revista  Johns  Hopkins,  vol.  45:2  (abril  de  1993),  
pág.  7.
53.  RL  Graham  et  al.,  Concrete  Mathematics,  Addison­Wesley,  Reading,  MA,
1990.

54.  JJ  Gray,  "Arthur  Cayley  (1821–1895)",  The  Mathematical  Intelligencer,
vol.  17:4  (1995),  págs.  62–63.
55.  TM  Green,  "Pascal's  Pizza",  Mathematics  Teacher,  vol.  81  (septiembre  de  1988),
pag.  445,  454.
56.  RP  Grimaldi,  Matemáticas  discretas  y  computacionales,  4.ª  ed.,  Addison
Wesley,  Lectura,  MA,  1999.
57.  A.  Guckin  et  al.,  El  proyecto  del  circuito  de  Euler,  COMAP,  Inc.,  Lexington,  MA,
1989.

58.  S.  Gudder,  Un  viaje  matemático,  McGraw­Hill,  Nueva  York,  1976.
59.  B.  Hamilton,  Brainteasers  and  Mindbenders,  Freeside,  Nueva  York,  1992.
60.  DK  Hanson  et  al.,  "Matching,  Derangements,  and  Rencontres",
Revista  de  Matemáticas,  vol.  56  (septiembre  de  1983),  págs.  224–229.
61.  F.  Harray  y  JS  Maybee  (eds.),  Graphs  and  Applications,  Wiley,  Nueva  York,
1985.

62.  B.  Hayes,  "Sobre  la  máquina  de  estados  finitos,  un  modelo  mínimo  de  ratoneras,  ribosomas  y  el  
alma  humana",  Scientific  American,  vol.  249  (diciembre  de  1983),  págs.  20–28,  178.

63.  L.  Henkin,  "Sobre  la  inducción  matemática",  The  American  Mathematical
Mensual,  vol.  67  (abril  de  1960),  págs.  323–338.
64.  VE  Hoggatt,  Jr.,  Números  de  Fibonacci  y  Lucas,  Houghton  Mifflin,  Boston,
1963.

65.  VE  Hoggatt,  Jr.  y  SL  Basin,  “A  Primer  on  the  Fibonacci  Sequence,  Part  II”,  The  Fibonacci  
Quarterly,  vol.  1:2  (abril  de  1963),  págs.  61–68.
Machine Translated by Google

902 Referencias

66.  VE  Hoggatt,  Jr.,  "Algunas  funciones  especiales  de  generación  de  Fibonacci  y  Lucas"
El  trimestral  de  Fibonacci,  vol.  9:2  (abril  de  1971),  págs.  121–133.
67.  TC  Hu,  Algoritmos  combinatorios,  Addison­Wesley,  Reading,  MA,  1982.
68.  KK  Huang,  Solución  al  problema  1160,  J.  Matemáticas  recreativas,  vol.  16:1
(1983–1984),  pág.  69.
69.  RV  Jean,  "La  secuencia  de  Fibonacci",  The  UMAP  Journal,  vol.  5:1  (1984),
págs.  23–47.
70.  JT  Johnson,  "Lógica  difusa",  Popular  Science,  vol.  237  (julio  de  1990),  págs.
87–89.

71.  R.  Johnsonbaugh,  Matemáticas  discretas,  5.ª  ed.,  Prentice­Hall,  Upper  Saddle  River,  NJ,  2001.

72.  E.  Just,  “Una  nota  sobre  el  término  n  de  la  sucesión  de  Fibonacci”,  Matemáticas
revista,  vol.  44  (septiembre­octubre  de  1971),  pág.  199.
73.  M.  Karnaugh,  "El  método  del  mapa  para  la  síntesis  de  circuitos  lógicos  combinacionales",  
Transactions  of  the  AIEE,  Part  I,  vol.  72:9  (noviembre  de  1953),  págs.  593–599.

74.  FH  Kierstead,  Jr.,  Problema  791,  J.  Matemáticas  recreativas,  vol.  11:4
(1978–1979),  pág.  302.
75.  FH  Kierstead,  Jr.,  Problema  1014,  J.  Matemáticas  recreativas,  vol.  14:4
(1981–1982),  pág.  309.
76.  M.  Keith  y  T.  Carver,  “The  Ultimate  Perpetual  Calendar”,  J.  Recreational
Matemáticas,  vol.  22:4  (1990),  págs.  280–282.
77.  DE  Knuth,  "Algoritmos",  Scientific  American,  vol.  243  (abril  de  1977),  págs.
63–80.

78.  DE  Knuth,  "Pensamiento  algorítmico  y  pensamiento  matemático",  The  American  Mathematical  
Monthly,  vol.  92  (marzo  de  1985),  págs.  170–181.
79.  B.  Kolman  et  al.,  Discrete  Mathematical  Structures,  4.ª  ed.,  Prentice­Hall,  Upper  Saddle  River,  NJ,  
2000.
80.  T.  Koshy,  Matemáticas  finitas  y  cálculo  con  aplicaciones,  Goodyear,
Pacific  Palisades,  California,  1979.
81.  T.  Koshy,  Fibonacci  and  Lucas  Numbers  with  Applications,  Wiley,  Nueva  York,
2001.

82.  T.  Koshy,  Elementary  Number  Theory  with  Applications,  Harcourt/Academic  Press,  Boston,  2002.

83.  B.  Kosko  y  S.  Isaka,  "Fuzzy  Logic",  Scientific  American,  vol.  269  (julio  de  1993),  págs.  76–81.

84.  JB  Kruskal,  “On  the  Shortest  Spanning  Subtree  of  a  Graph  and  the  Traveling  Salesman  Problem”,  
Actas  de  la  American  Mathematical  Society,  vol.  1  (1956),  págs.  48–50.

85.  EA  Kuehls,  "El  valor  de  verdad  de  { ,   ,  P  (x,  y)}:  un  enfoque  gráfico"


Profesor  de  Matemáticas,  vol.  43  (noviembre  de  1970),  págs.  260–261.
86.  NJ  Kuenzi  y  B.  Prielipp,  Problema  4026,  School  and  Science  Mathematics,  vol.  85  (diciembre  de  
1985),  págs.  714–716.
87.  LJ  Lander  et  al.,  "Una  encuesta  de  sumas  iguales  de  potencias  similares",  Matemáticas  de  
computación,  vol.  21  (1967),  pág.  446.
Machine Translated by Google

Referencias 903

88.  EL  Lawler  et  al.  (eds.),  El  problema  del  viajante  de  comercio,  Wiley,  Nueva  York,
1986.

89.  CT  Long,  "On  Pigeons  and  Problems",  Mathematics  Teacher,  vol.  81  (enero  de  1988),  págs.  28–
30,  64.
90.  E.  Maier,  "Contar  piezas  de  pizza  y  otros  problemas  combinatorios",
Profesor  de  Matemáticas,  vol.  81  (enero  de  1988),  págs.  22–26.
91.  CL  Mallows,  “Secuencia  de  desafío  de  Conway”,  The  American  Mathematical
Mensual,  vol.  98  (enero  de  1991),  págs.  5  a  20.
92.  LE  Mauland,  "Un  ejercicio  con  números  poligonales",  profesor  de  matemáticas,  vol.  78  (mayo  de  
1985),  págs.  340–344.
93.  SB  Maurer  y  A.  Ralston,  Matemática  algorítmica  discreta,  Addison
Wesley,  Lectura,  MA,  1991.
94.  M.  Martelli,  "El  granjero  y  el  ganso:  una  generalización",  Matemáticas
Profesor,  vol.  86  (marzo  de  1993),  págs.  202  y  203.
95.  WS  McCulloch  y  W.  Pitts,  "Un  cálculo  lógico  de  las  ideas  inminentes  en  la  actividad  nerviosa",  
Boletín  de  biofísica  matemática,  vol.  5  (1943),  págs.  115–133.

96.  WA  Miller,  "Números  polinómicos  y  recursividad",  Mathematics  Teacher,  vol.  83  (octubre  de  
1990),  págs.  555–558.
97.  BR  Myers,  “Número  de  árboles  de  expansión  en  una  rueda”,  IEEE  Transactions  on  Circuit  
Theory,  CT­18  (marzo  de  1971),  págs.  280–281.
98.  HL  Nelson,  Problema  702,  J.  Matemáticas  recreativas,  vol.  11:1  (1978–
1979),  pág.  36.
99.  HL  Nelson,  "Dos  falsificaciones",  J.  Matemáticas  recreativas,  vol.  15:1
(1982–1983),  pág.  sesenta  y  cinco.

100.  JC  Nichols,  Solución  al  problema  602,  J.  Matemáticas  recreativas,  vol.  11:1  (1978–1979),  pág.  
75.
101.  G.  Polya,  Mathematical  Discovery,  edición  combinada,  Wiley,  Nueva  York,  1981.
102.  RC  Prim,  "Redes  de  conexión  más  cortas  y  algunas  generalizaciones",  Bell  System  Technical  
Journal,  vol.  36  (1957),  págs.  1389–1401.
103.  I.  Vun  y  P.  Belcher,  "Catalan  Numbers",  Mathematical  Spectrum,  vol.  29:3  (1996–1997),  págs.  3–
5.
104.  A.  Wayne,  Solución  al  problema  E579,  The  American  Mathematical  Monthly,  vol.  51  (marzo  de  
1944),  pág.  165.
105.  A.  Ralston,  "Secuencias  de  De  Bruijn:  un  ejemplo  modelo  de  la  interacción  de  las  matemáticas  
discretas  y  la  informática",  Revista  de  matemáticas,  vol.  55  (mayo  de  1982),  págs.  131–143.

106.  B.  Recaman,  "Los  juegos  del  jamón",  J.  Matemáticas  recreativas,  vol.  10
(1977–1978),  págs.  251–253.
107.  JV  Roberti,  "El  método  indirecto",  profesor  de  matemáticas,  vol.  80  (enero.
1987),  págs.  41–43.
108.  KH  Rosen,  Matemática  discreta  y  sus  aplicaciones,  4ª  ed.,  McGraw­Hill,  Nueva  York,  1999.

109.  KA  Ross  y  CRB  Wright,  Matemáticas  discretas,  3.ª  ed.,  Prentice­Hall,  Englewood  Cliffs,  NJ,  1992.
Machine Translated by Google

904 Referencias

110.  GL  Ritter  et  al.,  "Una  ayuda  para  los  supersticiosos",  Mathematics  Teacher,  vol.  70
(mayo  de  1977),  págs.  456–457.
111.  S.  Sahni,  Concepts  in  Discrete  Mathematics,  2.ª  ed.,  Camelot,  Fridley,  MN,
1985.

112.  BJ  Schwartz,  Solución  al  problema  1014,  J.  Matemáticas  recreativas,  vol.  14:4  (1981–1982),  
pág.  309.
113.  J.  Sedlacek,  "On  the  Skeletons  of  a  Graph  or  Digraph",  Actas  de  la  Conferencia  Internacional  de  
Calgary  sobre  Estructuras  Combinatorias  y  sus  Aplicaciones,  Gordon  &  Breach,  Nueva  York,  
págs.  387–391.
114.  DE  Shasha,  Out  of  their  Minds:  The  Lives  and  Discoveries  of  15  Great  Computer  Scientists,  
Copernicus,  Nueva  York,  1995,  págs.  89–101.
115.  DR  Sherbert,  "Ecuaciones  de  diferencia  con  aplicaciones",  Módulo  322  de  UMAP,  Arlington,  MA,  
1980.
116.  RM  Smullyan,  ¿Cuál  es  el  nombre  de  este  libro?,  Prentice­Hall,  Englewood  Cliffs,  NJ,  1978.

117.  RM  Smullyan,  Alice  in  Puzzle­Land:  A  Carrollian  Tale  for  Children  Under
Ochenta,  Penguin  Books,  Nueva  York,  1982.
118.  RM  Smullyan,  “Leaps  of  Logic”,  Discover  (marzo  de  1993),  pág.  96.
119.  SK  Stein,  “El  matemático  como  explorador”,  Scientific  American,  vol.  204
(mayo  de  1961),  págs.  149–158.
120.  SK  Stein,  Matemáticas:  el  universo  hecho  por  el  hombre,  WH  Freeman,
San  Francisco,  CA,  1969.
121.  A.  Sterrett,  "El  juego  no  paga",  profesor  de  matemáticas,  vol.  60  (marzo  de  1967),  págs.  210–214.

122.  P.  Stevens,  Patrones  en  la  naturaleza,  Little  Brown,  Boston,  1974.
123.  DR  Stone,  "A  Different  Prime  Proof",  Mathematics  Teacher,  vol.  83  (enero.
1990),  pág.  63.
124.  AS  Tanenbaum,  Organización  informática  estructurada,  Prentice­Hall,
Englewood  Cliffs,  Nueva  Jersey,  1976,  págs.  420–423.
125.  The  Official  LSAT  PrepBook,  Law  Services,  Newtown,  PA,  1991.
126.  RM  Thrall,  “Requerimientos  de  insulina  como  un  proceso  lineal  en  el  tiempo,”  Algunos  modelos  
matemáticos  en  biología  (RF  Baum,  ed.),  The  University  of  Michigan  Press,  Ann  Arbor,  MI,  
1967,  pp.  0L2.1–0L2.  4.
127.  PM  Tuchinsky,  "Números  estándar  internacionales  de  libros",  The  UMAP  Journal,  vol.  6:1  (1985),  
págs.  41–53.
128.  A.  Tucker,  Combinatoria  aplicada,  Wiley,  Nueva  York,  NY,  1980.
129.  T.  Tymoczko,  "Computadoras,  pruebas  y  matemáticas:  una  investigación  filosófica  de  la  prueba  
de  cuatro  colores",  Revista  de  matemáticas,  vol.  53  (mayo  de  1980),  págs.  131–138.

130.  JH  van  Lint  y  RM  Wilson,  Un  curso  de  combinatoria,  Cambridge
Prensa  universitaria,  Nueva  York,  1992.
131.  M.  vos  Savant,  Ask  Marilyn,  St.  Martin  Press,  Nueva  York,  pág.  228.
132.  S.  Warshall,  "Un  teorema  sobre  matrices  booleanas",  J.  de  la  Asociación  de  Maquinaria  de  
Computación,  vol.  9  (1962),  págs.  11  y  12.
133.  J.  Williams,  “Graph  Coloring  Used  to  Model  Traffic  Lights”,  Matemáticas
Profesor,  vol.  85  (marzo  de  1992),  págs.  212–214.
Machine Translated by Google

Referencias 905

134.  RJ  Wilson  y  JJ  Watkins,  Gráficos:  un  enfoque  introductorio,  Wiley,  Nueva  York,  1990.

135.  D.  Wood,  "Towers  of  Brahma  and  Hanoi  Revisited",  J.  Recreational  Mathe
matics,  vol.  14:1  (1981–1982),  págs.  17–24.
136.  RV  Young  (ed.),  Matemáticos  notables,  Gale  Research,  Detroit,  MI,  1997.
Machine Translated by Google

Esta  página  se  ha  dejado  en  blanco  intencionalmente
Machine Translated by Google

Soluciones  a  números  impares
Ejercicios

Capítulo  1 El  lenguaje  de  la  lógica

Ejercicios  1.1  (pág.  17)

1.  si 3.  no 5.  F. 7.  T


9.  1+1  =  0 11.   p     q 13.   p     (q     r) 15.  T
17.  F. 19.  T 21.  F.
23 25
pq   p   q   p      q pq  p     q   q  (p     q)      q

TT  FF  F TT  TF  T
TF  FT  T TF  TT  T
FT  TF  T FT  TF  T
FF  TT  T FF  FT  T

27.  F. 29.  T 31
pq  p  XOR  q

TT F
TF T
PIE T
FF F

33.  Si  dos  rectas  son  perpendiculares  a  la  misma  recta,  entonces  son  paralelas.
35.  Si  x  =  1,  entonces  x2  =  1.

907
Machine Translated by Google

908 Soluciones  a  ejercicios  impares

37.  recíproco:  si  París  está  en  Inglaterra,  entonces  Londres  está  en  Francia.
inversa:  si  Londres  no  está  en  Francia,  entonces  París  no  está  en  Inglaterra.  
contrapositiva:  si  París  no  está  en  Inglaterra,  entonces  Londres  no  está  en  Francia.

39.   q     p  →  r 41.   p      q  ↔   r 43.  no


45.  si 47.  T 49.  F.
51.  T 53.  T
55.
pqp     q     pags     q  →     pags

TT  TF  F
TF  FF  T
FT  FT  T
FF  FT  T

57.
pq  p     qp     qp     q  ↔  p     q

tt  tt  tt
TF  TF  F
FT  TF  F
FF  FF  T

59.  si 61.  si 63.  si 65.  no

67.  (pag     q)  →  ((   pag)     (   q)) 69.  (pag  →  q)  ↔  ((   pag)     q)


71.  (A     C)     (B     C)     (A     B)
73. A 75. A B
B' A' B

A B' C

77.  Ellen  debe  usar  la  computadora  1.

Ejercicios  1.2  (pág.  29)

1.  F. 3. 5.
pag     pag     (   pag) págs     págs

TF T TT
PIE F F F

↑__  __↑  idéntico ↑_______↑
Machine Translated by Google

Capítulo  1 El  lenguaje  de  la  lógica 909

7.
pqp     qq     p

TTT T
TFT T
FTT T
FFF F

9.
pq  p  →  q     (p  →  q)   qp      q

TT T FFF
TF F T  T  T
PIE T FFF
FF T FTF

↑____idéntico____↑

11
pq  rq     rp     (q     r)  p     q  (p     q)     r

tt  tt  tt T T
TT  FF  F T F
TF  TF  F F F
TF  FF  F F F
FT  TT  F F F
FT  FF  F F F
FF  TF  F F F
FF  FF  F F F

↑_____idéntico______↑

13
pqr  pag     q  (pag     q)  →  rp  →  rq  →  r  (pag  →  r)     (q  →  r)

TTT  T T  TT  T
TTF  T FFF  F
TFT­T T  TT  T
TFF  T F  F  F  F
FTT  T T  TT  T
FTF  T F  TF  F
FFT  F T  TT  T
FFF  F T  TT  T

↑_________idéntico_____________↑

15.  T 17.  F. 19.  si 21.  no 23.  si 25.  si


Machine Translated by Google

910 Soluciones  a  ejercicios  impares

27 29.  T 31.  T 33.  T 35.  F.


pqp  NI  q
37.  T 39.  T 41.  F.
TT F
TF F 43.     ( p     q)  ≡     (   p)      q
PIE F
≡  pag      q
FF T

45.     (p      q)  ≡   p        ( q)   47.  pag  →   q  ≡     pag      q  
≡   p     q ≡     (pag     q)

49.  p     q 51.  p     q 53.  p      q


55.  (A     B )     [(A     B)     C]  ≡  A     C

57. 59.
p     p  p  |  p pq  p     q  p|q  (p|q)|(p|q)

TFF TT  TF  T
FTT TF  FT  F
FT  FT  F
↑______↑ FF  FT  F

↑___idéntico___↑

61.
pq  p  →  q  p|p  (p|p)|(p|p)  q|q  ((p|p)|(p|p))|(q|q)
TT  TF  T F T
TF  FF  T T F
FT  TT  F F T
FF  TT  F T T

↑________________idéntico________________↑

63.  p  XOR  q  ≡  (((p|p)|(q|q))|(((p|q)|(p|q))|((p|q)|(p|q))) )|  (((p|
p)|(q|q))|(((p|q)|(p|q))|((p|q)|(p|q))))
65.  1 67.  1 69.  0.3 71.  0.7 73.  0 75.  0.5

77.
1  −  x  si  0  ≤  x  ≤  1/2
X de  lo  contrario

Ejercicios  1.3  (pág.  36)

1.  F. 3.  T 5.  T 7.  F. 9.  F. 11.  T


Machine Translated by Google

Capítulo  1 El  lenguaje  de  la  lógica 911

13.  (   x)(P(x)     Q(x))  15.  (   x)(P(x)      Q(x))  17.  (   x)  (x2  ≤  0)


19.  Algunas  supercomputadoras  no  se  fabrican  en  Japón.
21.  ( x)( y)(xy  >  0) 23.  ( x)( y)(xy  =  x)
25.  F. 27.  T

29.  El  cuadrado  de  todo  número  entero  no  es  negativo.

31.  Hay  números  enteros  x  e  y  tales  que  x  +  y  =  7.
33.  Existe  un  entero  x  tal  que  y  −  x  =  y  para  todo  entero  y.
35.  T  37.  T  39.  T  41.  T  43.  F  45.  T  47.  T
49.  T  51.  T  53.  T  55.  F  57.  F  59.  T  61.  T
63.  T  65.  F  67.  T

Ejercicios  1.4  (pág.  45)

1.  p  →  q   3. 5.  inválido
pqp     qp  →  (p     q)
  q 7.  válido
TTT T
     pag TFT T 9.  válido
FTT T
11.  válido 13.  q  es  falso.
FFF T
15.  r  es  verdadera.

17.  El  programa  se  está  ejecutando. 19.  Carol  es  un  bebé.
21.  Benjamin  a  Cindy  y  Aaron  a   23.  azul
Daphne;  tienen  34,  27,  28  y  29  
años,  respectivamente.
25.  B
27.  A  es  caballero  y  B  escudero.   29.  si
31.  caballero  35.  Kitty  es  culpable. 33.  “Soy  rojo”.

37.  Al  menos  una  vaquera  escapará  de  las  
heridas.

Ejercicios  1.5  (pág.  54)

1.  si 3.  si
Machine Translated by Google

912 Soluciones  a  ejercicios  impares

5.  Sean  xey  dos  enteros  pares  cualesquiera.   7.  Sea  x  =  2  m  cualquier  número  entero  par.  
2  2  =  2(2  m  2(2  
)  ems  
 )  
Etntonces  
ambién  e
x  
s  
=  
Entonces  x  =  2m  y  y  =  2n  para  algunos   un  entero  par.
enteros  m  y  n.
Entonces  x  +  y  =  2  m  +  2  n  
9.  Sea  x  cualquier  entero  impar.  Entonces  x  
=  2( m  +  n )  
=  2  m  +  1  para  algún  entero  m  =  (2  m  +  .
que  también  es  un  número  entero  par. 2   2
x  _ 1)
11.  Sea  x  cualquier  entero  par  e  y  cualquier   =  2(2  metros 2 +  2  m )  +  1  

entero  impar.  Entonces  x  =  2  m  y  y  =  2  n   que  es  un  entero  impar.
+  1  para  algunos  enteros  m  y  n.  Entonces  
13.  Sea  x  =  4  k  +  1.  Entonces  =  
xy  =  (2  m)(2  n  +  1)  =  2(2mn  +  m),  un   2 2  =  16k +  8  k  +  1  
2x  _ (4  k  +  1)
entero  par.
=  4(4  k  +2
  2  m  
k )  
=+  4  1  k  =
  +  4
  2  m +s  
  k  e   1u,  n  
donde  
número  entero  par.   2 número    x2  también   entero   es  
du n  
e  
15.  Sea  x  cualquier  número  entero.   la  misma  forma.
Supongamos  que  ni  siquiera  es;  es  impar  
y  es  de  la  forma  2  m  +  1.  Entonces  =  (2  
2x  _ m  +  1) 2 2  =  4  metros  +  4  metros  +  1  
17.  Sean  x  e  y  dos  enteros  cualesquiera.
=  2(2  m   2 +  2  metros )  +  1
Suponga  que  la  conclusión  dada  es  
que  es  un  entero  impar.  Entonces  la  
falsa;  es  decir,  suponga  que  tanto  x  
hipótesis  dada  es  falsa  y  el  resultado  
como  y  son  enteros  impares.  Entonces  
sigue.
x  =  2m  +  1  y  y  =  2n  +  1  para  algunos  
19.  Supongamos  que  √2  no  es  un  número   enteros  m  y  n .     xy  =  (2  m  +  1)(2  n  +  1)  
irracional;  es  decir,  √2  es  un  número   =  2(2mn  +  mn
Esto     +iega  
  n )  +la  
1,  
hu n  entero  
ipótesis   impar.  
dada  y  
racional.  Sea  √  2  =  a /  b,  donde  a  y  b  no   así  sigue  el  resultado.
tienen  factores  comunes  positivos  
2 =
excepto  1.  Entonces  ( a /  b )
2  
2  2  o  un .   =  
  22    seegundo
s  un  factor  de  a2  y  por  
lo  tanto  de  a.  Entonces  a  =  2m  para  
21.  Suponga  que  √p  es  un  número  racional  a/
algún  entero  m.     (2m)2  =  2b2  o  b2  =  
b,  donde  a  y  b  no  tienen  factores  
2m2.     2  es  un  factor  de  b2  y  por  lo  
comunes  positivos  excepto
tanto  de  b.  En  consecuencia,  2  es  un   2
1.  Entonces  √p  =  a /  b,  ( a /   =  p  o
factor  tanto  de  a  como  de  b,  lo  que   2  
un b )  =  pb  2.  En  consecuencia,  p  es  un  
contradice  la  suposición.
factor  de  a  y  p2or   lo  tanto  
sea   dp.  
a  =  m e  aE.  ntonces  
Así  que  
2
(mp )  o  b =  pb   2
2
25.  prueba:  Todo  entero  n  es  de  la  forma  3  k,   =  pm  2.  Como  antes,  esto  
3  k  +  1,  o  3  k  +  2.  caso  1  Sea  n  =  3  k.   muestra  que  p  es  un  factor  de  b.  Así  p  
Entonces  −  n  =  (3  k )  −  (3  k )  =  27  k  3  −   es  un  factor  común  de  a  y  b  una   ,
3  k  =  3(9  k  3  −3  kk   contradicción.
3  
norte  )  
+c  aso  
1.  Entonces  
2  Sea  n  = 3  −3  
n  =  (3  k  +   1)  
3   −+    
2   (23  
kk  +  −  1(3  
7   )  =k  (+27  
=   k7  
  12)    
23.  prueba:  
k  3  2  +  27  k  +  6  k  =  3(9  k  3  2  +  9  k  +  2  k )
3   caso  1  n  es  un  entero  par,  digamos,  2  
norte

m.  Entonces  n  2++  
  2n  
m  ==  (  2  
orte   (2  m2  es  
2m  2etros ) caso  
2  n  
un  
+  9  k  +  1) entero  impar,  digamos,  2  m  +  1.
+  m),  un  entero  par.
Machine Translated by Google

Capítulo  1 El  lenguaje  de  la  lógica 913

caso  3  Sea  n  =  3k  +  2.  
Entonces  n3  −  n  =  (3k  +  2)3   Entonces  n2  +  n  =  (2m  +  1)2  +  
−  (3k  +  2)  =   (2m  +  1)  =  (4m2  +  4m  +  1)  +  
(27k3+54k2+36k+8)   (2m  +  1)  =  2(2m2  +  3m  +  1),  
−(3k  +  2)  =  27k3  +  54k2   de  nuevo,  un  entero  par.
+  33k  +  6  =  3(9k3+18k2+11k+2)
27.  prueba:  
Así,  en  todos  los  casos,  n3  −  n  es  
caso  1  Sea  x,  y  ≥  0.  
divisible  por  3.
Entonces  |x  ∙  y|  =  xy  =  |x|∙|
29.  prueba:  Elija  x  =  1.  Claramente,   y|  caso  2  Sea  x  <  0,  y  <  0.  
x2  =  x. Entonces  |x|=−x  y  |y|=−y.  |x∙  y|  
=  xy  =  (−x)∙(−y)  =  |x|∙|y|  caso  3  
31.  prueba:  Sea  a  cualquier  entero  distinto  
Sea  x  ≥  0,  y  <  0.  Entonces  |x∙  y|
de  cero.
3 =−(xy)  =  x∙(−y)  =  |x|∙|y|  caso  4  
Entonces  1729a3  =  a3  +  
3 3 Sea  x  <  0,  y  ≥  0.
(12a)  =  (9a)  
(10a)
+  
Este  caso  es  similar  al  caso  3.
Dado  que  a  es  arbitrario,  1729a3  
tiene  infinitas  opciones. 41.  demostración:  Como  a<b,  existe  
un  número  real  positivo  x  tal  
33.  0 35.  2
que  a  +  x  =  b.     (a  +  x)  +  c  =  b  
37.  x  =  2,  P(x):  x2  =  4. +  c  Es  decir,  (a  +  c)  +  x  =  b  +  c  
  a  +  c<b  +  c
39.  Cancelar  a  −  b  no  es  válido  ya  que  
a  =  b.
45.  11,  desde  2047  =  23  ∙  89.
43.  prueba:  Sea  a  ∙  b  =  0  y  a  =  0.
Como  a  ∙  0  =  0,  a  ∙  b  =  a  ∙  0.
Cancelando  a  de  ambos  lados,  
obtenemos  b  =  0.

Ejercicios  de  repaso  (pág.  58)

1.
pq  p     q   q  (p     q)     ( q)

TT  TF  F
TF  TT  T
FT  TF  F
FF  FT  F

3. 5.  T 7.  F.
pqr  q  →  rp  →  (q  →  r)

TTT  T T 9.  (w<x)     (y<z)  →  (w  


+  y<x  +  z)
TTF  F F
TFT­  T T
Machine Translated by Google

914 Soluciones  a  ejercicios  impares

pqr  q  →  rp  →  (q  →  r)
11.  si  13.  no
TFF  T T
15.  [A     (A     B)]     (A     B )
FTT  T T
FTF  F T 17.  T 19.  T 21.  T
T  FFT T 23.  F. 25.  T
FFF  T T 27.  F. 29.  F.

31.  inversa :  si  x  ≥  y,  
:  eSntonces  
i  x  +  z<y  
x  
++    
zz,  
  ≥
e  
ntonces  
y  +  z.   x<y.  
≥  y  +  z,  entonces  
contrapositiva  
x  ≥  y. inversa:  si  x  +  z  

33.  Si  x  ≤  3  y  x  ≥  −3,  entonces  |x|  ≤  3.  
35.  sí  37.  no  39.  sí  41.  sí  43.  V  45.  V  47.  F

49.  (A     B)     (A     B)     (A     B)  ≡  A     B


A'

B'
51.  inválido 53.  F. 55.  T 57.  F. 59.  F. 61.  F.

63.  prueba:  Sean  x  y  x  +  1  dos   65.  prueba:  n4−n2  =  [(n−1)n(n+1)]n  
enteros  consecutivos.   contiene  un  producto  de  tres  
caso  1  Sea  x  par.  Entonces   enteros  consecutivos.  Por  tanto,  
x  =  2m  para  algún  entero  m. por  casos,  es  divisible  por  3.
Entonces  x(x  +  1)  =  2m(2m  +  
67.  Prueba  indirecta:  Suponga  que  la  
1)  =  2[m(m  +  1)],
conclusión  dada  es  falsa,  es  decir,  
un  número  entero  par.  a  ≤  6  yb  ≤  6.  Entonces  a  +  b  ≤  caso  2  Sea  x  
impar.   Entonces  
+  b  ≤  12,  donde  x  =  2m  +  1  para  algún  entero   6  +  6,  lea  
m.  contradice   s  
hdipótesis.  
ecir,  a  
  x(x  +  1)  =  (2m  +  1)(2m  +  2)  69.  
Demostración:  Suponga  que  a2  =  b2  y  =  2[(m  +  1)(2m  +  1)],  a  =  b.  Como  
a2  −  b2  =  (a  −  b)  (a+b)  =  0  y  a−b  
=  0,  a+b  =  0.     a  =  −b.

de  nuevo  un  entero  par.

71.  0.5 73.  0.5 75.  0 77.  0.3

79.  Como  t(p)  =  x,  t(p )  =  1  −  x.  Entonces  t(p     p )  =  max{x,  1  −  x}.
Suponga  que  t(p     p )  =  1.  Entonces  max{x,  1  −  x}  =  1.  Suponga  que  0  ≤  x  ≤  1/2.
Entonces  max{x,  1  −  x}  =  1  −  x  =  1,  entonces  x  =  0;  es  decir,  t(p)  =  0.  Por  
otro  lado,  sea  1/2  <  x  ≤  1.  Entonces  max{x,  1  −  x}  =  x  =  1;  entonces  t(p)  =  1.
En  cualquier  caso,  t(p)  =  0  o  1.
Por  el  contrario,  sea  t(p)  =  0  o  1.  Entonces  t(p     p )  =  max{t(p),  t(p )}  
=  max{t(p),  1  −  t(p)}.  Si  t(p)  =  0,  entonces  t(p     p )  =  max{0,  1}  =  1;  por  
otro  lado,  si  t(p)  =  1,  entonces  también  t(p     p )  =max{1,  0}  =  1.  En  
ambos  casos,  t(p     p )  =  1.
Entonces  t(p     p )  =  1  si  y  solo  si  t(p)  =  0  o  t(p)  =  1.
Machine Translated by Google

Capítulo  1 El  lenguaje  de  la  lógica 915

81.  prueba:  t((p     q) )  =  1  −  t(p     q)  =  1  −  


max{t(p),  t(q)}  =  1  −  
max{x,  y}  y  si  x  ≤  yx  de  
lo  contrario
=  1  ­

= 1  −  y  si  x  ≤  y  1  −  
x  de  lo  contrario  t(p  
  q )  =  min{t(p),  t(q )}  =  min{1  −  x,  1  
−  y}  1  −  y  si  x  ≤  y  1  −  x  

= de  lo  contrario

Así  t((p     q) )  =  t(p )     t(q ).

Ejercicios  complementarios  (pág.  62)

1.  inversa :  si  −a<x<a,  entonces  |x|  <  un.  inversa :  Si  |
x|  ≥  a,  entonces  x≤  
  ≥−  a   oo  x  (  x  
a)   ≤  ≥−  a.  
ceontrapositivo:  
a),   ntonces  |x|  ≥  sui  
n.(x  

3.     q 5.     p      q 7.  si

9.     [( x)( y)(xy  =  yx)  ≡  ( x)( y)(xy  =  yx)

11.     [( x)( y)( z)(x  +  y  =  z)  ≡  ( x)( y)( z)(x  +  y  =  z)  13.  

Prueba:  Sea  K  =  (n2  +  n)(n2  +  n  +  1)(n2  +  n  +  2)
y   L  =  (n2  ­  n)(n2  ­  n  +  1)(n2  ­  n  +  2).
2n(3n4  +  7n2  +  2)  =  6n5  +  14n3  +  4n  =  (4n3  
+  6n)n2  +  (2n4  +  8n2  +  4)n  =  (n2  +  n)(n4  
+  2n3  +  4n2  +  3n  +  2)
−  (n2  −  n)(n4  −  2n3  +  4n2  −  3n  +  2)
=  K  ­  L
Como  K  es  el  producto  de  tres  enteros  consecutivos,  es  divisible  por  3.
Pero  n2  +  n  =  n(n  +  1)  es  divisible  por  2.  Entonces  n2  +  n  +  2  es  divisible  por  4.  Por  
lo  tanto,  K  es  divisible  por  24.  Asimismo,  L  también  es  divisible  por  24.
Por  lo  tanto,  K  −  L  es  divisible  por  24.  Por  lo  tanto ,  n(3n4  +  7n2  +  2)  es  divisible  por  
12.
15.  40

17.  t(p     p )  =  max{t(p),  t(p )}
=  max{t(p),  1  −  t(p)}  =  1  solo  
si  t(p)  =  0  o  t(p)  =  1.

19.  No  es  una  tautología  de  tres  valores.

21.  Es  una  tautología  de  tres  valores.
Machine Translated by Google

916 Soluciones  a  ejercicios  impares

23
pqp     q  (p     q)  pqp     q

0  0  0   0 1  11  1  1  tu  0  10  0
tu  0  1   tu tu tu
tu  0
1  tu  uu  1 tu
uu tu tu  tu  tu
tu  1  1   0  tu  0  0  0  01  0  0  0  00  
0  1  tu   0
1  1 1  1  1  00  tu  1

↑_____idéntico_____↑

Capitulo  2 El  lenguaje  de  los  conjuntos

Ejercicios  2.1  (pág.  76)

1.  {abril,  agosto} 3.  {enero,  marzo,  mayo,  julio,  agosto,  octubre,  diciembre}

5.  {x     Z|0  <x<  5} 7.  {x|x  es  miembro  de  las  Naciones  Unidas}

9.  si 11.  si 13.  F.

15.  T  17.  F  19.  F  21.  T  23.  T  25.  T  27.  T

29.  F. 31.  {Ø,  A} 33.  2n

35.  {3,  6,  9} 37.  {2,  3,  4,  6,  8,  9}

39.  {Ø,{1},{2},{3},{1,3}} bab,  b3 , ba2b


41.  b,  b2 ,   43.  λ,  b,  a2,  a2b,  aba
45.  3 47.  5 49.  00,  01,  10,  11 51.  0,  1,  2

53.  Considere  la  implicación  x      55.  Sea  x  un  elemento  arbitrario  en  A.  
Ø  →  x     A.  Dado  que  la   Como  A     B,  x     B.  Como  B     
hipótesis  es  falsa,  esta  es  una   C,  x     C.  Así,  todo  elemento  en  
implicación  verdadera.     Ø     A. A  también  está  en  C.     A     C.

Ejercicios  2.2  (pág.  93)

1.  {a,  b,  c,  g,  j,  k} 3.  {d,  h} 5.  {a,  b,  c,  f,  g,  i,  j,  k}

7.  {a,  c,  d,  e,  f,  h,  i,  j,  k} 9.  {un} 11.  {a,  b,  g}

13.  {w,  y,  z} 15.  {a,  b} 17.  {x,  y}


Machine Translated by Google

Capítulo  2 El  lenguaje  de  los  conjuntos 917

19.  {(b,  x),  (c,  x)} 21.  Ø 23.  {(b,  x),  (b,  z),  (c,  x),  (c,  z)}

25.  {(b,  x,  x),  (b,  x,  z),  (c,  x,  x),  (c,  x,  z)} 27.  T 29.  F. 31.  F.

33.  T 35.  F. 37.  F.

39.  A  =  {a},  B  =  Ø,  C  =  {a} 41.  A  =  {a},  B  =  {b},  C  =  {a,b}
43.  no 45.  no

47.  Sea  x     (A ) .  Entonces  x     A   , 49.  A  ∩  (A     B)  =  (A  ∩  A)     (A  ∩  B)


entonces  x     A.  Entonces  (A )     A. =  UN     (UN  ∩  B)
Por  el  contrario,  sea  x     A. =  A,  ya  que  A∩B     A.
Entonces  x     A,   entonces  x     
(A ) .     UN     (UN) .  Así  (A)  =  A.

51.  A     A  =  (A  −  A)     (A  −  A)  =  Ø  


  Ø  =  Ø

53.  UN     B  =  (A  ­  B)     (B  ­  A) 55.  (A     B     C)  =  [(A     B)     C]


=  (B  −  A)     (A  −  B) =  (A     B)  ∩  C  =  
=  segundo     un (A  ∩  B )  ∩  C  =  A  
∩  B  ∩  C

57.  UN  ∩  (A  ­  B)  =  UN  ­  B

59.  (A  −  B )  −  (B  −  A )  =  Ø 61.  UN     segundo  −  ( UN  ∩  segundo )  =  UN  ∩  segundo

63.  (A  ∩  B)     (A     B )  =  U 65.  (A     B )     (A  ∩  B)  =  B

67.  A      Bi)  =   (A     Bi)


( i I i I
un  ∩  ( Bi)  = (A  ∩  Bi)
i I i I

69.  Ø 71.  {Angelo  0.4,  Bart  0.7,  Cathy  0.6,  Dan  
0.7,  Elsie  0.2,  Frank  0.6}

73.  Ø 75.  {Ángel  0.4,  Bart  0.7,  Cathy  
0.6}

77.  {(Angelo,  Dan)  0,3,  (Angelo,  Elsie)  0,4,  (Angelo,  Frank)  0,4,  (Bart,  Dan)  0,3,  (Bart,  
Elsie)  0,7,  (Bart,  Frank)  0,4,  (Cathy,  Dan)  0.3,  (Cathy,  Elsie)  0.6,  (Cathy,  Frank)  0.4}

79.  {(Angelo,  Angelo)  0,4,  (Angelo,  Bart)  0,4,  (Angelo,  Cathy)  0,4,  (Bart,  Angelo)  0,4,  (Bart,  
Bart)  0,7,  (Bart,  Cathy)  0,6,  (Cathy,  Angelo)
0,4,  (Cathy,  Bart)  0,6,  (Cathy,  Cathy)  0,6}

81.  Demostración:  Por  la  ley  de  De  Morgan  en  conjuntos  ordinarios,  basta  mostrar  que  
d(A  ∩  B)(x)  =  dA     B(x)  para  todo  elemento  x.
Machine Translated by Google

918 Soluciones  a  ejercicios  impares

Sea  x     (A  ∩  B) .  Entonces  
d(A  ∩  B)(x)  =  1  −  d(A  ∩  B)(x)
=  1  −  min  {dA(x),  dB(x)}  1  −  

= dA(x)  si  dA(x)  ≤  dB(x)  1  −  dB(x)  
en  caso  contrario  Por  otro  lado,  
sea  x     A     B .  Después
dA     B(x)  =  max{dA(x),  dB(x)}  1  −  

= dA(x)  si  dA(x)  ≤  dB(x)  1  −  dB(x)  
en  caso  contrario  Así  d(A  ∩  B)( x)  
=  dA     B(x)  para  todo  elemento  x,  entonces  (A∩B)  =  A   B  para  conjuntos  
borrosos.

Ejercicios  2.3  (pág.  98)

1.  01010101 3.  11110001 5.  {s0,  s3} 7.  {s0,  s2,  s3}


9.  {Ø,{s0},{s1},{s0,s1}  11.  10010010 13.  01001001

15.  01001000 17.  11011010 19.  10010011

21.  00000000 23.  00000001 25.  01001001

Ejercicios  2.4  (pág.  102)

1.  26 3.  7 5.  3a 7.  a  +  2b  9.  a  +  2b  11.  a  −  b

13.  0 15.  segundo 17.  un 19.  segundo 21.  segundo  ­  un  23.  50

25.  366 27.  134 29.  1007  31.  1244  33.  55 35.  10

37.  13 39.  230  41.  60

43.  {Ø,{a},{b},{c},{a,b},{b,  c},{c,  a},{a,  b,  c}}

45.  4b 47.  b2 49.  |U|−|A     B     C|

51.  |A1     A2     A3     A4|=|A1     (A2     A3     A3)|  =  |A1|+|A2  


  A3     A4|−|A1  ∩  (A2     A3     A3)|  =  |A1|  +  [|A2|+|A3|+|
A4|−|A2  ∩  A3|
−|A2  ∩  A4|−|A3  ∩  A4|−|A2  ∩  A3  ∩  A4|]
−|A1  ∩(A2   A3   A3)| (1)
|A1  ∩  (A2     A3     A3)|=|(A1  ∩  A2)     (A1  ∩  A3)     (A1  ∩  A4)|
=  |A1  ∩  A2|+|A1  ∩  A3|+|A1  ∩  A4|
−|(A1  ∩  A2)  ∩  (A1  ∩  A3)|
−|(A1  ∩  A2)∩(A1  ∩  A4)|−|(A1  ∩  A3) (A1  ∩  A4)|
+|(A1  ∩  A2)  ∩  (A1  ∩  A3)  ∩  (A1  ∩  A4)|
Machine Translated by Google

Capítulo  2 El  lenguaje  de  los  conjuntos 919

=  |A1  ∩  A2|+|A1  ∩  A3|+|A1  ∩  A4|−|A1  ∩  A2  ∩  A3|
−|A1  ∩  A2  ∩  A4|−|A1  ∩  A3  ∩  A4|
+|A1  ∩  A2  ∩  A3  ∩  A4| (2)
Sustituyendo  (2)  en  (1),  obtenemos  el  resultado  deseado:
4 4 4 4
| Ay|  = |Ai|  −   |Ai  ∩  Aj|  + |Ai  ∩  Aj  ∩  Ak|−| Ay|
yo=1 yo=1 1≤i<j≤4 1≤i<j<k≤4 yo=1

Ejercicios  2.5  (pág.  107)

1.  1,  2,  4,  8 3.  e,  ee,  ee , eeee 5.  λ,  b2,  b4,  b6


7.Z  +

9.  conjunto  de  enteros  pares 11.  1)  0     L  


2)  x     L  →  1x,  xx     L

13.  1)  a     L   15.  1)  segundo  


2)  x     L  →  bxb     L   L  2)  x     L  →  bx     L

17.  1)  a     L   19.  si


2)  x     L  →  axa     L
21.  no 23.  5 25.  42

27.  ( )( )( ),(( )( )),(( ))( ),( )(( )),((( )))

29.  si 31.  no

33.  no,  ya  que  (( ))(( ))  no  se  puede  generar.

35.  1)  λ     
2)  x      ,  y     →  xy,  yx  

Ejercicios  de  repaso  (pág.  111)

1.  {b,  y,  z} 3.  {b,  f,  x,  y} 5.  {b,  c,  y}   7.  {b,  f,  x,  y}

9.  {a,  c,  d,  f},  {b,  d,  e,  f,  g} 11.  {b,  e,  k}

13.  {Ø,{Ø},{{Ø}},{Ø,{Ø}}}

15.  no 17.  01000100 19.  01100110 21.  54%

23.  21 25.  7 27.  2871

29.  3,  lg  3,  lg  lg  3,  lg  lg  lg  3

31.  1,  √  3,  1  +  2  √3,  1  +  2  1  +  2  √3
Machine Translated by Google

920 Soluciones  a  ejercicios  impares

33.  1)  1     S  2)   35.  1)  λ     S  2)  


x     S  →  1  +  2x     S x     S  →  bxa     S

37.  a,  ab,  ba,  abb,  bab 39.  1)  a,  b     L  2)  


x     L  →  ax,  bx     L

41.  1)  1     L  2)   43.  si


x     L  →  0x0     L

45.  Sea  x     A     (B  ∩  C).  Entonces  x     A  o  x     B  ∩  C.     x     A  o  (x     B  y  x     C).     x     
A     B  y  x     A     C.  Así,  x     (A     B)  ∩  (A     C).  Entonces  A (B∩C)     (A B)∩(A C).  
Del  mismo  modo,  (A B)∩(A C)     A (B∩C)
De  ahí  el  resultado.

47.  A∩(B−C)  =  A∩(B∩C )  =  (A∩B)∩C  =  (A∩B)−C  =  (A∩B)−(A∩C)

49.  Ø 51.  Ø

53.  {(Mike,  Jean)  0,6,  (Mike,  junio)  0,5,  (Andy,  Jean)  0,3,  (Andy,  junio)  0,3,  (Jeff,  Jean)  0,7,  
(Jeff,  junio)  0,5}

55.  Como  conjuntos  ordinarios,  (A  −  B)  =  (A  ∩  B )  =  A     B.  Así  que  queda  por  mostrar
que  d(A−B)(x)  =  dA     B(x)  para  todo  elemento  x.  d(A−B)
(x)  =  1  −  dA−B(x)  =  1  −  dA  ∩  B(x)  =  1  −  mín{dA(x),  
dB(x)}  =  1  −  mín{dA(x) ),  1  −  dB(x)}  si  
dA(x)  ≤  1  −  dB(x)  en  caso  contrario

= 1  −  dA(x)  
dB(x)
Similarmente,

1  −  dA(x)   si  dA(x)  ≤  1  −  dB(x)  en  
dA     B(x)  =
dB(x) caso  contrario

Así  d(A−B)(x)  =  dA     B(x)  para  todo  elemento  x;  entonces  (A  −  B)  =  A     B  como  


conjuntos  borrosos.

Ejercicios  complementarios  (pág.  113)

1.  (A  ­  B)  ∩  (A  ­  C)
=  (A  ∩  B )  ∩  (A  ∩  C )
=  UN  ∩  B  ∩  C  
=  UN  ∩  (B  ∩  C )
=  UN  ∩  (B     C)
=  UN  −  (B     C)
Machine Translated by Google

Capítulo  3 Funciones  y  Matrices 921

3.  (A  ∩  B)     (A  ∩  C)
=  [(A  ∩  B)  −  (A  ∩  C)] [(A  ∩  C)  −  (A  ∩  B)]
=  [(A  ∩  B)  ∩  (A  ∩  C) ] [(A  ∩  C)  ∩  (A  ∩  B) ]
=  [(A  ∩  B)  ∩  (A     C) ] [(A  ∩  C)  ∩  (A     B )]
=  [(A  ∩  B  ∩  A )     (A  ∩  B  ∩  C )] [(A  ∩  C  ∩  A )     (A  ∩  C  ∩  B )]  =  Ø  
  (A  ∩  B  ∩  C )     Ø     ( A  ∩  C  ∩  B )
=  (A  ∩  B  ∩  C )     (A  ∩  C  ∩  B )
=  UN  ∩  [(B  ∩  C )     (C  ∩  B )]
=  UN  ∩  [(B  ­  C)     (C  ­  B)]
=  UN  ∩  (B     C)

5.  A  ∩  B  ∩  C 7.  A  ∩  B 9.  266

11.  1)  λ     L   13.  Sean  x  =  x1 ...xn  y  y  =  y1 ...yn.  xy  =  


2)  Si  x     L,   x1 ...xny1 ...yn  (xy)
entonces  0x1     L. R  =  yn ...y1xn ...x1
=  yRxR

15.  Por  el  ejercicio  14,  basta  demostrar  que  (xxR)  
R  =  xxR.  
13  
(xxR)  
=  xxR,  
R  =y  (a  
xR)  
que  
R(xR,  
xR)  
pRor  el  Ejercicio  

=  x.

Capítulo  3 Funciones  y  Matrices

Ejercicios  3.1  (pág.  123)

1.  −40°F 3.  9.8 5.  5 7.  $75

9.  $84.60 11.  6 13.  7 15.  combinatoria

17.  & 19.  Z 21.  43 23.  122

25.  −23 27.  41 29.  azalea 31.  olor

33.  5 35.  indefinido 37.  6 39.  1  +  p  +  q


41.  5 43.  x2  +  2x  +  2
45.  demostración:  Sea  y     f(A     B).  Entonces  existe  un  elemento  x     A     B  tal  que  
y  =  f(x).  Si  x     A,  entonces  y  =  f(x)     f(A);  si  x     B,  entonces  y  =  f(x)     f(B).
En  cualquier  caso,  y     f(A)     f(B).     f(A     B)     f(A)     f(B).
Por  el  contrario,  sea  y     f(A)     f(B).  Entonces  y     f(A)  o  y     f(B).
Si  y     f(A),  entonces  existe  un  elemento  a     A  tal  que  y  =  f(a).  Como  a     A     
B,  y  =  f(a)     f(A     B).  Si  y     f(B),  entonces,  de  manera  similar,  y     f(A     B).
Así,  en  ambos  casos,  y     f(A     B).     f(A)     f(B)     f(A     B).
Machine Translated by Google

922 Soluciones  a  ejercicios  impares

47.  demostración:  Sea  y     f(A)  −  f(B).  Entonces  y     f(A)  y  y     f(B).  Entonces  existe  


un  elemento  a     A  tal  que  y  =  f(A);  ya  que  y  =  f(B),  a     B.     a     A−B.
En  consecuencia,  y  =  f(a)     f(A  −  B).     f(A)  −  f(B)     f(A  −  B).

Ejercicios  3.2  (pág.  134)

1.  norte 3.  n  +  1 5.  6 7.  9 9.  −4

11.  7 13.  {−1,  0}   15.  1538 17.  1435 19.  97


21.  358 23.  h(a)  =  h(c)  =  h(d)  =  1,  h(b)  =  h(e)  =  0

25.  h(b)  =  h(c)  =  h(g)  =  1,  h(a)  =  h(d)  =  h(e)  =  h(f)  =  0

27.  {c,e,g,h} 29.  {b,  d,  f,  h} 31.  jueves 33.  lunes


35.  T 37.  T 39.  miércoles  49.   41.  sábado
43.  Miércoles  45.  1  53.  Sea   47.  2 7  de  abril  51.  26  de  abril
n  =  2k  +  1.  Entonces  n/2=k  +   55.  Sea  n  =  2k  +  1.  Entonces  
1/2  =  k n2/4=k2  +  k  +  1/4  =  k2  +  k
=  (4k2  +  4k)/4  =  (n2  −  1)/4

57.  caso  1  Sea  n  =  2k  +  1. 59.  caso  1  Sea  x     Z.  Entonces


Entonces  n/2  =  k  y  n/2   x  =  x  =  −(−x)  =  −−x.
=  k  +  1.     n/2+n/2  =  
caso  2  Sea  x     Z.  
k  +  (k  +  1)  =  2k  +  1  =  n
Entonces  x  ,=donde
  k  +  x
k     Z  y  0  <  x  <  1.  −−x=−
caso  2  Sea  n  =  2k.  Entonces   (−k  −  1)  =  k  +  1  =  x
n/2  =  k  =  n/2.  Entonces  n/
2+n/2  =  k+k  =  2k  =  n.

61.  caso  1  Sea  x     A  ∩  B.  Entonces  x   63.  caso  1  Sea  x     A  ∩  B.  Entonces  


  A,  x     B,  y  x     A     B. x     A,  x     B,  y  x     A     B.     
Entonces,  fA  ∩  B(x)  =  fA(x)  =   RHS  =  1  +  1  −  2  =  0  =  LHS.
fB(x)  =  fA B(x)   caso  2  Sea  x     A∩B.  Entonces  x  
=  1     RHS  =  1+1−1  =  1  =  LHS.
  A  o  x     B.  Si  x     A  y  x     B,  
caso  2  Sea  x     A∩B.  Entonces  x   entonces  RHS  =  0  +  1  −  0  =  1  =  
  A  o  x     B.  Si  x     A  y  x     B,   LHS.  Si  x     A  y  x     B,  entonces  
entonces  RHS  =  0  +  1  −  0  =  1  =  1   RHS  =  0  +  0  −  0  =  0  =  LHS.  Si  x     
=  LHS.  Si  x     A  y  x     B,  entonces   A  y  x     B,  entonces  RHS  =  1  +  0  −  
RHS  =  0  +  0  −  0  =  0  =  LHS.  Si  x   0  =  1  =  LHS.
  A  y  x     B,  entonces  RHS  =  1  +  
0  −  0  =  1  =  LHS.
Machine Translated by Google

Capítulo  3 Funciones  y  Matrices 923

65.  caso  1  Sea  x  ≥  y.  Entonces  max{x,  y}  =  x  y  min{x,  y}  =  y.  
máx{x,  y}  −  mín{x,  y}  =  x  −  y  =  |x  −  y|
caso  2  Sea  x<y.  Entonces  max{x,  y}  =  y  y  min{x,  y}  =  x.  
max{x,  y}  −  min{x,  y}  =  y  −  x  =  |y  −  x|=|x  −  y|

Ejercicios  3.3  (pág.  141)

1.  si 3.  no 5.  si 7.  no 9.  no


11.  no 13.  sí   15.  no 17.  no 19.  no
21.  no 23.  no,  no  inyectivo. 25.  no,  no  sobreyectiva. 27.  si
29.  153 31.  190 33.  12
ABCDEFGHIJKLM
35.
AL  AZ  CA Florida MAMÁ

NOPQRSTUVWXYZ
NY  OH  MI

37.  demostración:  Sea  f :  A  →  A  tal 39.  prueba:  Como  A     B,  existe  una  


que  f(a)  =  a   a     A.  Dado   biyección  f :  A  →  B.  Sea  g :  A  ×  
que  f  es  la  función  identidad   {1}  →  B  ×  {2}  definida  por  g(a,  1)  =  
en  A,  es  biyectiva.     A     A. (f(a),  2) .  Claramente,  g  está  bien  
definida.
41.  demostración:  Sea  f :  [a,  b]→[c,  
Para  demostrar  que  g  es  inyectiva:  
d]  definida  por  d  −  c  f(x)  =  c  +  
(x  −  a).  b  −  a   Sean  aya  dos  elementos  cualesquiera  
Observe  que  f(a)  = d.  c  y  f(b)  =   en  A  tales  que  g  (a ,  
Entonces  
1)  =  g(a,  
(f(a ),  
1).   2)  =  
(f(a),  2).     a  =  a,  ya  que  f  es  inyectiva.  
Entonces,  (un
Para  demostrar  que  f  es   , 1)  =  (a,  1)  y  por  lo  tanto  g  es
inyectiva:  Sean  α,  β     [a,  b]   inyectable
tales  que  f(α)  =  f(β).  Entonces  d   Para  mostrar  que  g  es  
−  c  (α  −  a)  c  +  b  −  ad  −  c  (β   sobreyectiva:  Sea  (b,  2)  
produce  α  =−    βa;  
)  
e=ntonces  
  c  +  b  −  fa  e  E
s  sto   cualquier  elemento  en  B  ×  {2}.  Como  f  
inyectiva.
es  sobreyectiva,  existe  un  elemento  a  
  A  tal  que  f(a)  =  b.  Entonces  x  =  (a,  
1)     A  ×  {2}  y  g(x)  =  g(a,  1)  =  (f(a),  2)  
=  (b,  2).     g  es  sobreyectiva.
Para  probar  que  f  es  
sobreyectiva:  Sea  β     [c,  d].   Así  g  es  una  biyección  y  por  lo  tanto  el  
Elige  b  −  a  (β  −  c).   resultado.
d  −  c  puede  verificar  
Usted   qα
ue  
  =  
aa    
≤+  α
    
≤  b  y  f(α)  =  β.     f  es  
sobreyectiva.

Así  f  es  biyectiva  y  por  lo  tanto  
[a,  b] [c,  d].
Machine Translated by Google

924 Soluciones  a  ejercicios  impares

43.  demostración:  Sea  f :  Z  →  N  definida  45.  demostración:  Sea  A  =  {a1,  a2, ... ,  an, ...}  by  un  
conjunto  infinito.  Entonces  B  =  {a1,  a2,  a4, ...}  
f :  
es  
A  
u→n    sBubconjunto  
  definida  por  
propio  
f(ai)  =d  e  
a2i  
A.  
eLs  
a  
ufna  
unción  
biyección  (verifique).     A     B.
2n  +  1  si  n  ≥  0  −2n  
f(n)  =
si  n  <  0

Puedes  verificar  que  f  es   Por  el  contrario,  suponga  que  A  es  finito,  B  
inyectiva.   A  y  A     B.  Dado  que  A  es  finito  y  B     A,  |
Para  demostrar  que  f  es   B|  <  |A|.  Pero  como  A     B,  |A|=|B|.  Esto  es  
sobreyectiva:  Sea  m     N.  caso  1  
una  contradicción.     A  es  infinito.
Sea  m  par.  Luego  elige  x  =  −m/2.  
Claramente,  x     Z  y  f(x)  =  −2(−m/2)
47.  demostración:  Sea  f :  Q+  →  N  ×  N
=  metro definido  por  f(m/n)  =  (m,  n),  donde  m  y  n  
caso  2  Sea  m  impar.  Entonces  m   son  primos  relativos.
−  1  elige  x  = .  Otra   Claramente,  f  es  biyectiva.  
vez,
2   Q+     N  ×  N  y,  por  lo  tanto,  Q+  es  
metro  ­  1 contable.
x     Z.  También  f(x)  =  2

2  +1  =  m.     f  es  sobreyectiva. 51.  prueba:  Sea  ||  =  norte  Denote  con  i  el  conjunto  


Así  f  es  biyectiva  y  por  lo  tanto  Z      de  todas  las  palabras  que  contienen  i  
N. símbolos  sobre .  Claramente,  i  es  contable  
(de  hecho  finito)  y  |i|  =  ni  yo  es  un  =     i W
49.  demostración:  Sean  A1,  A2, ...  un   .  Desde  
número  contable  de  conjuntos  
contables. unión  contable  de  conjuntos  contables,     
es  contable.
Para  mostrar  que  A  =   Ai  es  
contable:  dado  que  cada  Ai  es  
contable,  sus  elementos  se  pueden  
enumerar  como  ai1,  ai2, ....  Ahora  
enumere  todos  los  elementos  aij  en  
A  con  i  +  j  =  2,  luego  enumere  todos  
los  elementos  aij  con  i  +  j  =  4,  y  así  
sucesivamente.  Este  procedimiento  
establece  un  procedimiento  
sistemático  para  listar  todos  los  
elementos  en  A.     A  es  contable.

Ejercicios  3.4  (pág.  149)

1.  No.  de  dígitos  en  el  entero  m  =  11
No.  de  dígitos  distintos  disponibles  n  =  10     El  
resultado  sigue  el  principio  del  casillero.

3.  Aplicar  el  principio  del  casillero  con  m  =  6  y  n  =  5.
5.  Elige  m  =  8  y  n  =  7. 7.  Elige  m  =  7  y  n  =  6.
Machine Translated by Google

Capítulo  3 Funciones  y  Matrices 925

9.  Aplicar  el  principio  del  casillero  generalizado  con  m  =  12,305  y  n  =  13.

11.  Sea  x  =  0  ∙  a1a2  ∙∙∙  aib1b2  ∙∙∙  bj.     10i  x  =  
a1a2  ∙∙∙  ai  ∙  b1b2  ∙∙∙  bj  10i+j  x  =  a1a2  ∙∙∙  
aib1b2  ∙∙∙  bj.  b1b2  ∙∙∙  bj     10i+j  x  −  10i  x  =  a1a2  ∙∙∙  
aib1b2  ∙∙∙  bj  −  a1a2  ∙∙∙  ai  a1a2  ∙∙∙  aib1b2  ∙∙∙  bj  −  a1a2  ∙∙∙  ai  10i+j  −  
10i  cual  es  un  número  racional.
  x  =

13.  Une  los  puntos  medios  de  los  lados  para   15.  Divide  el  triángulo  en  nueve  pequeños  
formar  cuatro  pequeños  congruentes triángulos  congruentes:

cuadrados,  cada  uno  de  lado  1/2. Por  PHP,  al  menos  dos  de  los  diez  puntos  
Por  el  PHP,  al  menos  dos  de  los  cinco   deben  estar  dentro  del  mismo  triángulo  
puntos  deben  estar  dentro  de  un  pequeño   pequeño.  Como  cada  lado  mide  1/3  de  
cuadrado.  Como  su  diagonal  mide  √2/2   unidad  de  largo,  el  resultado  es  el  siguiente.
unidades,  la  distancia  entre  los  puntos  
debe  ser  √2/2.
19.  Sea  S  un  conjunto  finito.  Asumir
|S|  no  es  único,  digamos,  |S|  =  my  |  S  |  
17.  Sea  |X|  =  metro,  |Y|  =  n,  y =  norte
Si  =  {f  −1(y)|y     Y}.  Entonces  {Sy}  divide   Para  mostrar  que  m  =  n:  Suponga  que  
X  en  b  bloques,  donde  b  ≤  n.  Supongamos   m>n.  Entonces,  por  PHP,  no  existe  
que  |Sy|  <  m/b   y.  Como  X  es  una  unión   ninguna  función  inyectiva  en  S.
disjunta  de  los  bloques  Sy, Esto  es  una  contradicción,  ya  que  la  
función  identidad  1S  es  inyectiva.     
b metro
metro  ≤  norte .  Del  mismo  modo,  n  ≤  
metro  =  |x|  = |Si|  <  segundo =  m,
m.  Así  m  =  n.
yo=1
b  una  contradicción.  Por  tanto,  uno  de  los  
bloques  Sy  tiene  al  menos  m/b  elementos.  
En  otras  palabras,  existe  un  elemento  t  
  Y  tal
que  f  −1(t)  tiene  al  menos  m/b
elementos.
Como  b  ≤  n  y  m  >  kn,  kn
metro
> =  k.  Entonces  |f  −1(t)|  >  k.  
b  De  ahí  el  resultado.  (Tenga  en  
norte

cuenta  que  k  =  1  produce  el  PHP).

Ejercicios  3.5  (pág.  155)

1.  10 3.  4x2  −  4x  +  2  5.  −3 7.  −5


Machine Translated by Google

926 Soluciones  a  ejercicios  impares

9.  0 11.  0 13.  si  15. x  1234  g(x)  


bdca

17.  si 19.  no  inyectivo;  no   21.  sí  23.  no  inyectivo;  no  invertible


invertible

25.  T 27.  F. 29.  T 31.  T

33.  T 35.  4 37.  16

39.  (1Y  ◦  f)(x)  =  1Y  (f(x))  =  f(x)     1Y  ◦  
41.  Sea  z  cualquier  elemento  de  Z.  Como  
f  =  f g  es  sobreyectiva,  existe  un  elemento  
y  en  Y  tal  que  g(y)  =  z.  Como  f  es  
43.  1)  Para  demostrar  que  1X  es  inyectiva:  
sobreyectiva,  existe  un  elemento  x  en  
Sea  1X  (x)  =  1X  (x ).  Después
X  tal  que  f(x)  =  y.  Entonces  g(f(x))  =  
x  =  x
g(y)  =  z.  Es  decir,  (g  ◦  f)(x)  =  z     g  ◦  f  
  1X  es  inyectivo.
es  sobreyectiva.
2)  Para  demostrar  que  1X  es  sobreyectiva:  
Sea  x  cualquier  elemento  de  X.  Como  1X  
(x)  =  x,  1X  es  claramente  sobreyectiva.

Por  lo  tanto,  1X  es  biyectiva.

45.  demostración:  Sea  z     Z.  Como  g  ◦   47.  Sea  x  cualquier  elemento  en  X  y  sea  


f  es  sobreyectiva,  existe  un  elemento   f(x)  =  y.  Entonces  ◦  f)(x)  =  f  −1(f(x))  
x     X  tal  que  (g  ◦  f)(x)  =  z,  es  decir,   (F−1 =  f  −1(y)
g(f(x))  =  z.  Sea  y  =  f(x). =  x
  f −1 ◦  f  =  1X
Entonces  y     Y  y  g(y)  =  z.  Así,  dado  
cualquier  elemento  z     Z,  existe  un  
elemento  y     Y  tal  que  g(y)  =  z.     
g  es  sobreyectiva.  49.  prueba:  Para  
probar  que  f−1  es  inyectiva:  Sea  f  −1(y1)  =  f  
−1(y2).  Entonces  f(f  −1(y1))  =  f(f  −1(y2)).  Es  decir,  (f  ◦  f  −1)(y1)  =  (f  ◦  f  
−1)(y2)
1  año  ( y1 )  =  1  año  ( y2 )
y1  =  y2
−1      es  inyectable.
f  Para  mostrar   −1 es  sobreyectiva:
que  f  Sea  x     X.  Entonces  f(x)     Y.  Sea  y  
=  f(x).  f  −1(y)  =  f  −1(f(x))  =  (−1 ◦  f)(x)  
f  Así  f =  1X  (x)  =  x.     f −1 es  sobreyectiva.
−1 es  biyectiva.

51.  prueba:  
−1 −1
(g  ◦  f)  ◦  (f ◦  g−1)  =  g  ◦  (f  ◦  (f ◦  g−1))  
=  g  ◦  (f  ◦  f  −1)  ◦  g−1  =  g  ◦  
(1Y  ◦  g−1)  =  g  ◦  g−1

=  1Z
Machine Translated by Google

Capítulo  3 Funciones  y  Matrices 927

−1
Del  mismo  
−1
◦  g−1)  ◦  (g  ◦  f)  =  1X .  ◦  
−1
modo,  (f     (g  ◦f)  
   =  f g−1

53.  (h  ◦  (g  ◦  f))(x)  =  h((g  ◦  f)(x))  =   55.  Sea  A     B.  Entonces  existe  una  


h(g(f(x)))  =  (h  ◦   biyección  f :  A  →  B.  Como  f  es  una  
−1
g)(f(x))  =  ((h  ◦   biyección,  f :  B  →
también  
  A  existe  
es  
yu  na  
g)  ◦  f)(x)     h  ◦  (g   biyección.     B     A.
◦  f)  =  (h  ◦  g)  ◦  f

57.  
demostración:  Para  mostrar  que  f  −1(S     T)     f  
−1(S)     f  −1(T):  Sea  x     f  −1(S     T).  Entonces  
f(x)     S     T.     f(x)     S  o  f(x)     T.  En  otras  palabras,  x     f  −1(S)  o  x  
  f  −1(T).     x     f  −1(S)     f  −1(T).  Así  f  −1(S     T)     f  −1(S)     f  −1(T).
Al  volver  sobre  los  pasos,  se  puede  demostrar  
que  f  −1(S)     f  −1(T)     f  −1(S     T).
De  ahí  el  resultado.

Ejercicios  3.6  (pág.  162)

1.  21 3.  5 5.  20 7.  21 9.  135 11.  19


12   11
17.  T
13 (2k  −  1)  i=1 15. k(k  +  1)  
k=1

19.  S  =  (am+1  −  am)  +  (am+2  −  am+1)  +∙∙∙+  (an  −  an−1)  =  an  −  am
norte norte

21
(yo  +  1)2  −n 2  yo  =  2 yo  +  n 1
yo=1 yo=1 yo=1 yo=1
norte norte

i 2  −n
2  

(n  +  1)2  −  1  + yo
=  2 yo  +  n
yo=2 yo=2 yo=1
norte

n2  +  n  =  2 i
yo=1
norte

n(n  +  1)  
yo  =
yo=1
2

23.  28 25.  350 27.  255 29.  15


31.  24 33.  1 35.  10 37.  210
39.  28 41.  6 43.  480,  480 45.  613

47.  a1j  +  a2j  +  a3j 49.  a11  +  a12  +  a21  +  a22  +  a31  +  a32

51.  2a1  +  2a2  +  2a3 53.  |A1  ∩  A2|+|A1  ∩  A3|+|A2  ∩  A3|
Machine Translated by Google

928 Soluciones  a  ejercicios  impares

Ejercicios  3.7  (pág.  172)

1.  x  =  −1,  y  =  −4,  z  =  −4 3.  −2  3  0  −4 5. 03  2


−1  2  −4
−2  
   5  −6   

−3  −2  5  0  13 0  4  −10  0  0  
7. 9.
−2

2  −6  13  0  49 3  4  −13  06  7
11 13

15.  p  =  r,  q  =  s 17.  metro  =  norte 19.  n  =  p  =  r,  q  =  s

21.  n  =  p,  q  =  r  23.  1470,  875,  1890  27.  prueba:  A  +  B   25.  2025,  1250,  2710

=  (aij)  +  (bij)  =  (aij  +  bij)  =  (bij  +  aij)  =  (bij)  +  (aij )
29.  prueba:  A  +  O  =  (aij)  +  (0)  =  (aij  +  
0)  =  (aij)

=  un
=  B  +  A Del  mismo  modo,  O  +  A  =  A.

31.  prueba:  c(A  +  B)  =  c(aij  +  bij)  =  (caij  +  
cbij)  =  c(aij)
+c(bij)  =  cA  +  cB

33.  Sean  A  =  (aij),  B  =  (bij)  y  C  =  (cij).

b1kck1 b1kck2
BC  =
b2kck1 b2kck2

a1t(b1kck1) a1t(b1kck2)
A(BC)  =
a2t( b2kck1) a2t( b2kck2)

( a1tbt1)ck1 ( a1tbtt1)ck2
=
( a2tbt2)ck1 ( a2tbt2)ck2

=  (AB)C

35.  demostración:  (A  +  
B)C  (a11  +  b11)c11  +  (a12  +  b12)c21  (a11  +  b11)c12  +  (a12  +  b12)c22
=
(a21  +  b21)c11  +  (a22  +  b22)c21  (a21  +  b21)  c12  +  (a22  +  b22)c22
Machine Translated by Google

Capítulo  3 Funciones  y  Matrices 929

=
a11c11+a12c21  a11c12+a12c22   b11c11+b12c21  b11c12+b12c22  
+
a21c11+a22c21  a21c12+a22c22 b21c12+b22c12  b21c12+b22c22

=  AB  +  CA

37. adf 39.  Sea  A  =  (aij).


mendigar AT  =  (aji)     
cfh
      (AT)  T  =  (aij)
=  un

41.  Sean  A  =  (aij)  y  B  =  (bij).
a11b11  +  a12b21  a11b12  +  a12b22  
AB  =
a21b11  +  a22b21  a21b12  +  a22b22

T = a11b11  +  a12b21  a21b11  +  a22b21  
(AB)
a11b12  +  a12b22  a21b12  +  a22b22

b11  b21   a11  a21
TABT  =
b12  b22   a12  a22

=
b11a11  +  b21a12  b11a21  +  b21a22  
b12a11  +  b22a12  b12a21  +  b22a22
T
=  (AB)

T
T  43.  (ABC) =  [A(BC)]
=  (BC)  TAT,  por  el  Ejercicio  41
=  (CTBT)AT,  por  el  Ejercicio  41
=  TPCEAT

1  −2  0 1  6  −2 47. X


1
45  AB  =  17 3 1  −1      
−8  3  −1 y
1   2  −3      
−5  4  −7   

17  0  0
1
= 0  17  0 =  I3
17
0  0  17

2  3   X 4   X 2  3


49. = 51. =
4  5 y 6
y 4  5−1  4 6
2  1   1 5  −3   4  
53.  f(2)  = =
1  1 2 −4  2 6

5  3   1 −1  
55.  f(4)  = = =
3  2 2 2  −4 2

entonces  x  =  −1,  y  =  2.
Machine Translated by Google

930 Soluciones  a  ejercicios  impares

57.  Demostración:  Como  B  es  inversa  de  A,  AB  =  I  =  BA.  Asimismo,  AC  =  I  =
CALIFORNIA.  Entonces  B  =  BI  =  B(AC)  =  (BA)C  =  IC  =  C

59.  demostración:  (AB)(B−1A−1)  =  A[B(B−1A−1)]
=  A[(BB−1)A−1]
=  A(IA−1)
=  AA−1  =  I  
Del  mismo  modo,  (B−1A−1)(AB)  =  I.
−1     (AB) =  B−1A−1

Ejercicios  de  repaso  (pág.  177)

1.  558 3.  761 5.  martes 7  de  julio

9.  miércoles 11.  lunes 13.  1  de  abril 15.  28  de  marzo

17.  no 19.  si 21.  no 23.  8


25.  0 27.  0 29.  5540 31.  5

A B CD  E ... k L
33.
CBS1AA  CBA3BA  AQX5CD ...  NBC4GK  VPS3SL

METRO norte OP ... XY Z


NCR4SK  CNN1TK ABC5ZZ

0 1 2 3 4 5
35.
AQZ5CD  CBS1AA  CNN1TK  VPS3SL  NBC4GK  CBA3BA
6 7 8 9
NCR4SK  ABC5ZZ

37.  Usa  m  =  19,  n  =  6. 39.  −27
41.  −39 43.  1 45.  36
47.  F. 49.  F. 51.  3.4,  4.6
53.  3.4,  4.5 55.  2,  3.5 57.  1,  1,  2,  3
59.  1,  1,  2,  3 61.  25 63.  72

65.  Demostración:  Sean  b1,  b2     B  tales  que  g(b1)  =  g(b2).  Entonces  f(g(b1))  =  
f(g(b2));  es  decir,  (f  ◦  g)(b1)  =  (f  ◦  g)(b2).  Así  1B(b1)  =  1B(b2);  es  decir,  b1  =  
b2.     g  es  inyectivo.
Machine Translated by Google

Capítulo  4 Inducción  y  Algoritmos 931

Ejercicios  complementarios  (pág.  179)

T T
1.  (A  +  B) =  (aij  +  bij)  =  (aji  
3.  (AAT)  T  =  (AT)  TAT  =  

+  bji)  =  (aji)  +(bji) AAT,  ya  que  (AT)  T  =  A.
Entonces  AAT  es  simétrica.
=  AT  +  BT
7.  63.
5.  prueba  (por  contradicción):
9.  134.217.728
Suponga  que  cada  uno  de  los  
números  es  menor  que  su  promedio  A. 11.  65.536
Después
norte
13.  prueba:  ak  =  a  +  (k  −  1)d,  k  ≥  1
ai  <  nA
yo=1 norte

1 norte norte
Sn  =  ak  k=1  norte
=  norte
ai  =  n  i=1 ai,  
i=1
= [a  +  (k  −  1)d
una  contradicción
k=1  
norte norte

15.  Sea  Sn  =  a  +  ar  +∙∙∙+  arn−1  rSn  =  ar   un  +  re  (k  ­  1)  k=1  
=
+  ar2  +∙∙∙+  arn−1  +  arn     rSn  −   k=1  (n  ­  1)n  =  na  +  re  ∙
Sn  =  arn  −  a  a(rn  −  1)

norte

Sn  =  (r  =  1) = 2  [2a  +  (n  −  1)d]  2
r  ­  1

17.  Sean  X  =  {a,  b,  c,  d,  e,  g},  A  =  {a,  b,  c},  B  =  {b,  c,  d,  e}  y  Y  =  {0,  3,  5 }.
Sea  f :  X  →  Y  definida  por  f(a)  =  0,  f(b)  =  f(c)  =  3,  f(d)  =  f(e)  =  f(g)  =  5.  Entonces  A  ∩  B  
=  { b,  c},  f(A)  =  {0,  3},  f(B)  =  {3,  5},  f(A  ∩  B)  =  {3}  =  f(A)  ∩  f(B).  No  obstante,  f  no  es  
inyectiva.


19.  demostración:  Sea  a1  =∙∙∙=  an.
norte
21 an  =  ai  i=n+1
Después ai  ­  norte  +  1  =  2n  ­  norte  +  1
yo=1 ∞
=  n  +  1   =  an+1  +  ai  i=n+2  =  an+1  
+  an+1  =  
Entonces,  por  el  Ejercicio  18,  si  n+1  
2an+1  1     an+1  =  an  2
palomas  ocupan  n  casilleros,  un  casillero  
debe  contener  al  menos  dos  casilleros.

Capítulo  4 Inducción  y  Algoritmos

Ejercicios  4.1  (pág.  188)

1.  si 3 ,  12;  5 5.  −3;  5


Machine Translated by Google

932 Soluciones  a  ejercicios  impares

7.  {0,  1}   9.  {0,  1,  2,  3,  4,  5,  6}
11.  prueba  (por  contradicción): 13.  prueba  (por  contradicción):
Suponga  que  hay  un  entero  n  tal   Supongamos  que  existe  un  
que  0  <n<  1.  Sea  S  =  {a|0  <a<  1}.   entero  n     S  tal  que  −  1.  
Como  n     S,  S  =  Ø.  Por  lo  tanto,   n  <  n0  +  n0   Entonces  −  1  <  
por  WOP,  S  tiene  un  elemento  mínimo   +  n  −  n0  +   .  Ya  que
donde  0  <  <  1.     0  <  2  por  lo  
, <  
t anto  
d onde
0    1     S   y  n  −  n0  +  1  <  
2 <  y  <  1.  
,  no  es  el  menor  elemento  de  
Entonces 2    S ,  una  contradicción.
2 <
, S,  una  contradicción.

15.  prueba  (por  contradicción):  Sea  D  =  S  −  T.  Entonces  D  ∩  T  =  Ø  y  D     T  =  S.  Para  
probar  que  S  =  T,  basta  con  demostrar  que  D  =  Ø.  Suponga  D  =  Ø.  Dado  que  S  está  
bien  ordenado,  también  lo  está  D.  Entonces  D  contiene  un  elemento  mínimo  d.  
Entonces  d  =  a,  ya  que  d     D,  a     T  y  D  ∩  T  =  Ø.  Como  d  =  a  y  S  =  {a,  a  +  1, ...},  d  
−  1  debe  estar  en  S.  Claramente,  d  −  1     D.  Entonces  d  −  1     T.     (d  −  1)  +  1  =  d  
  T,  una  contradicción.  Así  D  =  Ø  y  por  lo  tanto  S  =  T.

Ejercicios  4.2  (pág.  195)

1.  si 3.  no

5.  no,  3  es  un  contraejemplo. 7.  4

9.  4 11.  2=2  ∙  28  +  (−3)  ∙  18

13.  1  =  (−8)  ∙  28  +  15  ∙  15 15.  12

17.  39/18 19.  elige  a  =  3,  b  =  5,  c  =  7.

21.  23  de  abril 23.  13  de  abril
25.  4 27.  16

29 ,  5;  6;  10;  12 31.  prueba:  Dado  que  a|b  y  a|c,  hay  enteros  
positivos  d  y  d  tales  que  b  =  da  y  c  =  
33.  prueba:  Por  la  división d  a.
algoritmo,  existe  un  cociente   Entonces  b  −  c  =  da  −  da  =  (d  
q  tal  que  a  =  bq  +  r.
−  d )a     a|(b  −  c).
Dado  que  d  =  mcd{b,  r},  d  |b  y  d  |r.  
  d  |a.  Así  d  |a  y  d  |b.     re  |  mcd{a,   35.  demostración:  Sea  d  =  mcd{a,  b}  y
b};  es  decir,  d  |d. d  =  mcd{b,  a  +  b}.  Entonces  d|a  y  d|b.  
  d|(a  +  b)  y  por  lo  tanto  d|d .
37.  Suponga  que  p  |ab  yp  |  una.  Ya  que
Dado  que  d  =  mcd{b,  a  +  b},  d  |b  y  
pag  |  a,  mcd{p,  a}  =  1.  Entonces  hay  
d  |(a  +  b).     d  |[(a  +  b)  −  b],  es  decir,  d  
enteros  s  y  t  tales  que  1  =  sp  +  ta.     
|a.  Así  d  |a  y  d  |b,  y  por  lo  tanto  d  |d.     
b  =  spb  +  tabulador.
re  =  re .
Dado  que  p|spb  y  p|tab,  p|b.
Machine Translated by Google

Capítulo  4 Inducción  y  Algoritmos 933

39.  Dado  que  mcd{a,  b}  =  d,  por  el   41.  Sean  d  =  mcd{mcd{a,  b},c}  y  d  =  
teorema  4.6,  hay  enteros  s  y  t  tales  que   mcd{a,mcd{b,  c}}.  Entonces  d|  mcd{a,  
d  =  sa  +  tb.     1  =  s(a/d)  +  t(b/d).  Por   b}  y  d|c.     d|a,  d|b  y  d|c.  Entonces  d|
tanto,  mcd{a/d,  b/d}  =  1,  y  por  lo  tanto  a/ a,  y  (d|b  y  d|c).     d|a  y  d|mcd{b,  c}.
d  y  b/d  son  primos  relativos.

45.  Como  p2  +  8  es  primo,  p  debe  ser  impar.  
Por  el  algoritmo  de  división,  p  es  de  la  
43.  Sean  n  y  n  +  1  dos  cualesquiera forma  3n,  3n  −  1  o  3n  +  1.  Si  p  =  3n  ±  1,  
enteros  consecutivos.  Claramente,   entonces  p2  +  8  =  9n2  ±  6n  +  9  es  
uno  de  ellos  debe  ser  parejo. divisible  por  3.  Entonces  p  =  3  y  por  lo  
Supongamos  que  es  n.  Entonces  2|n.  Pero   tanto  n  =  1  y  por  lo  tanto  p  =  3.
n  es  un  número  primo,  por  lo  que  n  =  2  y,  
por  lo  tanto,  n  +  1  =  3.  El  caso  de  que  n  +  
1  sea  par  da  n  =  1,  lo  cual  es  una   Entonces  p3  +  4  =  31  es  un  número  primo.
contradicción.
49.  Falso.  Elija  a  =  3,  b  =  4,  47.  
Tenemos  p  =  q  +  3.  Suponga  que  q  es  c  =  9.  par.  Entonces  
510511  
q  ==    21  9  
y  ∙p  2
  =6869  
  5.  En  
es  
51.  
el  o
Etro  
7  =  
lado,  sea  q  impar.  compuesto.

Entonces  q  +  3  es  un  primo  par  >  
6,  lo  cual  es  imposible.

Ejercicios  4.3  (pág.  205)

1.  13 3.  1022 5.  10000110100dos

7.  3360ocho 9.  15ocho   11.  72ocho  

13.  1D  dieciséis 15.  75dieciséis 17.  110110dos

19.  1000110111dos 21.  11010dos 23.  87ECdieciséis

25.  10100dos 27.  3071730dos

29.  110,  1011,  10110,  11011,  101010 31.  0,  1

33.  2 35.  5

37.  1)  0     S. 39.  1)  1     S.


2)  Si  x     S,  entonces  0x,  1x     S. 2)  Si  x     S,  entonces  x0,  1x,  x1     S.

41.  165 43.  101 45.  10EFdieciséis

Ejercicios  4.4  (pág.  221)

1.  666 12 12  ∙  13  
3. yo  =  =  78
yo=1
2

(Nota:  en  los  ejercicios  5  a  13,  P(n)  denota  el  enunciado  dado).
Machine Translated by Google

934 Soluciones  a  ejercicios  impares

5.  Paso  base:  Cuando  n  =  1,  LHS  =  1  =  12  =  RHS.     P(1)  es  verdadera.
k
Paso  de  inducción:  suponga  que  P(k)  es  verdadero: (2i  −  1)  =  k2
k yo=1
Después (2i  −  1)  +  (2k  +  1)  =  k2  +  (2k  +  1)  =  (k  +  1)2
yo=1
  P(k  +  1)  es  verdadera.  Por  lo  tanto,  el  resultado  dado  sigue  por  PMI.

7.  Paso  base:  Cuando  n  =  1,  LHS  =  13  =  1  =  RHS.     P(1)  es  verdadera.
k 2
k(k  +  1)
yo  3 =
Paso  de  inducción:  suponga  que  P(k)  es  verdadero:
yo=1
2
Después
k 3
3  
k(k  +  1)
yo +  (k  +  1)3  = +  (k  +  1)2
2
yo=1

k2  +  4k  +  4
=  (k  +  1)2
4

(k  +  1)2(k  +  2)2
=
4
2
(k  +  1)(k  +  2)
=
2

  P(k  +  1)  es  verdadero.  Por  tanto,  el  resultado  es  verdadero  por  inducción.

9.  Paso  base:  Cuando  n  =  1,  n2  +  n  =  1  +  1  =  2  es  divisible  por  2.     P(1)  es
verdadero.

Paso  de  inducción:  suponga  que  P(k)  es  verdadera:  k2  +  k  es  divisible  por  2.  Sea  k2  +  
k  =  2m  para  algún  entero  positivo  m.  Después

(k  +  1)2  +  (k  +  1)  =  k2  +  3k  +  2  =  (k2  +  k)  +  2(k  +  1)

=  2m  +  2(k  +  1)

que  es  divisible  por  2.     P(k  +  1)  es  verdadera.  Así  el  resultado  sigue  por  PMI.

2(2−1)
11.  Paso  base:  cuando  n  =  2,  P(2)  =  1  = 2 .     P(2)  es  verdadera.
Paso  de  inducción:  Suponga  que  P(k)  es  verdadero.  Considere  un  nuevo  punto  Q.  Al  
unir  Q  a  cada  uno  de  los  k  puntos  originales,  obtenemos  k  nuevas  líneas.

k2  
k(k  −  1)     Nº  total  d e  +  k  
puntos  con  k  +  1  puntos  =  +  k  =
2 2
(k  +  1)[(k  +  1)  −  1]
=
2

  P(k  +  1)  es  verdadera.  Así  el  resultado  sigue  por  PMI.

13.  Paso  base:  Claramente,  P(1)  es  verdadero.
Paso  de  inducción:  suponga  que  P(k)  es  verdadera:  si  p|ak,  entonces  p|a.  Supongamos  
p|ak+1.  Como  ak+1  =  ak  ∙  a,  p|(ak  ∙  a).  Entonces  p|ak  o  p|a,  por  el  Ejercicio  33
Machine Translated by Google

Capítulo  4 Inducción  y  Algoritmos 935

en  la  Sección  4.2.  Como  p|ak  por  IH,  P(k  +  1)  es  verdadero.  Así,  el  resultado  se  sigue  
por  inducción.

15.  28,335 17.  n2/4  (n2   si  n  es  par


−  1)/4  de  lo  contrario

21.  n(n  +1)/2
norte

19.  x  = (2i  −  1)  =  n2
yo=1

23.  n(n  +1)(n  +2)/6 25.  n(n  +  1)(2n  +  1)/6  29.  

2  27.  (n!) 2n2(n+1)
i pi+1  −  1   i j k
31.  suma  = pd  =  p  −  1 33.  suma  = PD qs rs  
s=0 s=0 s=0 s=0

rk+1  −  1
= pi+1  −  1   ∙ qj+1  −  1   ∙
pag  −  1 q  −  1 r  ­  1
norte
n(n  +  1)   norte i n(n  +  1)  (n  +  2)  j  =  
35. yo  = 37. 6  i=1  j=1
yo=1 2

39.  Recuerda  que  el  k­ésimo  término  del  41.  caso  1  Sea  n  par.  Entonces  la  secuencia  
geométrica  an  =  0  +  1  +  1  +∙∙∙  a,  ar,  ar2, ...  es  +  
ak  
n/2  
=  ark−1.  
  no.  +d  
e  
(n/2  
granos  
−  1)  e+n  
  (n/2  
el  ú− ltimo  
  1)  
=  2[1  +  2  +∙∙∙+  (n/2  −  1)]  +  n/2  cuadrado  =  n2­ésimo   +  1
término  
 )  +  secuencia  
en  el  2(n/2  
1,  2−,    212,  
)(n/2  
23, ....  
Aquí  2  2  caso  2  Sea  n  impar.  Entonces  a  =  1,  1)/2  r  =  2  y  kn  o.  
=  nd2.  
e  garanos  
n  =  0  +e  n  
1  e+l  
  ú1ltimo  
  +∙∙∙+  (n  −  
cuadrado  +(n  −  1)/2  =  1∙(2n2−1)  =  2n2−1  =  2[1   n2
= +  2  +∙∙∙+  (n  −  1)/2]
norte

=
4

.
(También  podríamos  probar  este  PMI).  
2[(n  −  1)/2][(n  +  1)/2]  2
43.  93 =
n2  −  1
45.  249 =
4
47.  8tn  +  1  =  8n(n  +  1)/2  +  1  =  4n(n   49.  LHS  2  
+  1)  +  1  =  (2n  +  1)2 =  [(n  −  1)n/2]  
=  n2[(n  −  1)2  +  (n   2  +  [n(n  +  1)/2]
+  1)2]/4  =  n2(n2  +  1)/2  =  t
=  lado  derecho
n2

51.  paso  base:  Cuando  n  =  0,  LHS  =  A     B1  =  RHS.  Entonces  P(1)  es  verdadero.

paso  de  inducción:  Supongamos  que  P(k)  es  verdadera:  A     ( k   Bi)  = k   (A     Bi).  Después


∩  i=1 ∩  i=1

k+1
Un   i=1Bi  =  A     ∩   k   Bi  ∩  Bk+1  =  [A   k   Bi ]  ∩  (A     Bk+1)
∩  i=1 ∩  i=1
Machine Translated by Google

936 Soluciones  a  ejercicios  impares

k+1  
= k   (A     Bi)  ∩  (A     Bk+1)  = ∩   (A     Bi)
∩  i=1 i=1

  P(k)  →  P(k  +  1),  por  lo  que  el  resultado  sigue  por  PMI.

53.  paso  base:  Cuando  n  =  1,  LHS  =  p1  =  RHS.     P(1)  es  verdadera.  paso  de  
inducción:  Suponga  que  P(k)  es  verdadero.  Después

(p ∙∙∙ pk pk+1)≡   [(p1 ∙∙∙ pk) pk+1  

≡ (p1 ∙∙∙ pk) pk+1  

≡( p1   ∙∙∙ paquete) paquete+1

  P(k+1)  es  verdadero.  Así  el  resultado  sigue  por  PMI.

55.  paso  base:  Claramente,  P(2)  es  verdadera.  
paso  de  inducción:  Suponga  que  P(k)  es  verdadero.  Por  el  algoritmo  de  división,  k=3q+r,  
donde  0≤r<3.  Entonces  k+1=3q+r+1.  caso  1  Si  r=0,  entonces  k+1=3q+1=3(q−1)+4.  Por  
tanto,  un  cambio  de  3q+1  céntimos  se  puede  pagar  con  q−1  monedas  de  3  céntimos  y  dos  
monedas  de  2  céntimos.  caso  2  Si  r=1,  entonces  k+1=3q+2.  Evidentemente,  un  cambio  de  
3q+2  monedas  se  puede  pagar  con  q  monedas  de  3  céntimos  y  una  moneda  de  2  céntimos.  
caso  3  Si  r=2,  entonces  k+1=3(q+1).  Dicho  cambio  se  puede  pagar  con  monedas  de  (q+1)  
céntimos.  Por  tanto ,  P(k+1)  es  verdadera.

Por  lo  tanto,  el  resultado  sigue  al  PMI.

57.  prueba:  Suponga  que  q(n)  satisface  las  condiciones  del  segundo  principio.
Sea  p(n)=q(n0) q(n0+1) ∙∙∙ q(n),  donde  n≥n0.  Como  q(n0)  es  verdadera,  p(n0)  es  
verdadera.
Suponga  que  p(k)  es  verdadero,  donde  k≥n0.  Por  la  condición  (2),  q(k+1)  es  verdadera.  
Por  lo  tanto,  p(k+1)=q(n0) ∙∙∙ q(k) q(k+1)  es  verdadera.  Entonces  p(k)→  p(k+1).  
Entonces,  por  el  primer  principio,  p(n)  es  verdadero   n≥n0.  Por  lo  tanto  q(n)  es  verdadero  
n≥n0.

59.  n(3n4+7n2+2)/12

Ejercicios  4.5  (pág.  234)

1.  Cuando  n  =  0,  a0  =  1  =  x0.     P(0)  es  verdadera.  Suponga  que  P(k)  es  verdadera:  ak  =  xk.
Después

ak+1  =  ak     x,  por  el  paso  4  

=  xk  ∙  x,  por  la  hipótesis  inductiva  =  xk+1

  P(k  +  1)  es  verdadero.  Por  tanto ,  P(n)  es  un  bucle  invariante  por  PMI.
Machine Translated by Google

Capítulo  4 Inducción  y  Algoritmos 937

3.  Sea  mcd{a,  b}  =  d.  Cuando  n  =  0,  mcd{xn,yn}  =  mcd{x0,y0}  =  mcd{x,  y}.     P(0)  es  
verdadera.  Suponga  que  P(k)  es  verdadera:  mcd{xk,yk}  =  mcd{x,  y}  =  d.  Entonces  
xk  =  yk−1,  yk  =  rk−1,  rk  =  xk  mod  yk,  y  xk  =  qkyk  +  rk.  (1)
Dado  que  d|xk  y  d|yk,  d|rk.
Para  mostrar  que  P(k  +  1)  es  verdadera:  mcd{xk+1,  yk+1}  =  d.
Sea  mcd{xk+1,  yk+1}  =  d .  (2)  xk+1  =  yk ,  yk+1  =  rk.     d|xk+1  y  d|yk+1.  Así  que  d|
d .  (3)
De  (1),  d  |xk+1  y  d  |yk+1.     d  |yk  y  d  |rk.     d  |xk  por  (1).  Así  d  |xk  y  d  |yk.     d  |  d.  (4)

Así  por  (3)  y  (4),  d  =  d .  Por  tanto ,  P(n)  es  un  bucle  invariante.

5.  Cuando  n  =  0,  s0  =  x  =  x  +  0.     P(0)  es  verdadera.  Suponga  que  P(k)  es  verdadero:  
sk  =  x  +  k.  Entonces  sk+1  =  sk  +  1  =  (x  +  k)  +  1  =  x  +  (k  +  1).     P(k  +  1)  es  
verdadera.  Por  tanto ,  P(n)  es  un  bucle  invariante.
7.  3 9.  4

11.  7,  18,  19,  23,  53 13.  7,  18,  19,  23,  53
15. Algoritmo  factorial(n) 17 Producto  de  algoritmo  (A,B,C)
Empezar  (*algoritmo*)   Comenzar  (*algoritmo*)  
hecho  ←  1 para  i  =  1  a  n  hacer  
i  ←  1   para  j  =  1  a  n  hacer
while  i  ≤  n  do  fact   norte

←  fact  *  i cij  =  t=1 aitbtj


Fin  (*algoritmo*) Fin  (*algoritmo*)

19 Algoritmo  suma(X) 21.  Algoritmo  max(X)
Comenzar  (*algoritmo*)   Empezar  (*algoritmo*)  
suma  ←  0  para  i  =  1  a   max  ←  x1  para  i  =  2  a  
n  hacer  suma  ←  suma   n  hacer  si  xi  >  max  
+  xi entonces  max  ←  xi  Fin  
Fin  (*algoritmo*) (*algoritmo*)

23 Algoritmo  de  impresión  (X) 25.  Algoritmo  palíndromo  (S)
Empezar  (*algoritmo*)  i   Empezar  (*algoritmo*)  
←  1 para  i  =  1  an  do
flag  ←  true   escribir  (xi)
while  (i  ≤  n)  y  (flag)  do  si  si  =   Fin  (*algoritmo*)
sn+1−i  entonces  i  ←  i+1

más
bandera  ←  falso
Fin  (*algoritmo*)

(En  los  ejercicios  26  a  36,  P(n)  denota  el  enunciado  de  que  el  algoritmo  funciona  
correctamente).

27.  Inicialmente,  bandera  =  verdadero  e  i  =  j  =  1.  
paso  base:  Cuando  n  =  1,  si  a11  =  b11,  bandera  sigue  siendo  verdadera,  entonces  A  =  B;  
de  lo  contrario,  bandera  =  falso  (línea  10),  por  lo  que  A  =  B.  En  ambos  casos,  P(1)  es  verdadero.
Machine Translated by Google

938 Soluciones  a  ejercicios  impares

paso  de  inducción:  Suponga  que  P(k)  es  verdadero.  Para  demostrar  que  P(k  +  1)  es  
verdadera,  basta  con  mostrar  que  el  algoritmo  funciona  correctamente  para  las  (k  +  1)  
filas  y  columnas  de  A  y  B.
Cuando  j  =  k  +  1,  el  bucle  interno  (líneas  6  a  10)  comprueba  si  ai(k+1)  =  bi(k+1)  o  no.  
Entonces,  cuando  i  varía  de  1  a  k  +  1,  el  ciclo  externo  (líneas  4  a  12)  verifica  si  las  
(k+1)  primeras  columnas  de  A  y  B  son  iguales.  Cuando  i  =  k  +  1,  el  bucle  interior  
comprueba  si  las  (k  +  1)  filas  de  A  y  B  son  iguales.  Así,  P(k)  →  P(k  +  1).  Entonces,  por  
inducción,  el  algoritmo  funciona  correctamente   n  ≥  1.

29.  Inicialmente,  bandera  =  verdadero,  e  i  =  j  =  
1.  paso  base:  cuando  n  =  1,  si  a11  ≤  b11,  bandera  sigue  siendo  verdadera,  entonces  A  
=  B;  de  lo  contrario,  bandera  =  falso  (línea  10),  por  lo  que  A  ≤  B.  En  ambos  casos,  P(1)  
es  verdadero.  paso  de  inducción:  Suponga  que  P(k)  es  verdadero.  Para  demostrar  que  
P(k  +  1)  es  verdadera,  basta  con  mostrar  que  el  algoritmo  funciona  correctamente  para  
las  (k  +  1)  filas  y  columnas  de  A  y  B.
Cuando  j  =  k  +  1,  el  bucle  interior  (líneas  6  a  10)  comprueba  si  ai(k+1)  ≤  bi(k+1)  o  no.  
Entonces,  cuando  i  varía  de  1  a  k  +  1,  el  ciclo  externo  (líneas  4  a  12)  verifica  si  la  (k+1)  
columna  de  A  es  ≤  la  de  B.  Cuando  i  =  k+1,  el  ciclo  interno  verifica  si  la  (k  +  1)  ª  fila  de  
A  es  ≤  la  de  B.
Por  lo  tanto,  P(k)  →  P(k  +  1).  Entonces,  por  inducción,  el  algoritmo  funciona  
correctamente   n  ≥  1.

31.  paso  base:  Cuando  n  =  1,  A  =  [a11]  y  B  =  [b11].  Entonces  c11  =  a11b11  por
línea  3.     P(1)  es  
verdadera.  paso  de  inducción:  suponga  que  P(k)  es  verdadera,  es  decir,  el  algoritmo  
funciona  correctamente  para  dos  matrices  k  ×  k  cualesquiera.  En  otras  palabras,  la  
línea  3  da  el  valor  correcto  de  cij  para  1  ≤  i,  j  ≤  k.  Ahora  supongamos  que  n  =  k  +  1.  Por  la  línea  4,
k  +1 k  +1
aitbtj  = aitbtj  +ai(k+1)b(k+1)j  da  el  valor  correcto  de  cij,  por  la
t=1 t=1
hipótesis  inductiva.  Por  lo  tanto,  los  bucles  for  anidados  dan  el  valor  correcto  de  cij   i,  
j.     P(k)  →  P(k  +  1).  Entonces,  por  inducción,  el  algoritmo  funciona  correctamente   n  
≥  1.

33.  paso  base:  cuando  n  =  1,  se  omiten  las  líneas  2  a  4.  Entonces,  el  algoritmo  devuelve  el  
valor  correcto  de  la  línea  mínima  1.     P(1)  es  verdadero.  paso  de  inducción:  Suponga  
que  P(k)  es  verdadero.  Supongamos  que  el  algoritmo  se  invoca  con  n  =  k  +  1  elementos.  
Como  i  varía  de  2  a  k,  el  ciclo  for  encuentra  el  mínimo  de  los  primeros  k  números.  
Cuando  n  =  k  +  1,  este  valor  se  compara  con  xk+1  y  el  mínimo  de  x1  a  xk+1  se  
devuelve  en  la  línea  4.     P(k)  →  P(k  +  1).  Entonces,  por  inducción,  el  algoritmo  
funciona  correctamente   n  ≥  1.

35.  paso  base:  cuando  n  =  1,  se  imprime  xn−i+1  =  x1−1+1  =  x1 .     P(1)  es

verdadero.  paso  de  inducción:  Suponga  que  P(k)  es  verdadero.  Supongamos  que  el  
algoritmo  se  invoca  con  n  =  k  +  1  elementos.  Cuando  i  =  1,  se  imprime  xk+1 .  Entonces  como
Machine Translated by Google

Capítulo  4 Inducción  y  Algoritmos 939

i  varía  de  2  a  k,  xk,  xk−1, ... ,  x1  se  imprimen,  por  la  hipótesis  inductiva.     
P(k)  →  P(k  +  1).  Entonces,  por  inducción,  el  algoritmo  funciona  correctamente  
n  ≥  1.
37.  2,  3,  5,  6,  8,  13

39.  paso  base:  cuando  n  =  1,  se  salta  el  ciclo  for .  Entonces  P(1)  es  verdadero  por
por  
defecto.  paso  de  inducción:  Suponga  que  P(k)  es  verdadero.  Supongamos  
que  el  algoritmo  se  invoca  con  n  =  k  +  1  elementos.  Cuando  i  =  2,  el  algoritmo  
coloca  el  mayor  de  los  k  +  1  elementos  en  la  posición  correcta.  Esto  deja  una  
sublista  con  k  elementos.  Por  la  hipótesis  inductiva,  el  algoritmo  lo  ordena  
correctamente.     P(k)  →  P(k  +  1).  Entonces,  por  inducción,  el  algoritmo  
funciona  correctamente   n  ≥  1.

Ejercicios  4.6  (pág.  246)

1.  O(n) 3.  O(n3) 5.  O(largo  n)

7.  O(n  largo  n) 9.  O(n3) 11.  O(n2)


13.  2n  =  2  ∙  2  ∙∙∙∙∙  2  (n  veces)  ≤  2  
norte

15. i  k  ≤  nk  +∙∙∙+  nk
∙  3  ∙∙∙∙∙  n  =  O(n!) yo=1

=  n(nk)  =  O(nk+1)

19.  O(n2) 21.  O(n3)  
norte norte norte

17 yo(yo  +  1)  =  
2  
yo  +  yo  =  1   i
yo=1 yo  =  1
23.  (n) 25.  (n3)  
n(n  +  1)(2n  +  1)  + n(n  +  1)
= 27.  (lg  n)  31.   29.  (n3)
6 2
=  O(n3) (n2)
33.  (3n)!  =  3  ∙  6  ∙  9  ∙∙∙(3n) 35.  suma  =  n2  =  (n2)
=  3n!
37.  2n  +  3  ≥  2n  =  (n)
≥  3n2n,  donde  n  ≥  4  =  
39.  2n3  −  3n2  +  4n  ≥  2n3,
(6n)
donde  n  ≥  2  =  (n3)

41.  3lg  n  +  2  ≥  3lg  n  =  (lg  n) 43.  Cierto,  ya  que  sum  =  n2/4  si  n  es  
par  y  (n2  −  1)/4.

45.  Como  f1(n)  =  O(n(g(n)),  | 47.  Como  f(n)  =  O(h(n)),  |f(n)|  ≤  A|h(n)|  
f1(n)|  ≤  A|g(n)|  para  alguna   para  alguna  constante  A.
constante  A  >  0 ,  f2(n)  =  kf1(n).   Del  mismo  modo,  |g(n)|  ≤  B|h(n)|  
  |f2(n)|  =  k|f1(n)|  ≤  kA|g(n)|  =  C| para  alguna  constante  B.  Entonces  
g(n)|,  donde  C  =  kA. |(f  +  g)(n)|=|f(n)  +  g(n)|  ≤  |f(n)|+|g(n)|  
≤  A|h(n)|  +  B|h(n)|
=  O(g(n))
Machine Translated by Google

940 Soluciones  a  ejercicios  impares

49.  |f(n)|  ≤  A|g(n)|  para  alguna  constante   ≤  C|h(n)|+C|h(n)|  ≤  2C|
A  >  0.  |g(n)|  ≤  B|h(n)|  para  alguna   h(n)|
constante  B  >  0.     |f(n)|  ≤  AB|h(n)|   =  O(h(n))  
=  C|h(n)|,  donde  C  =  AB. donde  C  =  máx{A,  B}

=  O(h(n))

51.  Supongamos  que  f(n)  =  (g(n)).  Entonces  f(n)  =  O(g(n)),  f(n)  =  (g(n)).  Entonces  |f(n)|  ≤  B|
g(n)|  y  |f(n)|  ≥  A|g(n)|  para  algunas  constantes  A  y  B.  Así  A|g(n)|≤|f(n)|  ≤  B|g(n)|.

Por  el  contrario,  sea  A|g(n)|≤|f(n)|  ≤  B|g(n)|.  Como  A|g(n)|≤|f(n)|,  f(n)  =  (g(n)).  Del  mismo  
modo  f(n)  =  O(g(n)).  Entonces  f(n)  =  (g(n)).

Ejercicios  4.7  (pág.  251)

1.  No.  de  entradas  en  cada  matriz  =  n2 3.  sn  =  n(n  −  1)

  no.  de  adiciones  requeridas  =  n2  =   5.  No.  de  adiciones  =  n  =  O(n)

O(n2) 7.  an  =  n  =  O(n)

9.  cn  =  norte  −  1  =  O(n ) 11.  37,  2 13.  O(n)

Ejercicios  de  repaso  (pág.  254)

1.  2 3.  3 5.  1024 7.  2989

9.  11110101dos 11.  2305dos 13.  100001dos 15.  D033dieciséis

17.  265ocho 19.  463ocho 21.  B5dieciséis 23.  133dieciséis

25.  n2

n(n  +  2)/4  si  n  es  par  (n  +  
27.  un  =
1)2/4  en  caso  contrario

29.  demostración:  Sea  d  =  mcd{a  −  b,  a  +  b}.  Entonces  d|(a  −  b)  y  d|(a  +  b);  entonces  d|[(a  −  
b)  +  (a  +  b)].  Es  decir,  d|2a.  Del  mismo  modo,  d|2b.  Pero  a  y  b  son  primos  relativos.  
Entonces  d|2.     re  =  1  o  re  =  2.

31.  Claramente,  P(1)  es  verdadera.  Suponga  que  P(k)  es  verdadera;  es  decir,  3|
(k3  −  k).  (k  +  1)3  −  (k  +  1)  =  (k3  +  3k2  +  3k  +  1)  −  (k  +  1)
Como  3|(k3  −k),  la  RHS  es  divisible  por  3.     P(k+1)  es  verdadero.  Entonces  P(n)  es  
verdadero   n  ≥  1.

33.  Cuando  n  =  1,  LHS  = = 1  3
=  lado  derecho.     P(1)  es  verdadera.
1  (2−1)(2+1)
Machine Translated by Google

Capítulo  4 Inducción  y  Algoritmos 941

k 1 k
Supongamos  que  P(k)  es  verdadero: Entonces  +.  
  (2i  −  1)
(2i  +  1)  2k  +  1  i=1

k  +1 k
1 1 1
= +  
(2i  −  1)(2i  +  1) (2i  −  1)(2i  +  1)  (2k  +  1)(2k  +  3)  i=1
yo=1

k 1 k(2k  +  3)  +  1  
= +   =
+  1  (k  +  1)(2k  
(2k  ++    11)  
)(2k  
(2k  ++    31)  
)(2k  
2k   (2k  +  1)(2k  +  3)
+  3) k  +  1  
= =
(k  +  1)  +  1

  P(k)  →  P(k  +  1).  Entonces  el  resultado  se  mantiene  por  inducción.

35.  Sea  P(n):  Debemos  seleccionar  al  menos  2n  +  1  calcetines  para  asegurar  n  pares  
coincidentes.  Por  PHP,  P(1)  es  verdadero.  Supongamos  que  P(k)  es  verdadero,  es  decir,  
debemos  seleccionar  al  menos  2k+1  calcetines  para  asegurar  que  coincidan  k  pares.  
Agregue  un  par  coincidente.  Esto  asegura  k  +  1  pares  coincidentes  seleccionando  (2k  +  
1)  +  2  =  2(k  +  1)  +  1  calcetines.     P(k)  →  P(k  +  1).  Entonces  P(n)  es  verdadero   n  ≥  1.

5(n  +  1)n2 39.  2n+2  −  n  −  4  41.  3n(n+1)(n+2)/6  43.  (n  +  1)!  −  1
37.
2
metro

45. 5i /n,  donde  m  es  el  entero  más  grande  tal  que  5m  ≤  n.
yo=1

47. t19  +  t20  +  t21  +  t22  +  t23  +  t24  =  t25  +  t26  +  t27  +  t28  t29  +  t30  +  t31  +  
t32  +  t33  +  t34  +  t35  =  t36  +  t37  +  t38  +  t39  +  t40

49.  tn  +  tn−1tn+1  =  n(n  +  1)/2  +  (n  −  1)n/2  ∙  (n  +  1)(n  +  2)/2
=  [n(n  +  1)/2][1  +  (n2  +  norte  −  2)/2]  =  
[n(n  +  1)/2][n(n  +  1)/2]  2  =  t  norte

Ejercicios  complementarios  (pág.  256)

1.  (m2  −  n2)  2  2  +  (2mn) =  (m4  −  2m2n2  +  n4)  +  4m2n2  =  m4  
+  2m2n2  +  n4  =  (m2  +  n2)  2

3.  tk  =  k(k  +  1)/2.  Sea  n  =  m(m  +  1)/2.  Entonces  (2k  +  

1)2  ∙  m(m  +  1)  (2k  +  1)2n   k(k  +  1)  2
+  tk  = +
2
[(2k  +  1)2(m2  +  m)  +  k(k  +  1)]
=
2
[(2k  +  1)2m2  +  (2k  +  1)2m)  +  k(k  +  1)]
=
2
[(2k  +  1)m  +  k][(2k  +  1)m  +  (k  +  1)] N(N  +  1)
= =
2 2
Machine Translated by Google

942 Soluciones  a  ejercicios  impares

donde  N  =  (2k  +  1)m  +  k     (2k  +  1)2n  +  tk  es  un  número  triangular.

5.  prueba:  supongamos  que  111  es  un  cuadrado  perfecto  a2  en  alguna  base  b,  entonces  
a2  =  b2  +  b  +  1  <  (b  +  1)2.  Entonces  (b  +  1/2)2  =  b2  +  b  +  1/4  <  b2  +  b  +  1.  Es  decir,  (b  
+  1/2)2  <  a2  <  (b  +  1)2.  Esto  produce  b  +  1/2  <a<b  +  1;  es  decir,  a  se  encuentra  entre  
b  +  1/2  yb  +  1,  lo  cual  es  imposible.  Por  lo  tanto,  111  no  puede  ser  un  cuadrado  en  
ninguna  base.

7.  25  y  36  son  Duffinian,  pero  18  y  43  no  lo  son.

9.  σ(n)  = re  +  norte  =  s  +  norte.     Ninguno  de  los  factores  de  n,  excepto  1,  divide  a  s


d|n  
d=n

si  y  solo  si  ninguno  divide  a  σ(n).  Por  lo  tanto,  n  es  Duffiniana  si  y  solo  si  ninguno  de  
los  factores  de  n,  excepto  1,  divide  a  σ(n).

11.  En  el  Ejercicio  59  de  la  Sección  4.4,  establecimos  que  Sn  −Sn−1  =  n(3n4  +  7n2+2)/12.  
Dado  que  tanto  Sn  como  Sn−1  son  números  enteros  positivos,  se  deduce  que  
n(3n4+7n2+2)/12  también  es  un  número  entero.  En  consecuencia,  12|n(3n4+7n2+2).  
13.  n3 15.  Georgia

Capítulo  5 recursividad

Ejercicios  5.1  (pág.  273)

1.  1,  4,  7,  10 3.  1,  2,  3,  4 5.  1,  1,  2,  4

7.  1,  3,  4,  7,  11,  18 9.  3

11.  A(0)  =  1000  13.  $1024  15.  a1  =  1  A(n)  =  1.  015A(n−1),  n  ≥  1  an  =  
an−1+3,  n  ≥  2

17.  a0  =  0
an  =  2an−1  +  3,  norte  ≥  1

19.  00000,  00001,  00010,00011,00100,00101,00110,01000,01001,01010,
01011,01100,01101

21.  S1  =  A1  Sn   23.  S0  =  1  Sn   25.  a1  =  a  an  


=  Sn−1     An,  n  ≥  2 =  2Sn−1,  n  ≥  1 =  ran−1,  n  ≥  2
Machine Translated by Google

Capítulo  5 recursividad 943

27.  91 29.  91 31.  f(99)  =  f(f(110))  =  f(100)  =  f(f(111))  


=  f(101)  =  91

33.  Sea  k  el  entero  más  pequeño  tal  90  ≤  x  +   35.  429
11k  ≤  100.  Entonces  f(x)  =  f(f(x  +  11))  =  
37.  1,  2,  1.66666667,  1.5,
f(f(f(x  +  11  ∙  2)))
1.66666667,  1.6,  1.625,  1.  
.. 61538462,  1.61904762,  
. 1.61764706,  1.61818182,  
=  fk  +1(x  +  11k) 1.61797753
Como  90  ≤  x  +  11k  ≤  100  y  k  ≥  1,  39.  2Fn−2  
+  Fn−3  f(x  +  11k)  =  91,  
por  el  Ejercicio  43.
=  Fn−2  +  (Fn−2  +  Fn−3)     
fk  +  1(x  +  11k)  =  fk  (91)
=  Fn−2  +  Fn−1  
Dado  que  f(91)  =  91  por  el  ejercicio  32,  =  Fn  f  k(91)  =  91.     f(x)  =  91  
para  0  ≤  x  <  90.

41.  prueba  (por  PMI):  cuando  n  =  1,  F5n  =  F5  =  5  es  divisible  por  5.  Por  lo  tanto,  P(1)  es  
verdadera.  Suponga  que  P(k)  es  divisible  por  5:  F5k  es  divisible  por  5.

F5(k+1)  =  F5k+5  =  F5k+4  +  F5k+1
=  F5k+2  +  2F5k+3

=  F5k+2  +  2(F5k+1  +  F5k+2)

=  3F5k+2  +  2F5k+1

=  3(F5k+1  +  F5k)  +  2F5k+1

=  3F5k  +  5F5k+1

que  es  claramente  divisible  por  5.     P(k+1)  es  verdadera.  Así,  el  resultado  se  sigue  
por  inducción.

43.  prueba  (por  PMI):  Cuando  n  =  1,  F1  =  F2  =  1  ≤  2.     P(1)  es  verdadera.  Suponga  que  
P(i)  es  cierto  para  todo  i  ≤  k.  Después

Fk+1  =  Fk  +  Fk−1
≤  2k  +  2k−1

≤  2k  +  2k

=  2k+1

  P(k  +  1)  es  verdadero.  Así,  el  resultado  se  sigue  por  inducción.

45.  |A|=−1.  |Un|  =  Fn+1Fn−1  −  F2 norte. Pero  |An|=|A|  n  =  (−1)n


Machine Translated by Google

944 Soluciones  a  ejercicios  impares

47.  prueba  (por  PMI):  Sea  P(n)  el  enunciado  dado.  Cuando  n  =  1,
0
lado  derecho  =  3   Lk  =  3  +  0  =  3  =  L2  =  LHS.  Entonces  P(2)  es  verdadero.  Asumir
+  k=1
2  i−1
P(i)  es  verdadera:  L2i  =  3  + Lc.  Para  mostrar  que  P(i  +  1)  es  verdadera:
k=1
L2i+2  =  L2i  +  L2i+1,  por  definición.
2  
i−2  =  3  + Lk  +  L2i+1
k=1
2  
i−2  =  3  + Lk  +  (L2i−1  +  L2i)
k=1
2i
=  3  +  k=1 Lk

  P(i  +  1)  es  verdadera.  Entonces  P(n)  es  verdadero   n     N.

49.  Como  α  y  β  son  soluciones  de  la  ecuación  x2  =  x  +  1,  α  +  β  =  1.  α2  −  β2  b2  =  =  α  +  β  =  1  
α  −  β

51.  u1  =  α  +  β  =  1,  por  el  ejercicio  49.

53.  Como  α  y  β  son  soluciones  de  la  ecuación  x2  =  x  +  1,  α2  =  α  +  1  y
β2  =  β+  1.  

un−1  +  un−2  =  (αn−1  +  βn−1)  +  (αn−2  +  βn−2)  =  αn−2(α  +  1)  +  

βn−2(β  +  1 )  =  αn−2  ∙  α2  +  βn−2  ∙  β2  =  αn  

+  βn  =  un

55.  2 57.  a1  =  1  an   59.  a1  =  1,  a2  =  3  an  =  


=  an−1  +  norte,  norte  ≥  2 an−1  +an−2,  n  ≥  3

61.  a1  =  2,  a2  =  3  an  =   63.  15
an−1  +  an−2,  n  ≥  3  Sea  bn  =  
65.  3
an−2.  Entonces  b3  =  a1  =  2  y  b4  =  a2  
=  3.  Además,  bn−1  +  bn−2  =  bn.     bn   67.  7
=  Fn,  norte  ≥  3.
Así,  an  =  Fn+2,  n  ≥  1

69.  prueba  (por  PMI):  cuando  n  =  0,

IZQ  =  A(2,  0)  =  A(1,  1)
=  A(0,  A(1,  0))  =  1  +  A(1,  0)
=  1  +  A(0,  1)
=  1  +  2  =  3  =  lado  derecho
Machine Translated by Google

Capítulo  5 recursividad 945

  P(0)  es  verdadera.  Suponga  que  P(k):  A(2,  k)  =  3  +  2k.  Entonces  

A(2,  k  +  1)  =  A(1,  A(2,  k))
=  A(1,  3  +  2k)
=  A(0,  A(1,  2  +  2k))
=  1  +  A(1,  2  +  2k)
=  1  +  A(0,  A(1,  1  +  2k))
=  2  +  A(1,  1  +  2k)
=  2  +  A(0,  A(1,  2k))
=  3  +  A(1,  2k)  =  3  
+  (2k  +  2)  =  3  +  
2(k  +  1)

  P(k  +  1)  es  verdadero.  Así  el  resultado  sigue  por  PMI.

71.  prueba  (por  PMI):  cuando  n  =  0,

IZQ  =  A(3,  0)  =  A(2,  1)
=  A(1,  A(2,  0))  =  A(1,  3)
=  2  +  3  =  23  −  3  =  lado  derecho

  P(0)  es  verdadera.  Suponga  que  

P(k):  A(3,  k)  =  2k+3  −  3  A(3,  k  +  1)  
=  A(2,  A(3,  k))
=  A(2,  2k+3  −  3)  =  3  
+  2(2k+3  −  3)  =  2k+4  
−  3

  P(k  +  1)  es  verdadero.  Así  el  resultado  sigue  por  PMI.

Ejercicios  5.2  (pág.  282)

1.  sn  =  2n,  n  ≥  1

3.  an  =  n(n  +  1)/2  +  1,  n  ≥  1

5.  an  =  2n(n  +  1),  n  ≥  0

7.  sn  =  n(n  +  1)(2n  +  1)/6,  n  ≥  1  9.  Cuando  

n  =  0,  s0  =  20  =  1.     P(0)  es  verdadero.  Suponga  que  P(k)  es  verdadero:  sk  =  2k.
Entonces  sk+1  =  2sk  =  2(2k)  =  2k+1.     P(k  +  1)  es  verdadero.  En  consecuencia,  el  
resultado  sigue  por  PMI.

11.  Cuando  n  =  0,  a0  =  0  ∙  1/2  +  1  =  1.     P(0)  es  verdadera.  Suponga  que  P(k)  es  verdadera:  
ak  =  k(k+1)/2+1.  Entonces  ak+1  =  ak+(k+1)  =  [k(k+1)/2+1]+  (k  +  1)  =  (k  +  1)(k  +  2)/2  +  1.  
  P(k  +  1)  es  cierto.  Así  el  resultado  sigue  por  PMI.
Machine Translated by Google

946 Soluciones  a  ejercicios  impares

13.  Cuando  n  =  0,  a0  =  2(0)(0  +  1)  =  0.     P(0)  es  verdadero.  Suponga  que  P(k)  es  verdadera:  
ak  =  2k(k  +  1).  Entonces  ak+1  =  ak  +  4(k  +  1)  =  2k(k  +  1)  +  4(k  +  1)  =  2(k  +  1)(k  +  2)     
P(k  +  1)  es  verdadero.  Por  lo  tanto,  el  resultado  es  cierto  por  PMI.
15.  Cuando  n  =  1,  s1  =  1  ∙  2  ∙  3/6  =  1.     P(1)  es  verdadera.  Suponga  que  P(k)  es  verdadero:  
sk  =  k(k+1)(2k+1)/6.  Entonces  sk+1  =  sk  +(k+1)2  =  k(k+1)(2k+1)/6+  (k  +  1)2  =  (k  +  1)(k  
+  2)(2k  +  3) /6.     P(k  +  1)  es  verdadero.  Así  el  resultado  sigue  por  PMI.

17.  Cuando  n  =  0,  A(0)  =  1000(1.08)0  =  1000.  Entonces  P(0)  es  verdadero.  Ahora  suponga  
que  P(k)  es  verdadero:  A(k)  =  1000(1.08)k.  Entonces  A(k+  1)  =  (1.08)A(k)  =  
1000(1.08)k+1.     P(k  +  1)  es  verdadero.  Por  lo  tanto,  el  resultado  dado  sigue  por  PMI.

19.  Cuando  n  =  1,  f(1)  =  1  ∙  2/2  +  1  =  2.  Entonces,  P(1)  es  verdadera.  Ahora  suponga  que  
P(k)  es  verdadera:  f(k)  =  k(k  +  1)/2  +  1.  Entonces  f(k  +  1)  =  f(k)  +  (k  +  1)  =  k(k  +  1) /2  +  
1  +  (k  +  1)  =  (k  +  1)(k  +  2)/2  +  1.     P(k  +  1)  es  verdadera.  Por  lo  tanto,  el  resultado  
dado  sigue  por  PMI.
21.  demostración  (mediante  PMI):  Sea  P(n)  el  enunciado  de  que  an  es  una  solución  de  la  
relación  de  recurrencia.  Claramente,  P(1)  es  verdadera.  Suponga  que  P(k)  es  cierto  para
k

un  k  ≥  1  arbitrario :  ak  =  a0  + f(yo).  Después
yo  =  
1k k  +1

ak+1  =  ak  +  f(k  +  1)  =  [a0  + f(i)]  +  f(k  +  1)  =  a0  + f(yo)


yo=1 yo=1

23.  demostración  (mediante  PMI):  Sea  P(n)  el  enunciado  de  que  an  es  una  solución  de  la  relación  
de  recurrencia.  Claramente,  P(1)  es  verdadera.  Supongamos  que  P(k)  es  verdadero:
k

ak  =  cka0  + ck−i  f(i).  Después
yo=1
k

ak+1  =  cak  +  f(k  +  1)  =  c[cka0  + ck−i  f(i)]  +  f(k  +  1)
yo=  
1k  +1
=  ck+1a0  + ck+1−i  f(i)
yo=1

  P(k  +  1)  es  verdadero.  Así  el  resultado  sigue  por  PMI.
25.  caso  1  Sea  n  =  1.  Entonces  la  sentencia  x  ←  x  +  1  se  ejecuta  cero
veces.  Entonces  a1  
=  0.  caso  2  Sea  n  >  1  e  incluso.  Cuando  n  es  par,  n/2  =  n/2.     an  =  
an−1  +  n/2  caso  3  Sea  n  >  1  e  impar.  Cuando  n  es  par,  n/2  =  (n  −  1)/2.  
  an  =  an−1  +  (n  −  1)/2

27.  a1  =  1  an   29 n(n  +  2)   si  n  es  par


un  =
=  an−1  +  n/2,  n  ≥  2 (n  +  1)2/4  de  lo  contrario
31.  an  =  n(n  +  1)/2,  n  ≥  1
33.  an  =  n(n  +  1)(2n  +  1)/6,  n  ≥  1  37.  an  =  
35.  an  =  n(n  +  1)(n  +  2)/6,  n  ≥  1
[n(n  +  1)/2]  2,  n  ≥  1
39.  tn  =  n(n  +  1)/2
Machine Translated by Google

Capítulo  5 recursividad 947

41. 43.  pn  =  n(3n  −  1)/2

1 si  n  =  1
45.  hn  =
hn−1  +  4(n  −  1)  +  1  si  norte  ≥  2

47.  pn  +  tn  −  n  =  n(3n  −  1)/2  +  n(n  +  1)/2  −  n  =  2n2  −  n  
=  (2n  −  1)n  =  hn

49.  T1  =  1  
Tn  =  Tn−1  +  n(n  +  1)/2,  n  ≥  2

51.  Probaremos  por  PMI  que  Tn  =  n(n+1)(n+2)/6   n  ≥  1.  Cuando  n  =  1,  T1  =  1  ∙  2  ∙  3/6  =  
1,  lo  cual  es  cierto.  Entonces  la  fórmula  funciona  cuando  n  =  1.
Suponga  que  funciona  para  un  entero  positivo  arbitrario  k.  Usando  la  relación  de  
recurrencia,  Tk+1  =  Tk  +  (k  +  1)(k  +  2)/2  =  k(k  +  1)(k  +  2)/6  +  (k  +  1)(k  +  2)/  2  =  (k  +  
1)(k  +  2)(k  +  3)/6

Así,  por  PMI,  la  fórmula  se  cumple  para  todo  n  ≥  1.

53.  Sn  =  Sn−1  +  n2  =  
Sn−2  +  (n  −  1)2  +  n2  =  
Sn−2  +  (n  −  2)2  +  (n  −  1)2  +  n2
..
.
=  S1  +  22  +  32  +∙∙∙+  n2  =  12  
+  22  +  32  +∙∙∙+  n2  =  n(n  +  1)
(2n  +  1)/6
Entonces  conjeturamos  que  Sn  =  n(n  +  1)(2n  +  1)/6,  donde  n  ≥  1.
55.  1 57.  2

59.  a0  =  1,  a1  =  2  an   1 si  n  =  0
61.  un  =
=  an−1  +  an−2,  n  ≥  2 (1  +  an−1)/k  si  n  ≥  1

63.  demostración  (mediante  PMI):  Sea  P(n)  el  enunciado  dado.  Claramente  P(0)  es  
verdadero.  Así  que  suponga  que  P(m)  es  cierto  para  cualquier  m  ≥  0.  Entonces  am+1  

=  (1  +  am)/k  km  +  k  −  2  =  1  +  km(k  −  1)k
km+1  +  k  −  2  
=
km+1(k  −  1)
Por  lo  tanto ,  P(k)  implica  P(k  +  1),  por  lo  que  el  resultado  se  sigue  de  PMI.

Ejercicios  5.3  (pág.  296)

1.  si 3.  si 5.  no


Machine Translated by Google

948 Soluciones  a  ejercicios  impares

7.  si 9.  an  =  2(−1)n  +  2n,  n  ≥  0

11.  an  =  3(−2)n  +  2  ∙  3n,  n  ≥  0 13.  an  =  Fn+2,  n  ≥  0

15.  Ln  =  αn  +  βn 17.  an  =  2  ∙  3n  −  norte  ∙  3n ,  norte  ≥  0

19.  an  =  2n  −  3n  +  n  ∙  3n,  n  ≥  0  21.  an  

=  2n  +  (−2)n+1  +  2  ∙  3n  −  (−4)n,  n  ≥  0  23.  an  =  3  ∙  2n  −  

n2n  +  n22n  +  2  ∙  3n,  n  ≥  0  25.  a( p)  =  an  +  b

norte
27.  a( p)  =  an2  +  bn  +  c
norte

29.  a( p)  =  (an  +  b)2n  si  2  es  una  raíz  característica;  de  lo  contrario,  un  (p)
norte norte
=
nm(an  +  b)2n.

31.  a(p)  =  an22n
norte
33.  a(p)  =  n2(an2  +  bn  +  c)2n  37.  an  
norte

35.  an  =  2n  −  1,  norte  ≥  0   =  11  ∙  4n  −  11  ∙  3n  −  n  ∙  3n+1,  n  ≥  0
39.  an  =  1  +  n(n  +  1)/2,  norte  ≥  0

41.  Dado  que  rn  y  sn  son  soluciones  de  la  ecuación  (9),  rn  =  arn−1  +  brn−2  y  sn  =  asn−1  +  
bsn−2.     rn  +  sn  =  a(rn−1  +  sn−1)  +  b(rn−2  +  sn−2).  Entonces  an  es  una  solución  de  
(9).

43.  Como  α  es  raíz  de  x3  −  ax2  −  bx  −  c  =  0  con  multiplicidad  3,

x3  −  ax2−  bx  −  c  =  (x  −  α)     3α  3 =  x3  −  3αx2  +  3α2x  −  α3


=  a,  3α2  =  −b  y  α3  =  c (1)

Además,  3 =  aα2  +  bα  +  c (2)  

α  a)  Multiplique  la  ecuación  (2)  por  αn−3:  αn  =  aαn−1  +  bαn−2  +  cαn−3     αn   (3)


es  una  solución.

b)  Cuando  an  =  nαn,

aan−1  +  prohibición−2  +  can−3

=  a(n  −  1)αn−1  +  b(n  −  2)αn−1  +  c(n  −  3)αn−3

=  n(aαn−1  +  bαn−2  +  cαn−3)  −  (aαn−1  +  2bαn−2  +  3cαn−3)  =  nαn  −  

(3αn  −  6αn  +  3αn),  por  las  ecuaciones  (1)  =  an  =  nαn

  nαn  es  una  solución.

c)  Cuando  an  =  n2αn,

aan−1  +  prohibición−2  +  can−3

=  a(n  −  1)2αn−1  +  b(n  −  2)2αn−1  +  c(n  −  3)2αn−3
Machine Translated by Google

Capítulo  5 recursividad 949

=  n2(aαn−1  +  bαn−2  +  cαn−3)  −  2n(aαn−1  +  2bαn−2  +  3cαn−3)  +  

(aαn−1  +  4bαn−2  +  9cαn−3)

=  n2αn  −  2n(3αn  −  6αn  +  3αn)  +  (3αn  −  12αn  +  9αn),  por  (1)  =  an  

=  n2αn

  n2αn  es  una  solución.

Ejercicios  5.4  (pág.  306)

2 1 2 3 8 7x+4  
1. − 3. +   5.  −  +  13(x2+1)  111.  
3(2+3x)  
an  =  
x  ­  1 x  +  3 1  +  2x 1  ­  3x 2n,  n  ≥  0
1  −  x 2x x  −  1   2x  +  1  
7. +   9. +
x2  +  2  x2  +  3  13.   x2  +  1 x2  −  x  +  1  
an  =  2n  −  1,  norte  ≥  1 15.  an  =  3(−2)n  −  2n ,  norte  ≥  0
17.  an  =  5  ∙  2n  −  3n,  norte  ≥  0 19.  an  =  Fn+3,  n  ≥  0
21.  an  =  (2n  +  3)2n,  n  ≥  0 23.  an  =  3(−2)n  +  2n  −  3n,  n  ≥  0  27.  
25.  an  =  2n  +  3n  ∙  2n  −  3n,  norte  ≥  0 an  =  n  ∙  2n+1  −  n22n,  n  ≥  0

29.  an  =  (3n2  −  n  +  2)(−1)n  −  2n+1,  n  ≥  0

Ejercicios  5.5  (pág.  314)

1.  2 3.  13 5.  2


7.  7 9.  an  =  Fn  −  1  
11.  Algoritmo  Fibonacci(n) 13.  sí
Comenzar  (*algoritmo*)  si  (n   15.  2,  3,  5,  7,  9,  11,17,  19
=  1)  o  (n  =  2)  luego  responder  ←  1  
de  lo  contrario  comenzar  anterior   17.  Algoritmo  Lucas  (n)
←  1  actual  ←  1  para  i  =  3  a  n  
(*  Este  algoritmo  calcula  el  enésimo  
comenzar  respuesta  ←  anterior  +   número  de  Lucas  usando  recursividad  
actual *)
Empezar  (*Lucas*)  si  n  
=  1  entonces
Lucas  ←  1  
más  si  n=2  entonces
Lucas  ←  3  
anterior  ←  actual  actual  ←   más
respuesta
Lucas  ←  Lucas(n  ­  1)  +
final  para
Lucas  (n  ­  2)
terminar
Fin  (*Lucas*)
Fin  (*algoritmo*)
Machine Translated by Google

950 Soluciones  a  ejercicios  impares

19.  Suma  del  algoritmo  (X,i,n) 21.  Producto  algorítmico  (X,i,n)
(*  Este  algoritmo  calcula  la  suma  de  los   (*  Este  algoritmo  calcula  el  producto  
números  x  i  a  través  de  una  lista  X  de   de  los  números  x  i  a  x  n  en  una  lista  X  de  
izquierda  a  x  n  derecha.  *) izquierda  a  derecha  *)

Empezar  (*suma*)  si   Empezar  (*producto*)  si  i  =  
i  =  n  entonces n  entonces
suma  ←  x   producto  ←  x  yo  
yo  más más
suma  ←  x  yo +  suma  (X,i+1,n) producto  ←  x  i  *  producto
Fin  (*suma*) (X,i+1,n)
Fin  (*producto*)
23.  Algoritmo  máximo  
(X,i,n,max) 25.  Impresión  del  algoritmo  en  
orden  (X,i,n)
(*  Este  algoritmo  determina  el  máximo  de  
los  elementos (*  Este  algoritmo  imprime  los  elementos  
xi  a  xn  de  una  lista  X.  Se  devuelve  en  la   de  un
x  i  a  x  n  de  la  lista  X  usando  recursividad  
variable  max.  *) *)
Comenzar  (*  imprimir  en  orden  *)
Empezar  (*  máximo  *)  si  i  ≤   si  i  =  n  entonces  
n  entonces  empezar  (*   escribe(x  n )  
si  *) sino
si  yo  =  1  entonces
max  ←  x  1   comenzar  
otra  cosa  si  x  i  >  max  entonces escribir(x  i )  imprimir  en  
max  ←  x  i   orden(X,i+1,n)  terminar
máximo  (X,i+1,n,max)  endif Fin  (*  imprimir  en  orden  *)

Fin  (*máximo*)
27.  Ver  Algoritmo  9  en  la  Sección  6.
31.  Clasificación  de  burbuja  de  algoritmo  (X,  n)
29.  Algoritmo  palíndromo  
(w,i,j,flag) (*  Este  algoritmo  ordena  una  lista  X  de  n  
elementos.  Sorted  es  una  variable  booleana.  
(*  Este  algoritmo,  mediante  
Swap  es  un  algoritmo  que  intercambia  dos  
recursividad,  determina  si  una  palabra  
elementos.  *)
w  de  longitud  j  ­  i  +  1  es  un  palíndromo  o  no.  *)

Comenzar  (*  clasificación  de  
Comience  (*  palíndromo  *)  si  i  =  
burbuja  *)  ordenado  ←  falso
j  entonces
para  j  =  1  a  n  hacer  si  x  j  
flag  ←  true  else   >
xj+1  luego  cswap(x  
omenzar  
si  w  i  =  w  j  entonces  
begin  flag  ←  
j,xj+1 )
true  palindrome  
(w,i+1,  j­1,  flag)  
ordenado  ←  falso
endif
terminara  si

si  no  se  ordena  entonces
clasificación  de  burbuja  (X,  n­1)
otra  
Finalizar  (*  clasificación  de  burbuja  *)
marca  ←  falso
Fin  (*palíndromo*) 33.  4,  5,  6,  7,  8,  11,  13
Machine Translated by Google

Capítulo  5 recursividad 951

Ejercicios  5.6  (pág.  318)

En  los  ejercicios  1  a  29,  P(n)  denota  el  siguiente  enunciado:  El  algoritmo  funciona  correctamente.

1.  prueba:  cuando  n  =  0,  el  algoritmo  devuelve  el  valor  1  de  la  línea  2.  Entonces  funciona  
correctamente  cuando  n  =  0.
Suponga  que  P(k)  es  verdadera,  es  decir,  ¡suponga  que  el  algoritmo  devuelve  el  valor  
k!  para  un  entero  arbitrario  k  ≥  0.  Supongamos  que  invocamos  el  algoritmo  con  n  =  k  +  1.  
Entonces  factorial  =  (k  +  1)  *  factorial  =  (k  +  1)     k!  =  (k  +  1)!.     P(k  +  1)  es  verdadero.  
Así,  por  inducción,  el  algoritmo  funciona  correctamente   n  ≥  0.

3.  prueba:  Sea  bn  el  número  de  movimientos  necesarios.  Cuando  n  =  1,  el  disco  1  se  mueve  
de  X  a  Z  y  b1  =  1  (línea  4).  Entonces  P(1)  es  verdadero.
Suponga  que  P(k)  es  verdadero.  Supongamos  que  se  invoca  el  algoritmo  con  n  =  k+1.
Por  la  línea  8,  transfiere  correctamente  los  n  −  1  =  k  discos  superiores  de  X  a  Y;  cuenta  
=  bk.  El  disco  k  +  1  se  mueve  de  X  a  Z  (línea  9);  cuenta  =  bk  +  1.
Por  el  IH,  el  algoritmo  mueve  los  discos  k  en  Y  a  Z  usando  movimientos  bk  (línea  11).  
Ahora  cuente  =  (bk  +  1)  +  bk  =  2bk  +  1.  Por  lo  tanto ,  P(k)  →  P(k  +  1).
Así  el  resultado  sigue  por  PMI.

5.  prueba  (por  inducción  fuerte):  Inicialmente,  bajo  =  1  y  alto  =  n.  Cuando  n  =  1,  mid  =  1  (línea  
4).  Si  clave  =  x1,  entonces  la  lista  contiene  clave.  Si  clave  =  x1,  el  algoritmo  se  invoca  en  
la  línea  9  u  11.  En  cualquier  caso,  la  condición  de  la  línea  1  no  se  cumple;  por  lo  que  el  
algoritmo  devuelve  el  valor  falso  para  encontrado.

Suponga  que  P(i)  es  verdadero   i  ≤  k,  k  ≥  1.  Suponga  que  el  algoritmo  se  invoca  con  
n  =  k  +  1;  luego  bajo  =  1,  alto  =  k  +  1  y  medio  =  (k  +  2)/2.  Si  clave  =  Xmid,  P(k  +  1)  es  
verdadero.  De  lo  contrario,  se  ejecuta  la  línea  9  o  la  línea  11.
En  cualquier  caso,  el  algoritmo  se  invoca  para  una  sublista  que  contiene  <  k  elementos.  
Por  el  IH,  el  algoritmo  funciona  correctamente  en  cada  caso.  Entonces  P(k)  →  P(k  +  1).  
Así  el  resultado  sigue  por  PMI.

7.  prueba:  cuando  n  =  1,  sum  =  x1  por  la  línea  2.  Entonces  P(1)  es  verdadero.  Suponga  que  
P(k)  es  cierto  para  algunos  k  ≥  1.  Suponga  que  se  invoca  el  algoritmo  para  una  lista  con  
k  +  1  elementos.  Si  i<k  +  1,  entonces  el  algoritmo  calcula  la  suma  correcta  de  los  primeros  
k  elementos  de  la  hipótesis.  Cuando  i  =  k  +  1,  se  ejecuta  la  línea  2.  Luego  sum  =  xk+1  se  
suma  a  la  suma  anterior  en  la  línea  4.     P(k  +  1)  es  verdadero.  Así,  por  PMI,  el  resultado  
tiene   n  ≥  1.

9.  prueba:  cuando  n  =  1,  el  producto  =  x1  por  la  línea  2.  Entonces,  P(1)  es  verdadero.  
Suponga  que  P(k)  es  cierto  para  un  k  ≥  1  arbitrario.  Suponga  que  se  invoca  el  algoritmo  
para  una  lista  con  k+1  elementos.  Si  i<k+1,  entonces  el  algoritmo  calcula  correctamente  
el  producto  de  los  primeros  k  elementos  por  la  hipótesis  inductiva.  Cuando  i  =  k  +  1,  se  
ejecuta  la  línea  2.  Entonces  producto  =  xk+1  producto  anterior  en  *la  
es  
línea  
verdadero.  
4.     P(k  
A+sí  
  1e)  l  
resultado  sigue  por  PMI.
Machine Translated by Google

952 Soluciones  a  ejercicios  impares

11.  prueba:  cuando  n  =  1,  la  lista  contiene  un  elemento.  Entonces  P(1)  es  verdadero.  
Ahora  suponga  que  P(k)  es  verdadera.  Considere  una  lista  X  con  k+1  elementos.  
Cuando  i  =  1,  max  =  x1  de  la  línea  3.  En  la  línea  7,  se  invoca  el  algoritmo  para  una  
lista  con  k  elementos.  Por  la  hipótesis  inductiva,  el  valor  máximo  correcto  de  max  
y  los  k  elementos  se  devuelve  en  la  línea  7.  Entonces,  P(k  +  1)  es  verdadera.
Así  el  resultado  sigue  por  PMI.

13.  Prueba:  cuando  n  =  1,  x1  se  imprime  en  la  línea  2.  Entonces,  P(1)  es  verdadero.  
Suponga  que  P(k)  es  verdadera.  Considere  una  lista  con  k  +  1  elementos.  Luego  
se  ejecutan  las  líneas  4  y  5.  En  la  línea  5,  se  imprime  x1 .  Los  k  elementos  
restantes  se  imprimen  en  orden  por  la  línea  6,  por  la  hipótesis  inductiva.     P(k  +  
1)  es  verdadero.  Así,  por  PMI,  P(n)  es  verdadero   n  ≥  1.  De  ahí  el  resultado.

15.  Igual  que  el  Ejemplo  4.36.

17.  prueba  (por  inducción  fuerte):  Cuando  n  =  1,i  =  1  =  j;  bandera  =  verdadero  de  la  
línea  2.     P(1)  es  verdadero.  Suponga  que  P(i)  es  verdadero   i  ≤  k.  Considere  
una  palabra  con  k  +  1  elementos.  caso  1  Si  w1  =  wk+1,  entonces  flag  =  true  por  la  
línea  5.  Además,  el  algoritmo  se  invoca  con  i  =  2  y  j  =  k  para  una  palabra  con  k  −  
1  elementos.  Por  la  hipótesis  inductiva,  el  algoritmo  funciona  correctamente  para  
tal  palabra.  caso  2  Si  w1  =  wk+1,  entonces  flag  =  false  por  la  línea  9.  Entonces  
también  el  algoritmo  funciona  correctamente.

Así,  en  ambos  casos,  P(k+1)  es  verdadera.     Por  PMI,  el  resultado  tiene   n  ≥  1.


19.  0 21.  3 23.  lg  n

25.  Claramente,  el  algoritmo  funciona  cuando  n  =  1  o  n  =  2.  Entonces,  P(1)  y  P(2)  son  
verdaderos.  Suponga  que  P(k)  es  cierto  para  cualquier  k  ≥  2.  Suponga  que  el  
algoritmo  se  invoca  con  n  =  k+1  (≥  3).  En  la  línea  9,  el  número  anterior  (Fk−1)  y  el  
número  actual  (Fk)  se  suman  para  obtener  Fk+1.  Entonces  P(k)  →  P(k  +  1).
Así  el  resultado  sigue  por  PMI.

Ejercicios  5.7  (pág.  329)

1.  O(n2) 3.  O(n2) 5.  O(n2) 7.  O(n2n)


9.  3 11.  3 13.  5 15.  5

17.  bn  =  norte,  norte  ≥  0 19.  O(n2) 21.  O(n3)

23.  O(n2) 25.  O(n4)
0 si  n  =  1 29.  O(n2)
27.  mil  millones  =
bn−1  +  (n  −  1)  si  n  ≥  2 33.  cn  =  [n(n  +  1)/2  −  1]b,  norte  ≥  1
31.  cn  =  nb  +  1,  norte  ≥  0   37.  f(n)  =  lg  n  +  1
35.  f(n)  =  ( n  −  1)2  +  1,  norte  ≥  1
Machine Translated by Google

Capítulo  5 recursividad 953

39.  f(n)  =  (nº  de  bits  en  la  representación  binaria  de  n)
+  (nº  de  multiplicaciones)  ≤  (nº  
de  bits)  +  (nº  de  bits  en  n  −  1)  =  (lg  n  +  1)  +  

lg  n  =  2lg  n  +  1  =  O(lg  n)

41.  O(n3) 43.  cn  =  (a  +  1)n  −  1  =  O(n)

45.  f(n)  =  af(n/b)  +  g(n)

=  a2f(n/b2)  +  ag(n/b)  +  g(n)  =  

a3f(n/b3)  +  a2g(n/b2)  +  ag(n/b)  +  g(n)
..
.
k  ­1
=  akf(n/bk)  + ai  g(n/bi )
yo=0

k  ­1
=  akf(1)  + ai  g(n/bi )
yo=0

47.  Sea  n  =  2k.  Por  el  ejercicio  45, 49.  Por  el  ejercicio  48,
k   k  ­1
−1  an  =  2  ∙  0  + 2i  (n/2i ) f(n)  =  dak  +  cn yo  (a/b)
0
0  k  
=  0  +  nk  =  nlg  n =  0  +  cn −1  1  =  ckn
=  O(nlg  n) 0
=  O(nlg  n)

51.  cn  =  2n  −  2 53.  5a  +  12b,  7a  +  19b

55.  Como  n  es  una  potencia  de  2,

a si  n  =  1
t(n)  =
2t(n/2)  +  bn  en  caso  contrario

  Por  el  ejercicio  45,
k  ­1 bn  
t(n)  =  2kt(1)  + 2i  2i
yo=0

=  a2k  +  kbn  =  
an  +  kbn

=  Cn  +  Ckn,  donde  C  =  máx{a,  b}  ≤  Ckn

=  O(nlg  n),  ya  que  k  =  lg  n
Machine Translated by Google

954 Soluciones  a  ejercicios  impares

57.  Por  el  ejercicio   1 si  n  =  1
59.  hn  =
56,  f(n)  =  2cn2  +  (d  −   hn−1  +  1/n  si  n  ≥  2
2c)n  ≤  Cn2  +  Cn  ≤  
2Cn2  =  O(n2)

donde  C  =  máx{2c,  d  −  2c}.  
61.  demostración  (por  inducción):  Sea  P(m)  el  enunciado  dado.  Como  h20  =  h1  
=  1  ≥  1  +  0/2,  P(0)  es  verdadera.
Suponga  que  P(k)  es  verdadera:  h2k  ≥  1  +  k/2.  Después

2  k+1  
1  h2k  +  1  =
yo=1 i
k  2
1 1 1
= ++∙∙∙+  
yo=1 i 2k  +  1 2k+1
1 1
k  ≥  1  +  2   +  +∙∙∙+  2k  +  1  
2k+1  1

k  ≥  1  +  2 +  2k  ∙  
2k+1  
=  1  +  2 k  +  1  1  
1  =  1  +  k +
2
  P(k)  →  P(k  +  1).  Así  que  el  resultado  sigue  por  PMI.
norte
1 norte
dx
63.  hn  =  i=1 ≤
i 1 X
norte

=  lnx |
1

=  en  norte  ­  en  1  =  en  norte  ­  0

=  ln  n  =  (ln  2)lg  n
=  O(lg  n)
65.  prueba:  
caso  1  Sea  n  una  potencia  de  b,  digamos,  n  =  bk.  Después
f(n)  =  af(n/b)  +  c  =  
a2f(n/b2)  +  ac  +  c  =  
a3f(n/b3)  +  a2c  +  ac  +  c
..
.
=  akf(n/bk)  +  ak−1c  +∙∙∙+  a2c  +  ac  +  c
k  ­1

=  akd  +  c ai , (1)  ya  que  f(n/bk)  =  f(1)  =  d.
yo=0
Machine Translated by Google

Capítulo  5 recursividad 955

Como  a  =  1,  f(n)  =  d  +  ck  =  d  +  c  logb  n  =  O(lg  
n)  caso  2  No  sea  
n  una  
(Nota:  logb  n  =  log2  n  ∙  logb  2)
potencia  de  b.
Supongamos  que  bk  <n<bk+1  para  algún  entero  positivo  k.  Como  f  no  es  
decreciente,  esto  implica  que  f(n)  ≤  f(bk+1).
Si  a  =  1,  entonces  por  el  subcaso  (1)  anterior,

f(bk+1)  =  d  +  c(k  +  1)  =  (c  +  d)  +  ck

≤  (c  +  d)  +  c  logb  n,  ya  que  k  <  logb  n.     

f(n)  ≤  (c  +  d)  +  c  logb  n
=  O(lg  n)

67.  Utilizando  el  Ejercicio  45  con  g(n)  =  cn2  y  n  =  bk,
k  ­1
f(n)  =  akf(1)  + ai  (cn2/b2i )
yo=0

k  ­1
=  akd  +  cn2 (a/b2)  yo

yo=0

k  ­1
=  akd  +  cn2 1,  si  a  =  b2
yo=0

=  n2d  +  cn2k,  ya  que  ak  =  b2k  =  n2

=  n2d  +  cn2  logbn  _

69.  Si  a  =  b2,  por  el  ejercicio  67,

f(n)  =  n2d  +  cn2(log2  n)(logb  2)  =  n2d  +  Cn2lg  n,  donde  C  =  c  logb  2.
=  O(n2lgn )

Si  a  =  b2,  entonces  por  el  ejercicio  68,  f(n)  =  An2  +  Bak.  Si  a<b2,  ak  <  b2k;  es  
decir,  ak  <  n2.

  f(n)  ≤  An2  +  Bn2
=  (A  +  B)n2  =  O(n2)

Si  a>b2,  entonces  ak  >  b2k;  es  decir,  ak  >  n2.

  f(n)  ≤  Aak  +  Bak
=  (A  +  B)ak  =  (A  +  B)alog  n

=  (A  +  B)nlogb  a =  O(nlogb  a)
Machine Translated by Google

956 Soluciones  a  ejercicios  impares

Ejercicios  de  repaso  (pág.  334)

1.  7 3.  a1  =  0  =  a2  an  
=  an−1  +  an−2  +  1,  norte  ≥  3

5.  h(n)  =  2n(n  +  1),  norte  ≥  1 7.  an  =  n(n  +  1)(n  +  2)/3,  n  ≥  1
9.  an  =  2n  −  1,  norte  ≥  1

11.  Sea  P(n):  an  =  n(n  +  1)(n  +  2)/3.  Cuando  n  =  1,  a1  =  1  ∙  2  ∙  3/3  =  2.  Entonces  P(1)  es  
verdadero.  Suponga  que  P(k)  es  verdadera.  Entonces  ak+1  =  ak  +  (k  +  1)(k  +  2)  =  k(k  
+  1)(k  +  2)/3  +  (k  +  1)(k  +  2)  =  (k  +  1)(k  +  2)[k/3  +  1]  =  (k  +  1)  (k  +  2)(k  +  3)/3.  Así  P(k)  
→  P(k  +  1).  Así  que  el  resultado  sigue  por  PMI.

13.  Sea  P(n):  an  =  2n  −  1.  Dado  que  a1  =  21  −  1  =  1,  P(1)  es  verdadera.  Suponga  que  P(k)  
es  verdadera:  ak  =  2k  −  1.  Entonces  ak+1  =  ak  +  2k  =  (2k  −  1)  +  2k  =  2k+1  −  1.
Entonces  P(k)  →  P(k  +  1).  Así  el  resultado  sigue  por  PMI.

6  14  28  17.  an  =  (−1)n  +   28
15.  an  =  Fn+2 2n  −  5
5 25(−2)n  + 253n,  n  ≥  0

5 10  
3n  +  ∙  7n  +  1−333  
44 173  19.  an  =  ∙   + , norte  ≥  0
144 12n 9

21.  an  =  (n  +  1)/2 23.  cn  =  (n  −  1)n/2
≤  (n  +1)/2 27.  c1  =  0  cn  
=  O(n) =  cn−1  +  3n,  norte  ≥  2
25.  8

29.  cn  =  3n(n  +  1)/2  =   31.  9 33.  mil  millones  =  2n

O(n2)

35.  a1  =  2,  a2  =  4,  a3  =  7  an  =   37.  an  =  Fn
an−1  +  an−2  +  an−3,  n  ≥  4

39.  Algoritmo  suma  (n) 41.  Sea  P(n):  El  algoritmo  funciona  
Empezar  (*suma*)  si   correctamente     n  ≥  1.  Cuando  n  =  1,  
n  =  1  entonces sum  =  1,  por  la  línea  2.  Entonces  P(1)  
suma  ←  1 es  verdadero.
más   Suponga  que  P(k)  es  verdadera.  
suma  =  suma  (n­1)  +  (2n­1) Supongamos  que  n  =  k  +  1.  Luego,  
Fin  (*suma*) por  la  línea  4,  sum  =  sum  (k)+(2k+1),  
que  es  la  suma  de  los  primeros  k  
43.  c1  =  0  cn  
enteros  positivos  impares,  por  el  IH.  
=  cn−1  +  2,  norte  ≥  2
  P(k)  →  P(k  +  1).  Así  el  resultado  
45.  cn  =  O(n) sigue  por  PMI.
Machine Translated by Google

Capítulo  5 recursividad 957

47.  prueba  (por  PMI):  Cuando  n  =  1, 49.  prueba  (por  PMI):  Cuando  n  =  1,  LHS  =  
LHS  =  1  =  RHS;  entonces  el  resultado  es   L1  =  1  =  L3;  entonces  el  resultado  es  
verdadero  cuando  n  =  1.  Ahora  suponga   verdadero  cuando  n  =  1.
que  es  verdadero  para  an  para  un  entero   Ahora  suponga  que  es  cierto  para  un  
positivo  arbitrario  k.  Después entero  positivo  arbitrario  k.
Después
k  +1 k
F2i−1  = F2i−1  +  F2k+1 k  +1 k
yo=1 yo=1 Li  =   Li  +  Lk+1
yo=1 yo  =  1
=  F2k+F2k+1  =  F2(k+1)
=  (Lk+2  −  3)  +  Lk+1
Entonces,  por  PMI,  el  resultado  es  
verdadero   n  ≥  1. =  Lk+3  −  3

Por  tanto,  el  resultado  es  
verdadero   n  ≥  1.

51.  prueba  (por  PMI):  Cuando  n  =  1,  LHS  =  3  =  RHS;  entonces  el  resultado  es  verdadero  cuando  
n  =  1.  Ahora  suponga  que  es  verdadero  para  un  entero  positivo  arbitrario  k.
Después

k  +1 k  +1
L2i  = L2i  +  L2k+2  =  (L2k+1  −  1)  +  L2k+2
yo=1 yo=1
=  L2k+3  −  1

Así,  por  PMI,  el  resultado  es  verdadero   n  ≥  1.

53.  F2n   −  F2
+1 n−1 =  (Fn+1  +  Fn−1)(Fn+1  −  Fn−1)
=  LnFn  =  F2n

55.  Sea  P(n)  el  enunciado  dado.  Cuando  n  =  2,  RHS  =  F2x  +  F1  =  x  +  1  =  x2  =  LHS.     P(2)  es  
verdadera.
Supongamos  que  P(k)  es  verdadero:

xk  =  xFk  +  Fk−1.  Entonces  
xk+1  =  x2Fk  +  xFk−1  =  (x  +  1)Fk  
+  xFk−1  =  x(Fk  +  Fk−1)  +  
Fk  =  xFk+1  +  Fk

Así  P(k)  →  P(k  +  1).  Así  que  el  resultado  sigue  por  PMI.
2 n−1  
59.  A(n)  =  (n  +  1)  + A(i)  
n  i=2
n−1
nA(n)  =  n(n  +  1)  +  2 Ai)
yo=2
n−2
  (n  −  1)A(n  −  1)  =  (n  −  1)n  +  2  i=2 Ai)

nA(n)  −  (n  −  1)A(n  −  1)  =  2n  +  2  A(n  −  1)  nA(n)  =  2n  +  
(n  +  1)A(n  −  1)
Machine Translated by Google

958 Soluciones  a  ejercicios  impares

A(n)   UN(n  ­  1)   2
= +  
n  +  1 nortenorte  +  1
UN(norte  ­  2)  2  +  +   2
= norte  +  1
norte  ­  1 norte

..
.
A(1)  2  2  +  +   2
= +∙∙∙+  3n  
  4+    1
2  
0  2  +   2
= +∙∙∙+  2  3  norte  +  
1  norte  +1  1  =  2

yo=3
i

Ejercicios  complementarios  (pág.  338)

1.  t1  =  1  
tn  =  tn−1  +  n(n  +  1)/2,  n  ≥  2  −  2fn  

2  3.  f
norte
+  2  =  (22n  +  1)2  −  2(22n  +  1)  +  2  =  22n+1  +  
22+  1n  +  1  −  22+1n  −  2  +  2  =  22n+1  +  1  =  
fn+1  5.  (solución  de  VG  Feser)  a)  Considere  

la  sucesión  {an},  donde  an+2  =  an+1  +  an.  Entonces  an+3  =  
an+2  +  an+1  =  an+2  +  (an+2  −  an).  Así  an  +  an+3  =  2an+2  (1)     La  secuencia  
satisface  la  relación  de  recurrencia  dada.  b)  Usando  las  condiciones  dadas,  a4  =  2−  
a3  
a1  =  2a3  −  1

a5  =  5  
a6  =  2a5  −  a3  =  10  −  a3
..
.
a12  =  290  −  70a3  =  144     
a3  =  2  y  a4  =  3.  Dado  que  a5  =  2a4−a2  =  5,  a2  =  1.  Por  lo  tanto,  la  sucesión  {an}  
satisface  las  condiciones  dadas  y  la  relación  de  recurrencia,  por  lo  que  debe  ser  la  
secuencia  de  Fibonacci  secuencia;  es  decir,  an  =  Fn.

7.  Sn  =  (3n2  +  5)n3/8  9.  an  =  an+5,  n     Z  11.  8,  8

13.  f(n)  =  φ(n)  +  ∙∙∙  +  φ(2)  +  φ(1).  Como  φ(n)  es  par  para  n  >  2  y  φ(2)  +  φ(1)  =  1  +  1  =  2  es  
par,  f(n)  es  par.

15.  f1  =  2,  f2  =  3  fn  =  
fn−1  +  fn−2,  norte  ≥  3

17.  f(n,  1)  =  1
1  si  n  =  0  o  1
f(n,  n)  =
0  de  lo  contrario

f(norte,  k)  =  f(norte  ­  2,  k  ­  1)  +  f(norte  ­  1,  k)
Machine Translated by Google

Capítulo  6 combinatoria 959

Capítulo  6 combinatoria

Ejercicios  6.1  (pág.  349)

1.  1,317 3.  1,449   5.  312 7.  18

9.  200 11.  36n/2 13.  3n 15.  1000

17.  0 19.  260 21.  100,000 23.  0

25.  1.757.600 27.  0 29.  1.572.120.576 31.  186,624

33.  36,504 35.  32 37.  128 39.  192

41.  16 43.  24 45.  2n2 47.  364

49.  9 51.  1,053 53.  1  +  m  +  m2


min+1  −  1 57.  m! ¡norte!

55. 59.
metro  ­  1 (n  −  m)!

b1  61.  Sea  m  =  pa1 ...pam  
q ...qbm  
mm  y  n  =   1 ...qbm  m .  Como  mcd  {m,  n}  =  1,  mn  =
1  b1
pa1  
1 ...pam  m  q1 es  la  descomposición  en  factores  primos  de  mn.     τ  (mn)  =
(a1  +  1)...(am  +  1)(b1  +  1)...(bm  +  1)  =  τ  (m)τ  (n)

Ejercicios  6.2  (pág.  358)

1.  5 3.  60 5.  F.


7.  T 9.  F. 11.  F.
13.  F. 15.  F. 17.  20

19.  362,880 21.  40,320 23.  358.800


25.  1 27.  562,432 29.  208.827.062.548
31.  103,680 33.  1.693.440 35.  60.480
37.  120 39.  6 41.  6 43.  7

(n  ­  1)!  r(n  −  1)!
45.  P(n  −  1,  r)  +  rP(n  −  1,  r  −  1)  =  +  (n  −  r  −  1)!  (n  −  r)!

(n  −  1)!(n  −  r  +  r)  (n  
=
−  r)!
¡norte!

= =  P(n,  r)  
(n  −  r)!
Machine Translated by Google

960 Soluciones  a  ejercicios  impares

47.  (n  +  1)!  ­  norte!  =  (n  +  1)n!  ­  norte!

=  (n  +  1  −  1)n!  =  n(n!)

49. Permutaciones  de  algoritmos  (n,r,respuesta)
(*  Este  algoritmo  calcula  P(n,r)  donde  0  ≤  r  ≤  n  y  el  resultado  se  devuelve  en  la  
respuesta  variable.  El  factorial  recursivo
subalgoritmo  es  invocado  por  el  algoritmo.*)
Inicio  (*  permutaciones  *)  
respuesta  ←  factorial(n)/factorial  (n  ­  r)
Fin  (*permutaciones*)

51.  n!  =  n(n  −  1)...  2.  1  =  2n[(n  −  
1)...  3]  >  2n  si  n  ≥  4     
(¡n!)!  >  (2n)!,  si  n  ≥  4

Ejercicios  6.3  (pág.  364)

1.  44 3.  1,  854  5.  1 7.  265 9.  dn  =  =  (−1)n

11.  prueba:  Dn  =  (n−1)(Dn−1+Dn−2)  Supongamos  que  n  es  impar.  Entonces  n−1  es  par.
  (n  −  1)(Dn−1  +  Dn−2)  =  Dn  es  par.

13.  2 15.  8

17.  b0  =  0  =  b1  bn  =   19.  14
bn−1  +  bn−2  +  2,  norte  ≥  2
23.  4
21.  66
27.  c0  =  0  cn  
25.  8
=  cn−1  +  2,  norte  ≥  1
29.  10
31.  16
33.  cn  =  2n  =  O(n)
35.  an  =  (n  −  3)(n  −  2)
37.  0.368879 =  O(n2)

39.  0.367879

Dn   Dn−1  
41.  pn  −  pn−1  = −
¡norte! (n  −  1)!
Dn  −  nDn−1
=

¡norte!  
= (−1)n ,  por  la  ecuación  (4)
¡norte!

43.  Dn  =  (n  −  1)(Dn−1  +  Dn−2)
Dn  (n  ­  1)
= (Dn−1  +  Dn−2)
¡norte!

¡norte!  
pn−1  +  pn  = norte  −  1   1pn  −  
norte 2n
Machine Translated by Google

Capítulo  6 combinatoria 961

45.  Note  que  p0  =  1  y  p1  =  0.  Sea  gn  =  pn  −  pn−1.  Entonces  g1  =  −1.  Por
Ejercicio  43,  
1
−1 −1 −
gn  = gn−1  = gn−2
norte norte norte  ­  1
..
.
1
= −1 − ... −1
g1
norte norte  ­  1 2
(−1)  norte
=
¡norte!

47.  Trastornos  del  algoritmo  (n)
(*  Este  algoritmo  calcula  el  número  de  desarreglos  de  n  elementos  utilizando  la  
definición  recursiva  alternativa.  *)
Inicio  (*trastornos*)
si  n  =  0  entonces
trastornos  ←  1  más  
si  n  es  impar  entonces
trastornos  ←  n  *  trastornos(n­1)  ­  1  más

trastornos  ←  n  *  trastornos  (n­1)  +  1
Fin  (*trastornos*)

Ejercicios  6.4  (pág.  372)

1.  36 3.  84 5.  16,170 7.  45

9.  fn  =  fn−1  +  norte   11.  si
=  fn−2  +  (n  −  1)  +  norte 13.  78
..
.
15.  Nº  de  regalos  enviados  el  día  n  =
=  f0  +  (1  +  2  + ...  +  norte)  =   norte

1  +  norte  (n  +  1)/2,  norte  ≥  0 yo  =  n(n  +  1)/2  =  C(n  +  1,  2)
yo=1
17.  cualquier  entero  no  negativo.

19.  8 21.  C(n,  r) 23.  243 25.  260 27.  957

29.  42 31.  40 33.  105 35.  5


norte (n  ­  1)!  
37.  RHS  = ∙ 39.  RHS  =  C(n  −  1,  r)  +  C(n  −  1,  r  −  1)  (n  −  1)!  
r (r  −  1)!(n  −  r)!  n(n  
(n  ­  1)!  (r  −  1)!(n  −  r)!
−  1)!  [r(r  −  1)!] = +
= r!(n  ­  r  ­  1)!  (n  
(n  −  r)!  ¡norte!
−  1)!(n  −  r  +  r)  r!(n  −  
=
= =  LHS  r! r)!  ¡norte!
(n  −  r)!
= =  LHS
r!(n  −  r)!
Machine Translated by Google

962 Soluciones  a  ejercicios  impares

41.  Sea  P(n) :  gn  =  C(n,  0)  +  C(n,  2)+ 43.  2
C(n,  4).  Entonces  C(0,  0)  +  C(0,  2)  +  C(0,  4)  =  
1  +  0  +  0  =  1  =  g0.     P(0)  es  verdadera.

Supongamos  que  P(k)  
es  verdadera:  gk  =  C(k,  0)  +  C(k,  2)  
+  C(k,  4)  gk+1  =  gk  +  C(k,  1)  +  C(k,  3)
=  C(k,  0)  +  C(k,  2)  +  C(k,  4)  +  C(k,  1)  +  C(k,  3)
=  C(k,  0)  +  [C(k,  1)  +  C(k,  2)]+[C(k,  3)  +  C(k,  4)]
=  C(k  +  1,  0)  +  C(k  +  1,  2)  +  C(k  +  1,  4)
  P(k)  →  P(k  +  1).  Así  el  resultado  sigue  por  PMI.

0 si  r  =  0
45.  A(n,  r)  = 0 si  r  =  norte

A(n  −  1,  r)  +  A(n  −  1,  r  −  1)  +  1  si  0  <r<n

(2n  −  2)!
47.  RHS  =  C(2n,  n)  −  C(2n,  n  −  1)  (2n)!   49.  Cn−1  = ,  por  definición  (n  −  1)!n!  2(2n  −  
1)
(2n)!
= −
n!  n!  (n  −  1)!(n  +  1)!  (2n)!(n   RHS  =  Cn−1  (n  +  1)  2(2n  
−  1)  (2n  −  2)!  
+  1  −  n)  n!(n  +  1)!  (2n)!
= (n  +  1)  (n  −  1∙ )!n!  
(2n)(2n  −  
=
1)(2n  −  2)!  n!(n  
(2n)!
+  1)!  
= =  Cn  =  LHS  n!
(n  +  1)! =

6
51.  RHS  =  4Cn−1  −  Cn−1  n  +  1  2(2n   = =  C  =  LHS  n!
−  1) (n  +  1)!
= Cn−1   53.  42
n  +  1  =  Cn,  
por  el  Ejercicio  49
=  LHS

55.  Combinaciones  de  algoritmos  (n,r,  respuesta)

(*  Este  algoritmo  calcula  C(n,r)  usando  recursividad  *)
Empezar  (*combinaciones*)  si  r  =  
0  entonces
combinaciones  ←  1
sino  ifr=n  entonces
combinaciones  ←  1
else  
combinaciones  ←  combinaciones(n­1,r)  +  combinaciones  (n­1,r­1)
Fin  (*combinaciones*)

57.  3 59.  6 61.  2n  −  2


Machine Translated by Google

Capítulo  6 combinatoria 963

Ejercicios  6.5  (pág.  384)

1.  5040 3.  28 5.  93 7.  1 9.  8

11.  60 13.  10 15.  84 17 {un,  un,  un,  un}  19.  495

21.  8008 23.  120 25.  1365 27.  495 29.  3

31.  4 33.  (3,1,6),  (3,2,5),  (4,1,5) 35.  18

Ejercicios  6.6  (pág.  396)

1.  56 3.  112

5.  x4  +  4x3y  +  6x2y2  +  4xy3  +  y4  7.  32x5  

−  80x4  +  80x3  −  40x2  +  10x  −  1

9.  6 11.  17,920 13.  10 15.  35

17.  n  n/2 19.  Fn 21.  1 23.  8

25.  5 27.  52

29.  42n  +  10n  −  1  =  (5  −  1)2n  +  10n  −  1
2n 2n
= 52n−r(−1)r  +  10n  −  1
r=0
r
2n  −2 2n
= 52n−r(−1)r  −  10n  +  1  +  10n  −  1
r=0
r
2n  −2 2n
= 52n−r(−1)r
r=0
r

2n   2n  
=  52n  − 52n−1  +∙∙∙+ 52
1 2n  −  2

que  es  claramente  divisible  por  25.
norte

=
norte

31.  prueba:  (1  +  x)  n xr 2n  33.  prueba:  (1  +  x)  =  (x  +  1)(1  +  x)  n  El  
r=0
r
coeficiente  de  xn+1  en  la
Sea  x  =  2.  Entonces 2n  
norte
norte
LHS  es y  eso  en  el
3n  = 2r n  +  1
r=0
r norte
norte norte

RHS  es 2r
r=1
yo  ­  1 yo  _  Por  lo  tanto

el  resultado.
norte
norte

35.  Sea  S  = (a  +  kd) (1)


k=0
k
Machine Translated by Google

964 Soluciones  a  ejercicios  impares

Invertir  la  suma  en  el  lado  derecho:
norte norte

=
norte norte

S  = [a  +  (n  ­  k)  d] [a  +  (n  ­  k)  d] (2)


k=0 norte  ­  k k=0 k

Sumando  las  ecuaciones  (1)  y  (2),
norte norte

norte norte

2S  = (2a  +  sd) =  (2a  +  nd)


k=0 k k=0 k
=  (2a  +  nd)  ∙  2n     
S  =  (2a  +  nd)2n−1

37.  prueba:  por  el  ejercicio  36,
norte norte norte norte n  +  1 .   norte

< <  ∙∙∙  < < , donde  r  < es


0 1 r  −  1   r 2 r
máximo  cuando  r  es  el  entero  más  grande  <  (n  +  1)/2.  caso  1  
Sea  n  impar,  digamos,  n  =  2k  +  1.  Entonces  (n  +  1)/2  =  k  +  1.  Entonces  r  debe  ser  k  
=  (n  −  1)/2  =  n/2.  caso  2  Sea  n  par,  digamos,  n  =  2k.  Entonces  (n+1)/2  =  (2k+1)/2  =  
k+1/2.
Entonces  r  debe  ser  k  =  n/2  =  n/2.
norte

Así,  por  los  casos  (1)  y  (2), es  máximo  cuando  r  =  n/2.
r

39.  prueba  (por  PMI):  Sea  P(n)  el  enunciado  dado.  Cuando  n  =  1,  LHS  =  n  =  RHS.     
P(1)  es  verdadera.  Suponga  que  P(k)  es  verdadera,  donde
k
ki
k  ≥  1: i =  k∙2k−1
yo=1

k  +1   k  +1  
ki ki k  
yo = yo +
yo=1 yo=1 ­  1

k  +1   k  +1  
ki k  
= yo + yo
r=1 r=1 ­  1
k k  +1   k  +  1  
ki k   k   k  
= +(k+1) (i−1)  +  i=1 +  yo  =  1
yo  yo=1
+  1 ­  1 ­  1
k k k
ki ki ki
= i +   i +  
yo=1 yo=1 yo=0

=  k  ∙  2k−1  +  k  ∙  2k−1  +  2k  =  

(k  +  1)2k.

  P(k  +  1)  es  verdadero.  Así,  por  inducción,  P(n)  es  verdadera   n  ≥  1.
norte

n−1   = norte

rxr−1.  Sea  x  =  1.  Entonces  n  ∙  2n−1  =
41.  prueba:  n(1  +  x) r
r=1
norte

norte

r .
r=1 r
Machine Translated by Google

Capítulo  6 combinatoria 965

norte
norte

43.  prueba  (por  inducción):  Sea  P(n):  (x+y)  n = r xn−ryr.  Cuando  n  =  0,
r=0
LHS  =  1  =  RHS.  Entonces  P(0)  es  verdadero.  Supongamos  que  P(k)  es  cierto  para  algún  k  ≥  0:
k k
(x  +  y) k = r xk−ryr (1)
r=0

Entonces  (x  +  y)k+1 =  (x  +  y)  k(x  +  y)
k k
= xk−ryr  (x  +  y)
r
r=0
k k k k
= r xk+1−ryr  + r xk−ryr+1
r=0 r=0
k   k k k  ­1 k
= xk+1  + xk+1−r  + xk−ryr+1
0 r r
r=1 r=0
k
+ yk+1

k  +  1   k k k
= xk+1+
0 r xk+1−ryr+ kr  −  1 xk+1−ryr
r=1 r=1
k  +  1  
+
k  +  1 yk+1
k  +  1   k k
= xk+1  + +
0 r kr  −  1 xk+1−ryr
1
k  +  1  
+
k  +  1 yk+1
k  +  1   k k  +  1 k  +  1  
= xk+1  +
0 r xk+1−ryr  + k  +  1 yk+1
r=1
k k  +  1
= r xk+1−ryr
r=0
  P(k  +  1)  es  verdadero.  Por  lo  tanto,  por  PMI,  P(n)  es  verdadera  
norte  ­  r ¡norte! n  ≥  0.  
=
norte

45.  prueba:  RHS  = ∙
r metro  ­  r r!(n  −  r)! (n  −  r)!  (m  −  r)!(n  −  m)!
¡norte! ¡metro! metro
= =
norte

∙ =  LHS
m!(n  −  m)! r!(m  −  r)! metro r

metro norte
metro norte

47.  prueba:  (1  +  x)  m+n  =  (1  +  x)  m(x  +  1)n  = r xr
r xn−r
r=0 r=0
metro  +  norte
Coeficiente  de  xr  en  LHS  =
r
r metro norte

Coeficiente  de  xr  en  la  RHS  =
i r  ­  yo
yo=0
De  ahí  el  resultado.
Machine Translated by Google

966 Soluciones  a  ejercicios  impares

49.  prueba:  Sea  P(n)  el  enunciado  dado,  donde  n  ≥  2.  Cuando  n  =  2,
2   3   k
IZQ  = =  1  = =  lado  derecho.     P(2)  es  verdadera.  Suponga  que   =
2 3 yo  2
P(k) :  i=2
k  +  1  
donde  k  ≥  2.  3 ,

k  +1 k k  +  1   k  +  1   k  +  1  


Después = + = +
yo  2 yo  2 2 3 2
yo=2 yo=2

k  +  2  
=
3

  P(k  +  1)  es  verdadero.  Así,  por  PMI,  el  resultado  tiene   n  ≥  2.

51.  prueba:  Sea  P(n)  el  enunciado  dado,  donde  n  ≥  3.  Cuando  n  =  3,
3   4  
IZQ  = =  1  = =  lado  derecho.     P(3)  es  verdadera.  Supongamos  P(k) :
3 4
k k  +  1  
= donde  k  ≥  3.  4 ,
yo  3
yo=3

k  +1 k k  +  1  
Después = +
yo  3 yo  3 3
yo=3 yo=3

k  +  2  
=
4

  P(k  +  1)  es  verdadero.  Por  lo  tanto,  por  PMI,  la  fórmula  tiene   n  ≥  3.

Ejercicios  6.7  (pág.  408)

1.  74 3.  533 5.  266 7.  21 9.  30

11.  Sea  S  el  conjunto  de  soluciones  con  x  ≤  3,  y  ≤  4  y  z  ≤  4.  Entonces  |  S  |=  C(3  +  11  −  1,  11)  
=  78.  Sea  p1 :  x  >  3,  p2 :  y  >  4  y  p3 :  z  >  5.
Después:

N(p1)  =  no.  de  soluciones  con  x  ≥  4  =  C(3  +  7  −  1,  7)  =  36  N(p2)  =  no.  
de  soluciones  con  y  ≥  5  =  C(3  +  6  −  1,  6)  =  28  N(p3)  =  no.  de  soluciones  
con  z  ≥  6  =  C(3  +  5  −  1,  5)  =  21  N(p1p2)  =  no.  de  soluciones  con  x  ≥  4  y  
y  ≥  5  =  C(3  +  2  −  1,  2)  =  6  N(p1p3)  =  no.  de  soluciones  con  x  ≥  4  y  z  ≥  6  =  C(3  +  1  −  1,  
1)  =  3  N(p2p3)  =  no.  de  soluciones  con  y  ≥  5  y  z  ≥  6  =  C(3  +  0  −  1,  0)  =  1  N(p1p2p3)  =  
no.  de  soluciones  con  x  ≥  4,  y  ≥  5  y  z  ≥  6  =  C(3  −  4  −  1,  −4)  =  0

  respuesta  =  N(p  1p  2p  3)  =  78  −  (36  +  28  +  21)  +  (6  +  1  +  3)  −  0  =  3  13.  Cuando  
n  =  5,  LHS  =  600  =  5(5!)  =  RHS ;  cuando  n  =  6,  LHS  =  3600  =  5(6!)  =  RHS.
Machine Translated by Google

Capítulo  6 combinatoria 967

Ejercicios  6.8  (pág.  416)

1 2 1 1 9
1. 3. 5. 7. 9.
13 13 36 12 1000000
1890 98901 1 13 2
11 13 15. 17 19
1000000 1000000 17 102 429
2 39 9
21.  0 23 25.  0 27 29
5 95 19

Ejercicios  6.9  (pág.  425)

2 5 1 1 7 1
1. 3. 5. 7. 9.  0 11 13
11 13 4 4 15 2

1 1 1 15 20
15. 17 19.  0  21. 23.  0  25. 27
2 dieciséis 17 91 273

125 5 425 256 624


29.  −$1,75  31.  −92,11¢  33. 35. 37. 39. 41.
324 324 432 625 625

norte norte

43.  Mi  = kC(n,  k)pkqn−k  = kC(n,  k)pkqn−k


k=0 k=1
norte

n−1
Por  el  teorema  del  binomio,  n(1  +  x) = kC(n,  k)xk−1.  Sea  x  =  p/q.
k=1
Después

n−1  
n(p  +  q)  qn−1 = pk−1   pk−1  
norte norte

kC(n,  k)  qk−1 = kC(n,  k)  qk−1
k=1 k=1

Dado  que  p  +  q  =  1  en  un  ensayo  de  Bernoulli,  esto  produce
norte

np  = kC(n,  k)pkqn−k  =  E
k=1

Ejercicios  de  repaso  (pág.  429)

1.  1,038 3.  1,454 5.  1.572.120.576  7.  436.700.160

9.  1.542.636.576  11.  0 13.  80 15.  1,296

17.  2.401 19.  103,680 21.  39.916.800  23.  59.049

25.  176 27.  2.598.960  29.  3.744 31.  34,650

33.  20
Machine Translated by Google

968 Soluciones  a  ejercicios  impares

35.  h(n)  =  h(n  −  1)+  número  de  apretones  de  manos  realizados  por  la  n­ésima  
pareja  =  h(n  −  1)  +  4n,  donde  h(0)  =  0.

37.  84 39.  26 41.  165 43.  −330

45.  x6  +  6x5y  +  15x4y2  +  20x3y3  +  15x2y4  +  6xy5  +  y6  47.  

243x5  −  405x4y  +  270x3y2  −  90x2y3  +  15xy4  −  y5

49.  demostración  (por  inducción  fuerte):  Sea  P(n)  el  enunciado  dado.
Cuando  n  =  0,  r  =  0.  Entonces  C(n,  r)  =  C(0,  0)  =  1  es  un  número  entero.  Entonces  
P(0)  es  verdadero.
Suponga  que  P(i)  es  verdadera   i  ≤  k,  donde  k  ≥  0.  Por  el  teorema  6.12,  C(k+1,  r)  
=  C(k,  r  −  1)  +  C(k,  r).  Ambos  números  en  la  RHS  son  enteros,  por  la  hipótesis  
inductiva.  Entonces  su  suma  también  es  un  número  entero.     P(k  +  1)  es  verdadero.  
Así,  por  PMI,  el  resultado  tiene   n  ≥  0.
norte

51.  Nº  de  regalos  enviados  el  día  n  = yo  =  n(n  +  1)/2  =  C(n  +  1,  2)
yo=1

53.  prueba:  Considere  una  permutación  de  un  conjunto  de  n  elementos.  Supongamos  
exactamente  k  de  ellos  ocupan  sus  posiciones  naturales.  Ninguno  de  los  n  −  k  
elementos  restantes  ocupa  sus  posiciones  naturales;  el  número  de  dichos  arreglos  
es  Dn−k.  Pero  los  k  elementos  originales  se  pueden  elegir  de  C(n,  k)  formas  
diferentes.  Por  lo  tanto,  el  número  de  permutaciones  con  exactamente  k  elementos  
en  sus  posiciones  naturales  es  C(n,  k)Dn−k.

55.  2n 57.  A(n,  1)  =  norte  −  1

59.  prueba  (por  PMI):  Sea  A(n,  r)  =  C(n,  r)−1.  Cuando  n  =  1,  RHS  =  C(n,  1)−  1  =  n  −  1  =  
A(n,  1)  =  LHS.  Entonces  P(1)  es  verdadero.  Suponga  que  P(k)  es  verdadera:  A(k,  r)  
=  C(k,  r)  −  1.  Entonces

A(k+1,  r)  =  A(k,  r−1)+A(k,  r)+1  =  [C(k,  r−1)−1]+[C(k,  r)−1]  +1
=  [C(k,  r  −  1)  +  C(k,  r)]  −  1  =  C(k  +  1,  r)  −  1  =  A(k  +  1,  r)

  P(k)  →  P(k  +  1).  Así  el  resultado  sigue  por  PMI.

61.  $375,000 63.  0.40951

Ejercicios  complementarios  (pág.  432)

1.  Sea  k  el  menor  de  los  r  enteros  positivos  consecutivos.  Su  producto  k(k  +  1)...(k  +  r  −  
1)  es  k(k  +  1)...(k  +  r  −  1).  Dado  que  =  C(k  +  r  
−  1,  r)  es  un  número  entero,  k(k  +  1)...(k  +  r  −  1)  es  divisible  por  r!.
r!

3.  Cn  =  C(2n,  n)/(n  +  1)  es  un  número  entero;  entonces  n  +  1  |  C(2n,  n).
C(2n  +  1,  n) (2n  +  1)!   (2n)!
5.  RHS  =   = = =  Cn  =  LHS  n!
2n  +  1 n!(n  +  1)!(2n  +  1) (n  +  1)!
Machine Translated by Google

Capítulo  6 combinatoria 969

7 ,  13;  80

9.  Usaremos  las  siguientes  identidades:

norte   norte
norte
norte
norte
norte

1.  norte =  k 2. =  2n 3. =  n2n−1


­  1  k  ­  1 k k kk  =1 k
k=0
norte
norte
norte
norte
norte
norte   norte
norte  
k2  = k  ∙  k  = norte k  =  norte   (k  −  1  +  1)
k k ­  1  k  ­  1 ­  1  k  ­  1
k=0 k=0 k=0 k=0
norte
norte   norte
norte  
=  norte (k  −  1)  +  norte
­  1  k  ­  1 ­  1  k  ­  1
k=0 k=0
norte
norte  ­   n−1 norte  
=  norte (n  −  1)   +  
2  k  ­  2 ­  1  k  ­  1
k=0 k=1
n−2   norte  ­   n−1   norte  
=  n(n  −  1)  k=2 +  nk  =1
2  k  ­  2 ­  1  k  ­  1

n−2 norte  ­   n−1 norte  ­  


=  n(n  −  1) +  norte
2k 1  k
k=0 k=0

=  n(n  −  1)  ∙  2n−2  +  norte  ∙  2n−1  =  n(n  +  1)2n−2

n−1 d n−  d−1 n−1 d


j  +  kj j  +  norte  ­  dj  
11 =
d  =  1  j  =  0 k=0 d  =  1  j  =  0
+  1

n−1 norte  +  
= −  1
1  re  +  1
re=1
n−1 norte  +  1 norte  +   norte  +  
= − −
r 1  0 1  1
r=0

norte  +  

−  (n  −  1)
1  norte  +  1

=  2n+1  −  1  −  (n  +  1)  −  1  −  (n  −  1)  =  2n+1  −  
2n  −  2

13.  no,  no

norte
si  k  =  1  si  

15.  f(n,  k)  = 0 k  =  n

  f(n  −  2,  k  −  1)  +  f(n  −  1,  k)  si  1  <k<n

17.  Por  el  ejercicio  16,  f(n,  k)  =  C(n  −  k  +  1,  k).     Número  
total  de  subconjuntos  que  no  contienen  enteros  consecutivos  está  dada  por

norte norte norte

f(n,  k)  =  k=0 C(n  ­  k  +  1,  k)  =  k=0 C((n  +  2)  −  k  −  1,  k)  =  Fn+2


k=0

19.  Fn+2
Machine Translated by Google

970 Soluciones  a  ejercicios  impares

Capítulo  7 Relaciones

Ejercicios  7.1  (pág.  441)

1.  1  1  0  0 3.  1  1  0  0 5.  1  1  1  0

7.  1  0  0  0 9. 001 11 001


000 000
110    110   
101 15. 010 17 111
13.   001 000 000
110
      011 111
                       
19.  n2 21.  (2n  −  1)n2 23.  (2p  −  1)mn
25.  0  0  1  1 29.  1  0  1  1
27.  0  0  0  1

31.  1  1  0  0 35.  O
33.  Un

Nota:  en  los  ejercicios  37  y  48,  sea  A  =  (aij),  B  =  (bij)  y  C  =  (cij).
37.  A     A  =  (aij)     (aij)   39.  A     B  =  (aij)     (bij)  
=  (aij     aij)   =  (aij     bij)  
=  (aij)  =  A =  (bij     aij)  
=  (bij)     (aij)  =  B     A

41.  LHS  =  (aij)     [(bij)     (cij)]  =   43.  LHS  =  (aij)     [(bij)     (cij)]  =  


(aij)     (bij     cij)  =   (aij)     (bij     cij)  =  
(aij     (bij     cij))  =   (aij     (bij     cij))  =  
((aij     bij)     cij)  =   ((aij     bij)     (aij     cij) )  
[( aij)     (bij)]     (cij) =  ((aij)     (bij))     (aij     cij)
=  lado  derecho =  lado  derecho

45. Encuentro  del  algoritmo  (A,B,C)
(*  Sean  A  =  (aij)n×n  y  B  =  
(aij)n×n.  C  es  el  encuentro  de  A  y  B.)
Comenzar  (*cumplir*)  
para  i  =  1  a  n  hacer
para  j  =  1  an  do  cij  ←  aij  
  bij
Terminar  (*conocer*)
Machine Translated by Google

Capítulo  7 Relaciones 971

47. Algoritmo  producto  booleano  (A,B,C)
(*  C  es  el  producto  booleano  de  A  =  (aij)m×p  y  B  =  (bij)p×n  *)
Empezar  (*  producto  booleano  *)  para  
i  =  1  a  m  hacer
para  j  =  1  to  p  do  cij  ←  
(ai1     bij)     (ai2     b2j)∙∙∙   (aip     bpj)
Fin  (*  producto  booleano  *)

Ejercicios  7.2  (pág.  448)

1.  {(1,2),(1,4),(1,8),(3,4),(3,8),(5,8)}  3.  {(1,4),(3  ,2)}
5.  {(1,2)} 7.  Un;  B 9.  {1,3};  {2,4} 11.  {1};  {2}
111 010 100
13.     011 15.     100 17.     000
001    000    000   

19.  2 3 21.  2 3 23.  si 25.  no

27.  si 29.  si

5 31.  si 33.  si
8 5

35.  {(a,  b),  (b,  c),  (c,  b)} 37.  abc
010
39.  0000,  0001,  0011,  0010,  0110,  0111,  
001
0101,  0100,  1100,  1101,  1111,  1110,  
abdominales

C 010
1010,  1011,  1001,  1000      

41.  bbb,  bca,  bcd 43.  12 45.  no 49.  no 51.  2m2


53. Relación  de  impresión  del  algoritmo  (R,A)
Comenzar  (*  imprimir  *)
para  i  =  1  a  n  hacer
para  j  =  1  a  n  hacer  si  aij  
=  1  entonces  escribir  
((i,j))
Fin  (*  imprimir  *)

Ejercicios  7.3  (pág.  453)

1. Cabeza  de  cola  siguiente
3.
1 2 3
2   1   5  
1   3   1   1   2 1 3
2  inicio   1   3   0  
3   1   2   6   3 1 2
4   3   2   3  
5  6 2 3 2
Machine Translated by Google

972 Soluciones  a  ejercicios  impares

5.  123 7. 1 2 9.
010 1 2
1  2 010
3 111 2 1 3
      3

3 2 3

11 Lista  de  adyacencia  de  algoritmos  (R,list)
(*  Lista  es  la  matriz  de  punteros  que  apuntan  a  cada  lista  enlazada
en  la  representación  de  la  lista  de  adyacencia.  *)
Empezar  (*algoritmo*)  lista  
de  inicialización
mientras  que  hay  más  elementos  en  R  do
comenzar  (*mientras*)
leer  un  par  (i,j)  crear  un  
nodo  info(nodo)  ←  j  
enlace(nodo)  ←  nil  
insertar  el  nodo

mientras  tanto

Fin  (*algoritmo*)

13 Matriz  de  adyacencia  del  algoritmo  (A,list)
(*  Lista  es  la  matriz  de  punteros.  *)
Empezar  (*  algoritmo  *)  (*  
inicializar  A  con  ceros  *)  para  i  =  1  a  n  
hacer
for  j  =  1  to  n  do  aij  ←  0  (*  
atravesar  la  lista  de  
adyacencia  *)  for  i  =  1  to  n  do  begin  (*  for  *)  (*  
atravesar  cada  lista  enlazada  *)  k  ←  link(i)  
while  k  =  nil  do  comenzar  (*  while  *)  j  ←  
info(nodo(k))  aij  ←  1  k  ←  enlace(nodo(j))  
endwhile  endfor

Fin  (*algoritmo*)

Ejercicios  7.4  (pág.  459)

1.  simétrico,  antisimétrico,  transitivo
3.  simétrico,  antisimétrico,  transitivo
Machine Translated by Google

Capítulo  7 Relaciones 973

5.  si 7.  no 9.  si


11.  no 13.  no 15.  no

17.  reflexivo,  simétrico,  antisimétrico,  transitivo
19.  reflexivo,  antisimétrico
21.  R  no  es  simétrico  si  existe  un  a     A  tal  que  aRb,  pero  bRa.

23.  {(a,  a),  (b,  b),  (c,  c)} 25.  {(a,  a),  (a,  b),  (b,  b),  (c,  c)}
27.  {(a,  a),  (a,  b),  (b,  b),  (b,  c),  (c,  c)} 29.  {(a,  b)}
31.  {(a,  b),  (b,  a)} 33.  {(un,  un)}
35. 110 37. 001
011 110
101
      010
     

39.  Si  R  contiene  solo  elementos  de  la  forma  (a,  a).
41.  no 43.  no 45.  si 47.  mii  =  0   i.
49.  si 51.  si 53.  no 55.  mij  =  1  →  mji  =  0   i,  j.
57.  4 59.  2 61.  4

63.  Sean  MR  =  (xij)  y  M[2]  =  R( yij).  Supongamos  que  R  no  es  transitiva.  
Entonces  debe  haber  elementos  ai,  aj,  ak     A  tales  que  aiRaj  y  ajRak,  
pero  aiRak;  es  decir,  xij  =  1  =  xjk,  pero  xik  =  0.  Como  xij  =  1  =  xjk  y  yik  =  
...
(xi1     x1k)        (xin       (xxij  
nk),    yxik  =  ...
jk)   1.  E
contradice  
ntonces  yik  
la  >s  
uposición  
xik,  lo  que  
de  que  
M[2]  ≤  MR.     R  es  transitiva. R

Ejercicios  7.5  (pág.  469)

1.  {(a,  a),  (a,  b),  (a,  c),  (b,  b),  (b,  c),  (c,  a)};{(a,  b),  (b,  b)}
3.  ≤;  Ø  
5.  {(a,  a),  (a,  b),  (b,  b),  (b,  c),  (b,  d),  (c,  a),  (d,  a)};{(a ,  b),  (b,  c)}
7.  {(a,  a),  (a,  b),  (b,  c),  (c,  a),  (c,  b)}
9.  {(a,  a),  (a,  b),  (a,  c),  (b,  c),  (c,  c)}
11.  Ø

13 111 15. 111


000 010
101
      011
     
Machine Translated by Google

974 Soluciones  a  ejercicios  impares

2  17.  R =  comunicarse  a  través  de  un   19.  { ( un ,  un ),  ( segundo ,  un ),  ( c ,  b ),  ( un , c ) }


intermediario;  R  n  s=e  comunican   21
a  través  de  n  −  1  intermediarios. 110
110  
  001  
23
25
100
010
011  
  100
010  
  101
27.  {(1,  a),  (2,  b),  (3,  b ) } 29.  R

31.  { ( a,  1),  ( a,  2),  ( a,  3),  ( b,  1),  ( b,   33.  { ( un,  3) }

3) }  35.  {(1,  a),  (1,  b),  (2 ,  a),  (2,  b),  (3,  b ) } 37.  {(2,  b ) }

39.  Sea  MR  =  ( aij ) y norte  ×  norte
Entonces  41.  S,  
ea  
y )  
( xx )  
  ( R  −  R
  1   )  
−  1
−  (1 y ,  
.   y  por  

( MR )  =  ( bij )n  ×  n. Entonces  aij  =   1 )  −lo  


  1  tanto  
  R .( x ,  y )     R .     ( R  −  
1  si  y  1.
si  ( yo ,  j )     R ;  es  decir,  si  bij  =  
Entonces  aij  =  1  iff  bij  =  1.   Por  el  contrario,  sea  ( x ,  y )     R.  Entonces  
Así  MR  =  ( MR ) . x )  
−  1 )     R  −  R
−  1 .     1    (1 )  
 y ,  
­y  1  ­p  1
  ( R   or  
=   Rt­anto ,  
 )    .  1  ( X  yA )     ( R  
sí  ( R   ­  
.
43.  Suponga  que  R     S.  Sea ,  
( xx )  
y )     R .  
  −  1.  
  (E y ,  
ntonces  
x )     (S y ,  
  y  por  lo  
tanto,  yS  
 )  −  1  S
  ( x ,  
  −  1y.   )  
Entonces  
  ( R  ∩  R
S   )  
−  −1    1.     45.  Sea  ( x ,  y )     ( R  ∩  S ) .  Entonces  
Entonces  
Entonces  
( y ,  
47.  
x )  
Sea  
  R( x  
  y  (( y ,  
 y ,  xx )  
 )  S.      R ∩y    .
S  ,   ( x ,  
y )  
y )    ( R  
( x ,     R∩
  o  S ).  Entonces  
  ( x ,  
y )     
decir,  
S,  es  ( x ,  
y )     R    
( R   oS  ( )  
 x ,  
y  pyor  
 )  lo    tSanto  
 .  A( R  
sí  
( x ,  
( R  ∩  Sy ).
 )     
  S  −  1  ( x  y  ( x ,  
por  ylo  
 )  tanto ,  
  ( R  y− )  
  1  ∩  R
S  −  1 ,  
). y )  

Del  mismo  modo,  ( R     S )     ( R  ∩  S ) .


. De  ahí  el  resultado.
Por  lo  tanto  
( x     ( R  ∩  S )  −  1     ( R  −  1  ∩  S  −  1). 49.  Suponga  que  R  es  simétrico.  Sea  ( x ,  
De  manera   y )     R .  Entonces  
entonces   ( y ,  x )  ( x ,  
  Ry  es  
y )     R,  
a  sqimétrico.  
ue  R  
similar,  (R−1  1 )     ( R  ∩  S )  −  1.   ( y ,  
simétrico.
X )     R .  Por  lo  tanto,  R  es  
∩  S−  De  ahí  el  resultado.

51.  Sea  R  simétrico  y ,  y )     R   Por  el  contrario,  sea  R  simétrica.


−  
  R1 ..  Entonces  ( y ,  ( xx )   Entonces  ( R )  es  simétrico.  
Como  R  es  simétrico, ,  y )     
Pero  ( R )  =  R.     R  es  simétrico.
Entonces,  
R .     R  p−or  
  1  e( x  
l  Ejercicio  
  R.  
( R  
4−3,  
  1 )  −  1     
R  −  1 ;  es  decir,  R     R  −
=  
  1
R .  
 .   R  −  1   53.  Sean  R  y  S  simétricos.  Sea  ( x ,  y )  
  R  ∩y  S
( x ,   .  Entonces  
 )   ( x ,  
  S .     ( y ,   y( y ,  
x )   )     R
x   )  
R  y    y    
Por  el  contrario,  sea  R  −  1  =  R  y ,  y )   S.  Así  (tanto  
 y ,  x )  
R  ∩  R
S  ∩   Ss  imétrico.
es   y  por  lo  
y )  
lo  t  
anto,  
R.  Entonces  
R  es     (( xx )  
 y ,  simétrico.
    
RR    
−−    
11  .   .
( x ,  
Por  

55.  Las  relaciones  R  =  { ( a   , b ) }  y  S  =  { ( b , c ) }  en  { un, b, c }  son  transitivos,  pero


, c ) }  no  lo  es.
R     S  =  { ( a ,  b),  ( b
Machine Translated by Google

Capítulo  7 Relaciones 975

57. Unión  de  algoritmos  (MR S,  MR,MS)


(*  Sean  MR  =  (aij)m×n,  MS  =  (bij)m×n,  y  MR S  =  (cij)m×n.  *)
Empezar  (*algoritmo*)  
para  i  =  1  a  m  do
para  j  =  1  an  do  cij  
←  aij     bij
Fin  (*algoritmo*)

59. Algoritmo  inverso  (MR,MR−1 )
(*  Sea  MR  =  (aij)m×n  y  MR−1  =  (cij)m×n.*)
Empezar  (*algoritmo*)  
para  i  =  1  a  m  do
para  j  =  1  a  n  hacer  si  
aij=1  entonces  cij  
←  0  otra  cosa

←  1
Fin  (*algoritmo*)

61. Composición  del  algoritmo  (X,Y,Z)
(*  X  =  MR,  Y  =  MS  y  Z  =  MRS  *)
Empezar  (*algoritmo*)  
para  i  =  1  a  m  do
para  j  =  1  a  p  do  zij  
←  (xij     yij)   ∙∙∙   (xin     ynj)
Fin  (*algoritmo*)

Ejercicios  7.6  (pág.  475)

1.  {(un,  un)} 3.  {(a,  a),  (b,  b),  (c,  c)} 5.  Ø


7.  {(a,  a),  (b,  b),  (c,  c)} 9.  A  ×  A

11.  {(a,  b),  (a,  c),  (a,  d),  (b,  c),  (d,  b),  (d,  c)}

Ejercicios  7.7  (pág.  481)

1.  {(a,  a),  (a,  b),  (b,  a),  (b,  b)} 3.  {(b,  a),  (b,  b),  (b,  c),  (c,  a),  (c,  b),  (c,  c)}
5.  {(a,  a),  (a,  b)} 7.  {(a,  b),  (a,  c),  (b,  c)}
9.  A  ×  A 11.  {(a,  a),  (a,  b),  (a,  c),  (b,  b),  (c,  b),  (c,  c)}

13 011 15. 010 17 1000 1000


001 011 0101 0101
000
      110
     
  

0101      

0101   

0101 ,   0101

19.  {(a,  b),  (a,  c),  (b,  c)}
Machine Translated by Google

976 Soluciones  a  ejercicios  impares

21.  {(a,  b),  (a,  c),  (b,  a),  (b,  b),  (b,  c),  (c,  b),  (c,  c)}
23.  {(a,  a),  (b,  b),  (b,  d),  (c,  b),  (c,  d),  (d,  b),  (d,  d)}

25.  {(a,  a),  (b,  a),  (b,  b),  (c,  c)} 27.  {(a,  a),  (b,  a),  (b,  b),  (b,  c),  (c,  b),  (c,  c)}
29.  {(a,  a),  (a,  b),  (b,  b),  (c,  c)}   31.  {(a,  a),  (a,  b),  (a,  c),  (b,  b),  (b,  c),  (c,  c)}  
33.  {(a,  a),  (a,  b),  (b,  b),  (b ,  c),  (c,  a),  (c,  c)} 35.  ≥

37.  {(a,  b),  (a,  c),  (b,  a),  (b,  c),  (c,  a),  (c,  b)}
39.  {(a,  a),  (a,  c),  (b,  b),  (c,  a),  (c,  b)}

41.  {(a,  b),  (a,  c),  (b,  a),  (b,  c),  (c,  a),  (c,  b)}
43.  {(a,  a),  (a,  c),  (b,  b),  (b,  c),  (c,  b),  (c,  c)}
45.  Suponga  que  R  es  reflexivo.  Sea  (a,  a)    .  Como  R  es  reflexiva,  (a,  a)     R.  
     R.  Inversamente,  sea     R.  Sea  a     A.  Como  (a,  a)    ,  (a,  a)     R.
Por  lo  tanto,  R  es  reflexivo.

47.  R     es  reflexiva,  por  el  Ejercicio  46.  Sea  S  una  relación  reflexiva  tal  que  R  
  S     R    .  Dado  que   R     S,  
entonces   SR            S
  S.  
  Pero  
 .  CSomo   S    es  
     R reflexivo,  
 .     S  =  R     S;  
.

49.  R     R−1  es  simétrica,  por  el  ejercicio  48.  Sea  S  una  relación  simétrica  tal  
que  R     S     R     R−1.  Como  R     S,  R−1     S−1  por  el  ejercicio  61  de  la  
sección  7.5.  Además,  dado  que  S  es  simétrico,  S−1  =  S.     R−1     S.
Así  R     S  y  R−1     S.     R     S−1     S     S;  es  decir,  R     R−1     S.     
S  =  R     R−1.

Ejercicios  7.8  (pág.  490)

1.  no 3.  si 5.  no

7.  si 9.  no 11.  {a,  b}


13.  {d} 15.  {A,  B,  C,  F,  H} 17.  {J}

19.  {a,  b} 21.  {aaa,  aab,  aba,  abb,  baa,  bab,  bba,  bbb}
23.  {a,  b,  c} 25.  {aa,  ab,  ac,  ba,  bb,  bc,  ca,  cb,  cc}
27.  {{a},{b},{c}} 29.  {{A,  B,  C,  F,  H},  {D,  E},  {G},  {I},  {J}}

31.  {(a,  a),  (b,  b),  (b,  c),  (c,  b),  (c,  c),  (d,  d)}

33.  2 35.  15
37.  2 39.  15
Machine Translated by Google

Capítulo  7 Relaciones 977

41.  R  =  {(a,  a),  (a,  b),  (b,  a),  (b,  b),  (c,  c)}  y  S  =  {(a,  a),  (a,  c) ,  (b,  b),  (c,  a),  (c,  
c)}  son  relaciones  de  equivalencia  sobre  {a,  b,  c},  pero  R     S  =  {(a,  a),  (a,  
b),  ( a,  c),  (b,  a),  (b,  b),  (c,  a),  (c,  c)}  no  lo  es.

43.  lunes 45.  martes

47.  Sean  a  ≡  b  (mod  m)  yc  ≡  d  (mod  m).  Entonces  a  −  b  =  lm  y  c  −  d  =  km  para  
algún  l,  k     Z.  Entonces  (a  −  b)  +  (c  −  d)  =  lm  +  km,  es  decir,  (a  +  c)  −  (b  
+  d )  =  (l  +  k)m.     a  +  c  ≡  (b  +  d)  (mod  m).
49.  Por  el  algoritmo  de  división,  a  =  bm  +r  para  algún  b     Z.     a  ≡  r  (mod  m).
51.  Por  el  algoritmo  de  división,  a  =  d1m  +  r1  yb  =  d2m  +  r2 ,  donde  0  ≤  r1,  r2  
<  m.  Entonces  a  −  b  =  (d1  −  d2)m  +  (r1  −  r2).     a  ≡  b  (mod  m)  si  y  solo  si  
r1  ≡  r2  (mod  m).
norte

53.  Sea  N  = ai10n−i  sea  la  expansión  decimal  de  N.  Dado  que
yo=0

10  ≡  1  (mod  9),  10k  ≡  1  (mod  9)  por  el  ejercicio  48.  Así ,  ai10n−i  ≡  ai i  
por  el  ejercicio  48.
norte

  norte  ≡ ai  (mod  9).  En  consecuencia,  N  es  divisible  por  9  si  y  sólo  si
yo=0
norte

ai  es  divisible  por  9.
yo=0

55.  0 57.  8 59.  4 61.  15,  621

Ejercicios  7.9  (pág.  503)

1.  no 3.  si 5.  si 7.  si


9.  no 11.  no 13.  si 15.  si

17.  si 19.  (2,  6)  (3,  5) 21.  (2,  3,  2),  (2,  3,  3),  (2,  2,  4)

23.  lujo,  maximo,  momento,  descuido,  vecino,  neófito
25.  descuento,  discurso,  discreto,  discreto,  discreción,  disquete

27 18 29 C

6 9 b

a
2 3

1
Machine Translated by Google

978 Soluciones  a  ejercicios  impares

31 aaa  aab aba  abb bebe  bebe bba  bbb

Automóvil  club  británico abdominales licenciado  en  Letras cama  y  desayuno

a b

33.  c,  d;  a 35.  ninguno;  ninguno 37 ,  18;  1 39.  ninguno;  ninguno

41.  18,  1 43.  F. 45.  F. 47.  F.

49.  (Z,  ≤) 51.  (Z,  ≤) 53.  a,  b,  c,  d,  e 55.  a,  c,  b,  d,  e,  f


57.  1,  2,  3,  6,  9,  18 59.  t1,  t2,  t3,  t4,  t5,  t6,  t7,  t9,  t8,  t10,  t11,  t12,  t13

61 ,  26;  C,  G,  B,  E,  A,  F,  D
63.  Supongamos  que  hay  dos  elementos  mayores,  ay  a  en  la  poset.
Como  a  es  un  elemento  mayor,  a  a.  Del  mismo  modo,  aa .  Como  es  
antisimétrica,  a  =  a .
65.  Sea  a1  cualquier  elemento  de  A.  Si  a1  no  es  maximal,  debe  haber  un  
elemento  a2  tal  que  a1     a2.  Si  a2  es  máximo,  entonces  hemos  terminado.  
Si  no  lo  es,  debe  haber  un  elemento  a3  en  A  tal  que  a2     a3.  Si  a3  no  es  
máximo,  continúe  con  este  procedimiento.  Como  A  es  finito,  este  
procedimiento  debe  terminar  con  algún  elemento  an.  Así  a1  ...   aEntonces  
2        an.  
an  es  un  elemento  maximal.  De  ahí  el  resultado.

Ejercicios  de  repaso  (pág.  508)

1.  simétrico 3.  {(1,  3),  (2,  1),  (3,  1),  (3,  3)}
5.  {(1,  2),  (1,  3),  (2,  1),  (2,  3),  (3,  1),  (3,  3)}

7.  {(1,  1),  (2,  1),  (2,  2),  (2,  3),  (3,  2)} 9.  {(1,  1),  (2,  2),  (2,  3)}
11.  {(1,  1),  (1,  2),  (1,  3),  (2,  2),  (2,  3),  (3,  2),  (3,  3)}
13.  {(1,  1),  (1,  2),  (2,  2),  (3,  2)}
15. 001 17 111
101 011
101
      011
     

19.  {(1,  1),  (1,  2),  (1,  3),  (2,  2),  (2,  3),  (3,  2),  (3,  3)}
21 110 23.  A  ×  A 25.  2
010 27.  no 29.  {a,  c}
010
     
Machine Translated by Google

Capítulo  7 Relaciones 979

31.  {a,  c} 33.  {{a,  c},  {b,  d}}

35.  {(2,  2),  (2,  4),  (3,  3),  (4,  2),  (4,  4),  (7,  7)}

37.  877  39.  T 41.  T 43.  T 45.  T 47.  T

49.  λ,  0,  00,  01,  1,  10,  11,  000,  001,  010,   51.  CS  100,  CS  150,  CS  200,  CS  250,  CS  
011,  100,  101,  110,  111 300,  CS  350,  CS  400,  CS  450  55.  Sea  

53.  ninguno;  a (a,  b)     (R  ∩  S)  2.  Luego  hay


existe  un  elemento  c  en  A  tal  que  
57.  Suponga  que  R  es  antisimétrica.  Sean  (a,  
(a,  c)     R  ∩  S  y  (c,  b)     R  ∩  S.     
b)     R  ∩  R−1.  Entonces  aRb  y  aR  −1b.  
(a,  c)     R  y  (c,  b)     R,  entonces  (a,  
Como  aR−1b,  bRa.  Así  aRb  y  bRa.  
b )     R2.
Entonces  a  =  b.  Así  (a,  a)     R  ∩  R−1  
De  manera  similar,  (a,  b)     S2.  
 .
Entonces  (a,  b)     R2  ∩  S2.  Entonces  
Por  el  contrario,  sea  R  ∩  R−1    .
(R  ∩  S) 2   R2  ∩  S2.
Supongamos  aRb  y  bRa.  Como  
bRa,  aR  −1b.  Así  aRb  y  aR  −1b.     
a(R  ∩  R−1)b.  Como  R  ∩  R−1    ,  
(a,  b)    .  Entonces  a  =  b  y  R  es  
antisimétrica.

Ejercicios  complementarios  (pág.  511)

1.  domingo

3.  Recuerda  que = ¡pag!  es  un  número  entero.  Cuando  0  <k<p,  0  <  p−k  <  k!(p  
paquete
−  k)!  pag.  Como  p  es  un  primo,  ninguno  de  los  factores  de  k!  o  
(p  −  k)!  puede  dividir  p.

Entonces  pag
paquete  _

pag
pag
5.  Por  el  teorema  del  binomio,  (a  +  b)  p = ap−rbr  =  ap  +
r
r=0
pag  ­1
pag

r ap−rbr  +  pb  ≡  ap  +  0  +  pb  ≡  ap  +  pb  (mod  p)
r=1
7.  2

9.  Sea  a  un  entero  cualquiera.  Luego,  por  el  algoritmo  de  división,  a  ≡  0,  1  o  2  módulo  3.  Si  
a  ≡  0  (mod  3),  entonces  a(a  +  1)(a  +  2)  ≡  0  (mod  3).  Asimismo,  a(a  +  1)(a  +  2)  ≡  0  (mod  
3)  cuando  a  ≡  1  o  2  módulo  3.  Así,  en  todos  los  casos,  el  producto  es  congruente  con  
0  módulo  3.

11.  5,  13  

13.  Por  el  teorema  de  Euler,  aφ(b)  ≡  1  (mod  b).  Dado  que  b  |  φ(a),  bφ(a)  ≡  1  (mod  b).
Por  lo  tanto,  aφ(b)  +bφ(a)  ≡  1  (mod  b).  Asimismo,  aφ(b)  +bφ(a)  ≡  1  (mod  a).
Machine Translated by Google

980 Soluciones  a  ejercicios  impares

Dado  que  a  y  b  son  primos  relativos,  se  sigue  que  aφ(b)  +  bφ(a)  ≡  1  (mod  ab).

15.  El  conjunto  tiene  relaciones  2n2  (palomas).  Usando  matrices  de  adyacencia,  2(n−1)n/
2+n  =  2n(n+1)/2  de  ellas  son  relaciones  simétricas  (casilleros).
Supongamos  que  cada  relación  se  asigna  a  su  cierre  simétrico.  Entonces,  por  el  
GPHP,  al  menos  uno  de  los  agujeros  debe  contener  (2n2  −1)/2n(n+1)/2+1  =  2n(n−1)/
2  −  1/2n(n+1)/2  +  1  =  2n(n−1)/2  −  1  +  1  =  2n(n−1)/2  palomas;  es  decir,  al  menos  
2n(n−1)/2  relaciones  deben  tener  el  mismo  cierre  simétrico.

Capítulo  8 gráficos

Ejercicios  8.1  (pág.  533)

1.  no 3.  no 5.  3;  4 7.  4;  4;  4

9.  abc 11 a b
a 021
b 201 C d
C 110
     

13.  El  gráfico  está  formado  por  dos  componentes  disjuntos.

15.  Suma  de  los  grados  =  6  +  4  +  4  =  14  =  2  ∙  7  =  2  (nº  de  aristas)

17.  4 19.  si 21.  13 23.  12 25.  15 27.  45

1  si  i  =  j  0  
29.  (aij)n×n  donde  aij  =
en  caso  contrario

31.  si;  V1  =  {a,  c},  V2  =  {b,  d,  e} 33.  min
norte

35.  Por  el  teorema  8.1, vi  =  2e 37. 39.


yo=1
norte

nm  ≤ vi  ≤  nM
yo=1
41.
Es  decir,  nm  ≤  2e  ≤  nM  2e   43.  sí,  n  −  1
≤  M
metro  ≤
norte

no.
45.  Por  el  ejercicio  44,  e  = . 47.  no 49.  •b
2
  nr  es  par. a• •C

51.  Sea  A  la  matriz  de  adyacencia  de  G.   53.  Km ,n consta  de  un  bucle  en  cada  


Entonces  A  es  la  matriz  de  adyacencia   vértice  y  un  camino  que  conecta  los  
de  G. vértices  en  cada  conjunto  de  vértices.
Machine Translated by Google

Capítulo  8 gráficos 981

55. C 57.  K5
59.  K6
a b mi

61.  nudo
d

63.
equipo

abecedario  redondo

1  badcxe  2  ceaxbd  3  dxeacb  4  edxbac  
5  xcbeda

65.  Sea  P(e)  la  proposición  dada.  Cuando  e  =  1,  un  gráfico  que  consta  de  un  
bucle  en  un  vértice  satisface  la  condición.     P(1)  es  verdadera.
norte

Supongamos  que  P(k)  es  verdadero: grado  (vi)  =  2k.  Agregar  un  borde  entre  vértices
yo=1

vs  y  vt,  donde  1  ≤  s<t  ≤  n.  Luego,  tanto  el  grado  (vs)  como  el  grado  (vt)  se  
incrementan  en  1.
norte

   grado  (vi)  =  2k  +  2  =  2(k  +  1).  Entonces  P(k  +  1)  es  verdadero.  Así  el  resultado
i=1

sigue  por  PMI.

Ejercicios  8.2  (pág.  539)

1. 3.
112 1 2

2 1 3 2 3 3

3 2 3 2 2

5. 7.
1 1 2 3 1 2 3

2 1 2 3 2 1 3

3 2 2 3 3 1 2
Machine Translated by Google

982 Soluciones  a  ejercicios  impares

9. 11.  123 13.  123
1 1   110 1 010
2   101 2   102
2 3 010 3 020
           

15.  123 17.  1 2
1   111
2   111
3 111
     
3

19 21
1 2 5 3  13 1 3 1 4 1 5 2

2 1 5 3 8 2 3 3 4 5 5 8

3 1  13 2 8 3 1 1 2 3

4 1  1 2  5

5 1  2 2  8

Ejercicios  8.3  (pág.  544)

1.  no 3.  sí;  f(a)  =  segundo,  f(b)  =  f,  f(c)  =  e,  f(d)  =  gramo

5.  sí;  f(a)  =  f,  f(b)  =  e,  f(c)  =  g,  f(d)  =  h 7.  no 9.  no

11.  sí;  f(a)  =  yo,  f(b)  =  h,  f(c)  =  g,   13.  no
f(d)  =  f,  f(e)  =  j
15.  1)  Todo  grafo  es  isomorfo  a  sí  mismo.     La  relación  es  reflexiva.
−1
2)  Sea  f :  G1  →  G2  un  isomorfismo.  Entonces  f   :  G2  →  G1  también  es
un  isomorfismo.
3)  Sean  isomorfismos  f :  G1  →  G2  y  g :  G2  →  G3 .  Entonces  g  ◦  f :  G1  →  
G3  también  es  un  isomorfismo.     La  relación  es  transitiva.  Por  lo  tanto,  
la  relación  es  una  relación  de  equivalencia.

Ejercicios  8.4  (pág.  554)

1.  2 3.  4 5.  7 7.  1


Machine Translated by Google

Capítulo  8 gráficos 983

9.  badb,  bacb,  bdcb 11.  no 13.  no 15.  si


17.  5 19.  10 21.  45 23.  0
25.  10 27.  0 29.  10 31.  2
33.  0
35.  comienzo  —  ab  —  a  —  aA  —  A  —  AB  —  aA  —  AC  —  A  —  aA  —  C  —  cC
C  —  cC  —  A  —  aA
37.  Inicialmente,  el  granjero  toma  a2,  a4, ... ,  a2n  al  otro  lado  del  río  y  luego  regresa.  
Toma  a3,  a5, ... ,  a2n−1  de  lado  a  lado  
yt  
Ella   trae  a1  
oma   2,  
ya  a4, ... ,  
2n+1   dae  
l  ovtro  
uelta  
al  lyado  
lado   a2n
original.  
  luego  
regresa.  Finalmente,  toma  a2,  a4, ... ,  a2n  al  otro  lado.

39.  Gráfico  de  algoritmo  (G,A,conectado)
(*  Este  algoritmo  determina  si  el  grafo  G  con  n  vértices  es  conexo  usando  los  
Teoremas  8.4  y  8.5,  y  su  matriz  de  adyacencia  A.  La  variable  booleana  conexionado  denota  
si  G  es  conexo  o  no.  *)
Comenzar  (*algoritmo*)
(*  encontrar  S  =  B  ← Ai  
  A
*)  
  
(*  inicializar  *)
S  ←  A  (*  inicializar  *)  para  i  =  
2  a  n  ­  1  hacer
empezar  (*  por  *)
B  ←  AB
S  ←  S+B
endfor  
(*  comprueba  si  alguna  entrada  de  S  es  distinta  de  
cero.  *)  i  ←  1
j  ←  1  
hecho  ←  falso
conectado  ←  verdadero  
mientras  (no  hecho)  y  (i  ≤  n)  y  (j  ≤  n)  sí
si  sij  =  0  entonces  
comenzar  (*  el  gráfico  no  está  conectado  *)
conectado  ←  falso  
hecho  ←  verdadero  fin  
si  más  empezar  (*  más  *)  i  
←  i+1

j  ←  j+1  final

Fin  (*algoritmo*)

Ejercicios  8.5  (pág.  573)

1.  no 3.  no 5.  si


Machine Translated by Google

984 Soluciones  a  ejercicios  impares

7.  a­e1­b­e2­c­e3­a­e4­c­e5­a  9.  a­

e1­b­e2­a­e5­c­e6­a­e7­c­e3­b  ­e4­c­e9­d­e8­a

11.  a­e1­b­e4­d­e7­e­e3­b­e5­c­e6­d­e2­a

13.  no 15.  contiene  un  circuito  euleriano.

17.  contiene  un  circuito  euleriano. 19.  no

21.  ninguno 23.  impar  _

25.  n  =  2 27.  r  incluso

29.  contiene  un  ciclo  hamiltoniano. 31.  contiene  un  ciclo  hamiltoniano.

33.  no  es  un  grafo  conexo. 35.  contiene  un  ciclo  hamiltoniano.

37.  contiene  un  ciclo  hamiltoniano.  41.  sin   39.  no  contiene  ninguno

ciclo  hamiltoniano;  contiene  un  camino  hamiltoniano,  abced.

43.  norte  ≥  3 45.  contiene  un  camino  hamiltoniano.

47.  tampoco 49.  metro  =  norte

51.  Tn  es  euleriano   n  ≥  1. 53.  Sn  no  es  euleriano  si  n  ≥  2.

55. 57.

59.  acbdea,  53 61.  ninguno 63.  ninguno

65.  1­15­12­4­5­11­16­9­7­2­14­13­3­6­10­17­8

67.  Algoritmo  camino  euleriano  (G,A)
(*  G  es  un  grafo  conexo  con  n  vértices  y  con  matriz  de  adyacencia  A  =  (aij)n×n.  Utilizando  
el  teorema  8.8,  este  algoritmo  determina  si  G  contiene  un  camino  euleriano.  Impar  es  un  
contador  que  realiza  un  seguimiento  del  número  de  vértices  impares  En  g.  *)

Begin  (*  algoritmo  *)  impar  
←  0  (*  inicializa  el  contador  *)  bandera  ←  falso  (*  
bandera  booleana  que  existe  cuando  impar  >  2  *)  i  ←  1  (*  inicializa  el  
índice  de  fila  *)  mientras  que  (i  ≤  n)  y  (no  flag )  do  (*  calcular  la  suma  de  
cada  fila  *)  begin  (*  while  *)  sum  ←  0

para  j  =  1  a  n  hacer
suma  ←  suma  + aij
si  la  suma  es  impar  entonces  (*  actualiza  el  contador  *)
impar  ←  impar  +  1  
si  impar  >  2  entonces  (*  salir  del  ciclo  *)  marcar  
←  verdadero  endwhile
Machine Translated by Google

Capítulo  8 gráficos 985

si  impar  =  2,  entonces  
el  gráfico  tiene  un  camino  euleriano,  
de  lo  contrario,  el  gráfico  no  tiene  un  
camino  euleriano
Fin  (*algoritmo*)

Ejercicios  8.6  (pág.  584)

1. 3.
a
a b
mi

d F
c  de b C

5.  011 111 7. b

a C
010 110
mi
d

000  100

001 101

9. mi 11.  e  =  25,  v  =  15,  r  =  12  r  
=  e  −  v  +  2
b C
13.  e  =  25,  v  =  15,  3v  −  6  =  39  ≥  e
15.  13 17.  6,  9 19 ,  5;  2;  7
a d

21 a C d 23.  metro,  norte  ≤  2 27.  re a
i
j
25.  3
C
h
w  x  z b F
mi

gramo

Ejercicios  8.7  (pág.  595)

1.  4 3.  2 5.  4 7.  2 9.  2  11.  5 13.  3  15.  2


17.  mín{m,  n}  ≤  máx{m,  n} 19 ,  1,  6;  2,  3;  4;  5
21 ,  1,  3;  2;  4,  7;  5;  6 23.  1,  2,  4;  3,  5;  6;  7,  8,  9
25.  A,  D;  ANTES  DE  CRISTO 27.  A,  B,  F;  CD;  mi
Machine Translated by Google

986 Soluciones  a  ejercicios  impares

Ejercicios  de  repaso  (pág.  601)

a  B  C a  B  C  D  e
a 101 a 01110
b 011 b 10011  10010
1. 3.
C 111
           c     

d 11110         

mi 01010

5.  no 7.  no 9.  17 11.  nudo 13.  metro  =  norte

15.
a a C

b b C

C a b C

17
a b C d

b a d mi

C a d

d a b C mi

mi b d

19.  si;  f(a)  =  yo,  f(b)  =  j,  f(c)  =  k,  f(d)  =  l,  f(e)  =  metro,  f(f)  =  norte,  f(g)  =  o,
f(n)  =  pag

21.  si 23.  no 25.  no 27.  no 29.  si 31.  no

33. 35.  no
C d
37.  e  =  11,  v  =  7,  r  =  6;  r  =  e−v+2
ser
39.  13
F a
41.  si

43.  Suponga  que  K3,3  es  plano.  Para  K3,3,  v  =  6  y  e  =  9.  Entonces  2v  −  4  =  
12  −  4  =  8  <  e,  es  una  contradicción.     K3,3  no  es  plano.
45.  Suponga  que  K3,5  es  plano.  Entonces  2v  −  4  =  16  −  4  =  12  <  e,  que  
es  una  contradicción.     K3,5  no  es  plano.
Machine Translated by Google

Capítulo  9 Árboles 987

Ejercicios  complementarios  (pág.  604)

1.  n,  n  +  1,  2n 3.  norte  ≥  3 5.  si 7.  si


9.  Etiqueta  el  vértice  de  grado  n   11 vi,   vive
con  1  y  los  demás  (n  +  1):  2,   i=j
3, ... , 13.  Sean  G  =  (V1,  E1)  y  H  =  (V2,  E2)  dos  
gráficas  simples.  Suponga  que  f:  G  →  
2  3 nn  1
H  es  un  isomorfismo.  Sean  G  =  (V1,  E  
1)  y  H  =  (V2,  E  2)  sus  complementos.  
Sea  g:  G  →  H  definida  por  g(u)  =  f(u).  
Dado  que  |E1|=|E2|,  |E  1|=|E  2|.  Para  
1
mostrar  que  g  conserva  la  relación  de  
adyacencia,  suponga  {u,  v}     E  
15.  C5
Entonces  1.  {u,  v}     E1.  Entonces     f(v)}  
E{2;  
f(u),  
17.  Supón  que  el  gráfico  contiene  e   es  decir,  {g(u),  g(v)}     E  Así,  si  2.  {u,  
aristas.  Entonces  e  =  C(n,  2)/2   v}     E  entonces  {g(u),  g(v)}     E  1,  2.
=  n(n  −  1)/4.     4|n  o  4|(n  −  1),  
es  decir,  n  ≡  0  (mod  4)  o  n  ≡  1     g  es  un  isomorfismo.
(mod  4). Por  el  contrario,  sean  G  y  H  
isomorfos.  Entonces  (G )  =  G  y  (H )  =  
19.  (n  −  1)!/2
H  son  isomorfos,  por  la  primera  parte.
21.  (n!)  2/2

23.  f mi d

a  B  C

Eliminar  anuncio.  El  subgráfico  
resultante  es  plano:

alimentados

a b C

Capítulo  9 Árboles

Ejercicios  9.1  (pág.  613)

1.  si 3.  no 5.  no 7.  no;  contiene  ciclos.


9.  norte  =  8,  mi  =  7  =  norte  ­  1 11.  norte  =  12,  mi  =  11  =  norte  ­  1
Machine Translated by Google

988 Soluciones  a  ejercicios  impares

norte

13.  Por  el  teorema  8.1,   grado  (vi)  =  2  (número  de  aristas).  Pero,  por  el  Teorema  9.2,
i=1
norte

número  de  aristas  =  n  −  1.     i=1 grado  (vi)  =  2(n  −  1)  =  2n  −  2.

15.  si 17.  no 19.  m  =  1  o  n  =  1

21 23 25.  4,  6,  3,  5,  4,  4,  4,  5,  5,  5,  6

27.  Árbol  de  algoritmos  (A,n)

(*  Utilizando  el  Teorema  9.2,  este  algoritmo  determina  si  un  gráfico  con  matriz  de  
adyacencia  A  =  (aij)n×n  es  un  árbol.  Flag  es  una  variable  booleana  y  almacena  el  valor  
verdadero  si  el  gráfico  es  un  árbol  y  falso  en  caso  contrario.  *)

0.  Empezar  (*árbol*)  1.  
marcar  ←  falso  (*asumir  inicialmente  que  el  grafo  no  es  un  árbol*)  (*calcular  la  suma  de  los  grados  
de  los  vértices*)
2.  suma  ←  0
3.  Para  i  =  1  an  hacer
4. Para  j  =  1  an  hacer  aij  
5. suma  ←  suma  + 6.  
Si  sum  =  2n  ­  2  entonces
7.  bandera  ←  verdadero  8.  
Fin  (*árbol*)

Ejercicios  9.2  (pág.  624)

1.  norte  ­  1 3.  4 5.  un b 7.  segundo


C

a d

a 11.  un b
9. 13 d
F
b
gramo

b C
mi
i
mi h a
mi hg  fd i h j
d C
C
F k

gramo

15. b 17.  un b 19 b mi

C F
C d a h
a C mi F d gramo
Machine Translated by Google

Capítulo  9 Árboles 989

21 d 23.  un b 25.  segundo
C
b F
h
a d
a i
gramo

mi

27 a 29.  un 31 . d
F gramo

b mi
i
b C
mi h a
mi f.d. i h j
d C

hg C
F k

gramo

33. 2 35. 2 3 37.  sin  solución

3 1 5

4
1 4 5

39. q 41. 43.


q
q
q
q
q

45.  1 47.  16 49. 51.

53.  1 55.  9 57.  12 59.  125

Ejercicios  9.3  (pág.  633)

1.  segundo C 3. C

mi
2 4
2 5b  3   re  
_ 3
a d a mi
3 2 7
F
Machine Translated by Google

990 Soluciones  a  ejercicios  impares

5. b gramo
7. b C
8 mi
2 mi 2 4
7 6 4 2
h d
a 3 a
d 3
5
C
F
3 i
11 b gramo

8
9. 2 7
mi
C 6 4
5 h
a 3
re   d 5
segundo   3 C
F
mi
3 i
3  un

2 7  f

13 15. 4  6  2  45  6
4  6  2  45  6

5 5 7
5 5 7

1 3
1 3

17 19 2
2
5
5

3  3 6
3  3 6
1 4
1 4

Ejercicios  9.4  (pág.  643)

1.  1 3.  2 5.  metro  =  2 7  m  =  4


9.  no 11.  si 13.  no 15.  no,  no,  si
17 19 {a} 21.  13

{una,  b} {a},  {b}

23.  40,  81 25.  243 27.  4 29.  8


31.  31 33.  si 35.  sí   37.  si
39. 41.  mh mh+1  −  1
43.
metro  ­  1
45.  4
47.  729
Machine Translated by Google

Capítulo  9 Árboles 991

49.  l  =  norte  −  yo  =  (mi  +  1)  −  yo  =  (m  −  1)i  +  1 51.  si

53.  Como  m  ≥  2,  m  ∙  mh  ≥  2  ∙  mh.  Es  decir,  mh+1  ≥  2mh,  entonces  mh(m  −  1)  >
mh  −  1.  
  mh  >  (mh  −  1)/(m  −  1).  Es  decir,  l>i,  por  el  ejercicio  41.

Ejercicios  9.5  (pág.  661)

1.  a,  b,  d,  c,  e,  f 3.  a,  b,  d,  h,  i,  e,  c,  f,  g,  j,  k 5.  d,  h,  b,  e,  a,  f,  c,  i,  g

7.  d,  b,  e,  f,  c,  a   9.  h,i,  d,  e,  b,  f,  k,  j,  g,  c,  a 11.  a,  b,  d,  h,  e,  c,  f,  g,  i


13.  a,  b,  c,  d,  e,  f 15.  a,  b,  c,  d,  e,  f,  g,  h,  i,  j,  k

17.  −+  ↑  a  ↑  ac/def 19.  −  −  a  +   bcd     ef  ↑  gh

21.  abc  ↑↑  de/  +  f  − 23.  abc     re  +  e/f     g  ↑  −−


– –
25 27 29.  a  +  [b/(c  −  d)]  ↑  e
+ –
F

/
a * gramo h
a d mi
/ F
b C
+
mi

*
d

antes  de  Cristo

31.  [a  −  (b  +  c)]     re     mi  ↑  (f  −  g ) 33.  a/[(b  +  c)     re     mi  −  f


a C
37.  22
35. +     [e  +  (f  −  g)]
b   d
39.  32 43. *

41. + *

* – + C d

C – a b
a b mi F

Delaware
53.

X
45.  196
X +
47.  ↑ +  432/6  −  85  49.  
/ –
43  +  2     685  − /  ↑  51.  

[(4  +  3)     2]↑[6/(8  −  5)] xyz

yz
Machine Translated by Google

992 Soluciones  a  ejercicios  impares

55. 57.

abdominales
* w –  z

b  cd  e / z yw

xy

59.  Sea  l  el  número  de  hojas  y  n  el  número  de  vértices  en  un
n  +  1  
árbol  binario  completo.  Entonces,  por  el  Ejercicio  58,  l  = .  Como  l  es  un  número  entero,
2
n  debe  ser  impar.

61.  n  =  2l  −  1  63.  14;  42 65.  si  si  n  ≥  3 67.  no

69.  l1  =  1  =  l2  ln  
=  ln−1  +  ln−2,  norte  ≥  3

1 si  n  =  1  o  2
71.  vn  =
vn−1  +  vn−2  +  1  si  n  ≥  3

73.  e1  =  0  =  e2  
en  =  en−1  +  en−2  +  2,  norte  ≥  3

75.  Sea  an  =  in  +  1.  Entonces,  por  el  ejercicio  70,  a1  =  1  =  a2  y  an  =  an−1  +
an−2,  n  ≥  3.  Entonces  in  =  an  −  1  =  Fn  −  1.

77.  Sea  an  =  en  +  1.  Luego,  por  el  ejercicio  73,  a1  =  1  =  a2  y  an  =  an−1  +  an−2  +  1,  
donde  n  ≥  3.  Por  el  ejercicio  76,  en  =  an  −  1  =  (2Fn  −  1)  −  1  =  2Fn  −  2.

79.  Algoritmo  en  orden  (vértice) 81.  Evaluación  del  algoritmo  (vértice)

(*  Imprime  un  árbol  binario  usando   (*  Este  algoritmo  evalúa  un  árbol  de  
recursividad  *) expresión  binaria  usando  recursividad.  *)
Empezar  (*en  orden*)
si  árbol  =  Ø  entonces   Begin  (*  eval  *)  if  tree  
comenzar  (*  si  *)  en   =  Ø  then  begin  (*  if  *)  
orden  (subárbol  izquierdo)   eval  (subárbol  
imprimir  (vértice)  en  orden   izquierdo)  eval  (subárbol  
(subárbol  derecho)  endif derecho)  realiza  la  
operación  en  el  vértice  endif  End  
Fin  (*  en  orden  *) (*  eval  *)
Machine Translated by Google

Capítulo  9 Árboles 993

Ejercicios  9.6  (pág.  668)

1. i 3. tu 5. 8

a tu o
5 13
eo i

mi 32  21

7. hacer
a 9.  orden

re Ay

yo  sa
vestimenta

fa la Salir excursión

grito toma  de  corriente

11.  3 13.  5 15.  3 17.  4 19.  5


21 ochenta 23.  todos

eso
y Siete
brilla

atrás años es

oro  no

25.  Poligonal  del  algoritmo  (vértice) 27 34
(*  Este  algoritmo  imprime  el  contenido  
de  un  árbol  de  búsqueda  binario  enraizado  
13 21
en  el  vértice  en  orden  lexicográfico  utilizando  
el  recorrido  de  preorden.  *)
Inicio  (*  poligonal  *)
2 8  3 5
Si  árbol  =  Ø  entonces  
comenzar  (*  si  *)  
poligonal  (subárbol  izquierdo)  
1
imprimir  (vértice)  poligonal  
(subárbol  derecho)  endif

Fin  (*  poligonal  *)
Machine Translated by Google

994 Soluciones  a  ejercicios  impares

Ejercicios  9.7  (pág.  675)

1.  011101101110 3.  1101011011010001  5.  AÑO

7.  UNA  HORMIGA 9.  110 11.  011

13.  01000111 15.  11011100010110111

17.  cierva 19.  café

21 23.  00,  1110,  1111,  110,  10,  01
0 1
25.  111000111010
0 01 1

1 27.  11111011010110
a fe 0

1 29.  cama
d 0

C
31.  borrar
b
35.  0001
33.  1100
39.  11001010011111110001100
37.  0011010010011010000100
43.  equipaje  45.  47
41.  egloga

Ejercicios  9.8  (pág.  679)

1.  7 9. C.A
3.  11 < = <
5.  3
ab bd ab
7.  4 =
<
<
< = <
aL  CH bL  dH  cL  aH

11.  Ordenación  por  algoritmo  (a,b,c) 13.  Algoritmo  de  moneda  falsa  
(*  Este  algoritmo  ordena  los  elementos  
(a,b,c,d,e,f,g)
a,  b  y  c  en  orden  léxico­gráfico.  *) (*  Hay  siete  monedas  de  la  a  a  la  
g;  una  de  ellas  es  más  pesada.  
Empezar  (*  ordenar  *)   Este  algoritmo  identifica  la  moneda  
si  a  <  b  entonces falsa  utilizando  una  balanza  de  brazos  
si  c  <  a  entonces iguales  y  un  número  mínimo  de  pesajes.  *)
imprimir  (c,a,b)  
más  si  b<c  entonces Begin  (*moneda  falsificada*)
imprimir  (a,  b,  c)   si  a  +  b  +  c  <  d  +  e  +  f  entonces  si  d  <  e  
más entonces
imprimir  (a,  c,  b) es  mas  pesado
Machine Translated by Google

Capítulo  9 Árboles 995

else  ifc<b  entonces   else  ifd=e  entonces  f  es  
print(c,b,a)  else   más  pesado  else
ifc<a  entonces
imprimir  (b,  c,  a)   d  es  más  pesado
más más  ifa+b+c=d+e+f  entonces
imprimir  (b,  a,  c) g  es  más  pesado  
Fin  (*  ordenar  *) ifa<b  entonces
b  es  mas  pesado
si  no  ifa=b  entonces
c  es  mas  pesado
más
a  es  mas  pesado
Fin  (*moneda  falsificada*)

Ejercicios  de  repaso  (pág.  681)

1. gramo
3. b

b mi C

a
d h novia
a d

mi
C
F
i

5. b F 7. 2 9. b F
C C

a a
134
d d
gramo
5 gramo

mi mi

11 2 13 b 2 mi 15.  1  35
2 4 3
4 2 d 2 2 3 4
3 a gramo

1 5 3
2 4
C F

17 b 2 mi
19 1  35
3
3 4
2 d 2 2 3 4
a gramo

3
2 4
C F
Machine Translated by Google

996 Soluciones  a  ejercicios  impares

21 libro

CH1 CH2 CH3 CH4

S1  S2  S1  S2  S3  S1 S2  S3  S1  S2

SUB1  SUB1  SUB2

23.  3 25.  2187  27.  61 29.  75 31.  3 33.  3

0  si  n  =  1  rn/2  +  1  si  
35.  rn  =
n  ≥  2

37.  prueba  (por  PMI):  Sea  P(n) :  rn  =  lg  n.  Cuando  n  =  1,  r1  =  0  =  lg  1.     P(1)  es  
verdadera.  Suponga  que  P(k)  es  verdadera;  es  decir,  suponga  que  rk  =  lg  k.
Entonces  rk+1  =  r(k+1)/2  +1  =  lg  [(k+1)/2]+1  =  lg  (k+1).     P(k+1)  es  verdadero.  
Así,  el  resultado  se  sigue  por  inducción.
39.  abdgehcfij,  dgbehacijf,  gdhebjifca

41.  −  +  a    /bc/  +  de  −  fg  ↑  hola 43.  abc/de  +  fg  − /     +hola  ↑  −


45. 47.

↑ ≤ ≤
a * h i
X y
/ /
X z w z
b C

d  e  g

49.  +  +  un  −  bc  ↑  re  ↑  ef 51.  abc  −  +def  ↑↑  +
53.  27 55.  78
57. Matemáticas
59. 0 1

no  es 0 1 0 1

a espectador mi
un  0 1  01

deporte norte
do  0 10  1

61.  1010110011010000100 0 1  tonelada 0 1

63.  111101100011100011111 dl ( )
65.  (c  +  a) 67.  5 69.  7
Machine Translated by Google

Capítulo  10 dígrafos 997

Ejercicios  complementarios  (pág.  686)

1. 3.  10,  6

5.  Sea  e  el  número  de  aristas  en  F.  Sea  Ti  =  (Vi,  Ei)  un  árbol  en  F,  1  ≤  i  ≤  k.  Sea  |
Vi|  =  vi  y  |Ei|  =  ei.  Entonces  vi  =  n  y  ei  =  vi  −  1.     e  =  ei  =  (vi  −  1)  =  vi  −  k  =  n  −  
k.

7.  Dado  que  un  árbol  es  plano,  el  número  r  de  regiones  formadas  es  1.  Por  la  
fórmula  de  Euler,  el  número  de  aristas  e  del  árbol  viene  dado  por  e  =  r  +v−2  
=  1  +  n  −  2  =  n  −  1 .

9.  Sean  T  y  T  dos  árboles  generadores  de  un  grafo  conexo  G  con  n  vértices.  
Suponga  que  T  y  T  contienen  aristas  e  y  e  respectivamente.  Entonces,  
por  el  Teorema  9.2,  e  =  n  −  1  =  e .
11.  •————•

13.  Dado  que  e  es  un  puente,  (V,  E­{e})  se  desconecta  con  dos  subgrafos,  cada  uno
siendo  un  árbol.  Así  que  es  un  bosque  de  dos  árboles.

15.  3  pesajes

Capítulo  10 dígrafos

Ejercicios  10.1  (pág.  704)

1.  2,  1,  3;  2,  3,1;  4,  4,  4 3.  sin  fuentes  ni  sumideros.
a  B  C
a 101 grado  (vi)  =  2  +  1  +  3
5. 7.
b 111
C 001
      =  6  =  e  =  fuera  de  grado  (vi)

9.  C(n,  2) 11.  3 13.  2 15.  segundo 17.  si


111
19 111 21.  si
111
     

23.  Sea  A  =  (aij)n×n  la  matriz  de  adyacencia  de  un  dígrafo.  Supongamos  que  la  fila  k  
es  cero;  es  decir,  akj  =  0   k.  Entonces,  la  fila  k  de  cada  potencia  de  A[m]  es  cero,  
por  lo  que  la  fila  k  de  la  matriz  de  accesibilidad  es  cero.     Por  el  Teorema  10.3,  
el  dígrafo  no  está  fuertemente  conectado.
Machine Translated by Google

998 Soluciones  a  ejercicios  impares

25 b 27.  si 29.  si
31.  si 33.  fregadero

a C 35.  0011 37.  00101110

39. 0
41.
0 0 1
0 0
1 1
0 1
0 1
0 0 1

43.  16 0 0

47.  0011
1 1
49.  0000111101100101
1 0
1 0
1

Ejercicios  10.2  (pág.  713)

1.  no 3.  si 5.  sin  fuentes;  una,  c,  d,  f


7.  un;  sin  sumideros. 9.  sin  fuentes;  una,  c,  d,  f 11.  a  y  e;  F
13.  no 15.  si 17.  si
19 mi F
21 ↓
23 ↓

BD X *
GRAMO

X
C A
↓ w

y z z
y

25 27 *

a


*

C a d

b C
b

29.  10 31.  306
33.  ab  +  ab  +  bc  +  − 35.  abc−  ↑  bc  −  +bc  −  abc−  ↑  bc  −   /−
37.     +  ab  −  +ab  +  bc 39.  −+  ↑  un  −  bc  −  bc/  −  bc     ↑  un  −  bc  −  bc
Machine Translated by Google

Capítulo  10 dígrafos 999

41.  (a  +  b)     ((a  +  b)  −  (b  +  c))

43.  ((a  ↑  (b  −  c))  +  (b  −  c))  −  (b  −  c)/((a  ↑  (b  −  c)     (b  −  c))

45.  Suponga  que  dag  D  contiene  n  vértices  y  ningún  sumidero.  Entonces  outdeg  
(v)  ≥  1   v.  Como  outdeg  (v)  ≥  1,  v  tiene  un  sucesor  inmediato  v1.  Como  
outdeg  (v1)  ≥  1,  v1  tiene  un  sucesor  inmediato  v2.  Continuando  así,  resulta  
un  camino  dirigido:  v­v1­v2­  ∙∙∙  ­vi.  Si  dos  vértices  a  lo  largo  de  este  camino  
son  iguales,  entonces  D  sería  cíclico,  una  contradicción.  Si  no  hay  dos  
vértices  idénticos,  se  puede  continuar  el  procedimiento  hasta  obtener  un  
camino  que  continúe  con  todos  los  vértices  en  D :  v­v1­v2­  ∙∙∙  ­vi­  ∙∙∙  ­vj.  Como  
outdeg  (vj)  ≥  1,  vj  tiene  un  sucesor  inmediato  y  debe  ser  uno  de  los  vértices  
del  camino.  Entonces,  el  camino  y,  por  lo  tanto ,  D  contiene  un  ciclo,  una  
contradicción.     D  contiene  una  fuente.

47.  t1,  t2,  t3,  t4,  t5,  t6,  t7,  t9,  t8,  t10,  t11,  t12,  t13 49.  E,  G,  B,  F,  C,  A,  D

Ejercicios  10.3  (pág.  723)

1.  9 a  B  C  D  e
a 08502
3.  12
b 00660
5.
C      00000  
d 30300
        

mi 30740

7.  acbef,  13;  abef,  11; 9.  aceg,  11;  acbeg,  11;  abeg,  
abedbef,  21;  adbe,  15 9;  anuncio,  13
11.  2 13.  3 15.  2 17.  3 19.  2 21.  3
23.  2 25.  4 27.  abc 29.  2 31.  4

33  ab,  2;  adc,  4;  anuncio,  1;  ade,  6;   35.  acb,  6;  ca,  2;  anuncio,  
adcf,  11;  abg,  8;  ad,  4;  adici,  12 5;  as,  9;  adf,  13;  anuncio,  13

37  ab,  2;  ca,  4;  anuncio,  1;  ade,  6;  adcf,  11;  abg,  8;  ad,  4;  adici,  12
39.  Algoritmo  Floyd  (D,W,P)
(*  P  es  una  matriz  unidimensional  tal  que  Pi  es  el  predecesor  inmediato  del  vértice  i  a  
lo  largo  del  camino  más  corto  desde  la  fuente  1.  *)
Comenzar  (*  Floyd  *)  (*  
inicializar  P.  *)  para  i  =  1  
a  n  hacer
Pi  ←  1  
para  i  =  1  para  n  hacer
para  j  =  1  a  n  hacer
para  k  =  1  a  n  hacer  
si  wji  wjk  entonces  +  wik  <
Machine Translated by Google

1000 Soluciones  a  ejercicios  impares

comenzar  (*  si  
*)  wjk  ←  wji  +  wik
Pk  ←  i  
endif
Fin  (*Floyd*)

41.  ab,  ac,  ad,  ade,  adcf,  abg,  adh,  adci

Ejercicios  de  repaso  (pág.  727)

1.  1,  3,  1,  0,  1;  1,  2,  0,  2,  1 3.  si

01101 7.  si 9.  010,  101,  011,  111,  110,  100,  001


01101 11.  no 13.  no
5.      00000     
11101         

15.  221201100
01101

17.  33,  32,  22,  21,  12,  20,  02,  23,  31,  11,  13,  11,  13,  30,  01,  10,  00,  03
19.  3 21.  re 23.  no
25 → 27 29.  31

X

* C d

y
z a b

31.  ab     c  ↑  ba  −  d  +  + 33.  325  ↑  +  25  ↑  24    /−


35.  +↑   abc  +  −malo 37.  −  +  3  ↑  25/  ↑  25     24
39.  (3  +  2  ↑  5)  −  2  ↑  5/(2     5) 41.  7
43.  15 45.  suplicar 47.  acfg;  11

Ejercicios  complementarios  (pág.  730)

1.  norte  ­  1 3.  n2  −  2n 5.  C(n,  2)

7.  prueba  (por  PMI):  Supongamos  que  el  torneo  tiene  n  vértices.  Cuando  n  
=  2,  el  torneo  tiene  un  solo  borde  y  el  borde  es  un  camino  hamiltoniano.
Ahora  suponga  que  todo  torneo  con  n  vértices  tiene  un  camino  
hamiltoniano.  Sea  T  un  torneo  con  n  +  1  vértices,  v1,  v2, ... ,  vn+1.  
Eliminar  vn+1  de  T.  Esto  produce  un  torneo  con  n  vértices;  por  el  IH,  
tiene  un  camino  hamiltoniano,  digamos,  v1­v2­  ∙∙∙  ­vn.
Machine Translated by Google

Capítulo  11 Lenguajes  formales  y  máquinas  de  estados  finitos 1001

Volviendo  al  torneo  T,  si  vn­vn+1  es  una  arista  en  T,  entonces  v1­v2­  ∙∙∙  ­vn­vn+1  
es  un  camino  hamiltoniano  en  T;  por  otro  lado,  si  vn+1­v1  es  una  arista  en  T,  
entonces  vn+1­v1­v2­  ∙∙∙  ­vn  es  un  camino  hamiltoniano  en  T.  De  lo  contrario,  debe  
haber  un  entero  positivo  i  tal  que  ambos  vi­vn+1  y  vn+1­vi+1  son  aristas  en  T.  
Entonces  v1­v2­  ∙∙∙  vi­vn+1­vi+1­  ∙∙∙  ­vn  es  un  camino  hamiltoniano  en  T.

Así,  por  PMI,  el  resultado  es  cierto   n  ≥  2;  es  decir,  cada  torneo
con  n  ≥  2  vértices  tiene  camino  hamiltoniano.

9.  Tanto  en  D1  como  en  D2,  a  es  una  raíz.  En  D3,  cada  vértice  es  una  raíz.

11.  aecdb 13.  abdc 15.  no

17.  Un  grafo  es  fuertemente  orientable  si  es  conexo  y  no  tiene  puentes.

Capítulo  11 Lenguajes  formales  y  máquinas  de  estados  finitos

Ejercicios  11.1  (pág.  741)

1.  b,  bb,  bab,  bbabb,  bbabb 3.  λ,  b2,  b4,  b6,  b6
5.  λ,  a,  b     L;  x   7.  0     L;  x     L     1x,  xx     L.
  L     axa,  bxb     L.
11.  00     L;  x     L     x0,  x1     L.
9.  1     L;  x     L     xx     L.
13.  F. 15.  F.

17.  no 19.  inválido

21.  λ     S;  x     S,  y     S     xy     S. 23.  λ      ;  x      ,  y  


25.  00,  01,  10,  11      xy      .

27.  0,  1,  2

29.  prueba  (por  contradicción):  Suponga  que     es  finito.  Como  =  Ø,  contiene  un  elemento  
a.  Sea  x  una  palabra  de  mayor  longitud  en   .  Entonces  l(xa)  =  l(x)+  l(a)  >  l(x).     
xa     y  es  más  largo  que  x,  lo  cual  es  una  contradicción.

31.  {a,  bc,  aba,  abbc,  bca,  bcbc}  

33.  {a3,  a2bc,  abca,  a(bc)  2,  bca2,  bcabc,  bcbca,  (bc)  3}

35.  A  =  {λ}{a,  bc}={a,  bc}  =  A

37.  (B     C)A  =  {aa,  aab,  ba,  bab,  ca,  taxi,  aba,  abab}
=  BA     CA

39.  (B  ∩  C)A  =  Ø  =  BA  ∩  CA
Machine Translated by Google

1002 Soluciones  a  ejercicios  impares

41.  F. 43.  T 45.  T 47.  T

49.  0,  01,  011 51.  01,  013,  015 53.  λ,  01,  0101 55.  λ,  0,  1,  01

57.  demostración  (por  PMI):  Dado  que   ,  el  resultado  es  verdadero  cuando  n  =  0.  Suponga  
que  es  verdadero  para  un  entero  arbitrario  k  ≥  0.  Entonces  Ak+1     ABk+1     BBk+1  
=  Bk+1.  Así  el  resultado  sigue  por  PMI.

59.  Como  A     A ,  A      (A )   .  Por  el  contrario,  sea  x     (A )   .  Entonces  x     (A )  
m  para  algún  entero  m  ≥  0.  Entonces  x  =  ym,  donde  y     A .  Como  y     A ,  y     
A .  Entonces  (A )        A .  Así  (A )     =  A .  x  =  y

61.  A  =  {λ}A  =  {λa|a     A}={a|a     A}  =  A  63.  


Como  A1  =  A,  el  resultado  es  el  siguiente.
65.  Sea  x     (B     C)A.  Entonces  x  =  ya,  donde  y     B     C  y  a     A.  Si  y     B,  entonces  ya  
  BA,  entonces  x     BC     CA.  Si  y     C,  entonces  ya     CA,  entonces  x     BC     CA.
Así,  en  ambos  casos,  (B     C)A     BC     CA.
Por  el  contrario,  sea  x     BA     CA.  Si  x     BA,  entonces  x  =  ba,  donde  b     
B  y  a     A.  Como  b     B     C,  x     (B     C)A.  De  manera  similar,  si  x     CA,  
entonces  también  x     (B     C)A.  De  nuevo  en  ambos  casos,  BC     CA     (B     
C)A.  Así  (B     C)A  =  BC     CA.

67.  (A B )     =  (A     B)     =  (B     A)     =  (B A )  

Ejercicios  11.2  (pág.  755)

1.  si 3.  no 5. frase

notario  público
verbo op

arte  sustantivo come Arte sustantivo

la gato el  pollo

7. σ 9. σ 11.  si 13.  si

a A a A

a b A

b A

15. σ
a

a A

b
Machine Translated by Google

Capítulo  11 Lenguajes  formales  y  máquinas  de  estados  finitos 1003

17 σ 19.  no 21.  si
23.  {y  |  norte  ≥  1 } 25.  no
a σ
27.  si 29.  {aa,  abb}
a σ
31.  σ  →  σ  σ,  σ  →  1
a A 33.  σ  →  σ0,  σ  →  σ1,  σ  →  00
35.  σ  →  σ0,  σ  →  1σ,  σ  →  1
b

37.  σ  →  aσ,  σ  →  aA,  A  →  b

39.  σ  →  aAB,  A  →  Aa,  B  →  Bb,  A  →  a,  B  →  b

41. entero 43. id::=  letra|alfa  alfa::=  


letra|letrafa|  ui|uialfa  ui::=  dígito|
interfaz  de  usuario

letra  digitui::=  a|b|
c| ...  |z  dígito::=  0|1|2| ...  |9
dígito interfaz  de  usuario

2 dígito interfaz  de  usuario

3 dígito

45.  no 47.  no
51.
49.
identificación

identificación

esparto
esparto

dejar esparto
dejar esparto

metro dejar esparto


v dejar esparto

a
a dejar esparto
dejar esparto

t dejar
yo dejar esparto

h
tu dejar

mi

53.  si 55.  no
Machine Translated by Google

1004 Soluciones  a  ejercicios  impares

57. expr

expr optar expr

dígito expr

5
expr optar expr

dígito ↑ dígito

4 3

59. expr

expr

expr optar expr

↑ expr
dígito

3
expr optar expr

dígito dígito

5 2

61.  si 63.  si

65.  número 67. número

dn dn
signo


interfaz  de  usuario

dígito interfaz  de  usuario interfaz  de  usuario

interfaz  de  usuario
dígito 6 dígito interfaz  de  usuario

dígito

dígito 7 0 dígito 3

3 2

69.  si 71.  si
Machine Translated by Google

Capítulo  11 Lenguajes  formales  y  máquinas  de  estados  finitos 1005

73. expr

término

término mes factor

factor * expr

dejar expr ao término

a término término mes factor

factor factor dejar

dejar dejar d

b C

75.  si 77.  si

79.  un::=  ui∙uiEui|ui∙uiEsignui  ui::=  digit|digitui  
sign::=  +|−

81.  no 83.  si

Ejercicios  11.3  (pág.  768)

1.  s0­s1­s1­s1­s2 3.  s0­s1­s2­s3­s0
5.  s0­s1­s2­s3­s1­s2 7.  s0­s1­s4­s4­s4­s4­s4
9.  no 11.  si
13.  no 15.  no

17
a b
b b
→ s0 s1 s2
a
a

19 a a a a,  b

b  bb  s2  s1
→ s0 3
Machine Translated by Google

1006 Soluciones  a  ejercicios  impares

21 23
F F
yo yo
S abdominales
S abdominales

     
s0 s1  s0 s0 s1  s2
s1 s1  s2 s1 s1  s3
s2 s1  s0 s2 s2  s2
s3 s1  s3

25.  Palabras  terminadas  en  bb. 27.  Palabras  terminadas  en  bb.

29.  Palabras  que  contienen  exactamente  dos  b. 31.  Palabras  terminadas  en  ab.

33.  Palabras  que  comienzan  con  a  y  terminan  en  b.
35.  Palabras  que  contienen  un  número  impar  de  a  y  un  número  impar  de  b.
37. a,  b

a a
→ s0 s2 s3

bb  a,  b

s1

39. b a,  b
a a a
3
→ s0 s1 s2
b
b

41.
a b a,  b
b a a b
→ s0 s1 s2 s3 s4
b
a

43. b

b b
→ (0,0) (0,1) (0,2)
a a a a
b b
a (1,0) (1,1) (1,2) a

a a b a

b b
(2,0) (2,1) (2,2)

b
Machine Translated by Google

Capítulo  11 Lenguajes  formales  y  máquinas  de  estados  finitos 1007

45. 3
2
re  
≠  
3 s1 s2
d
→ d  ≠  2,4
s0 s5
d
re  ≠  4
4
s3 s4
5

47. d
, d
d • d 3
→ s0 s1 s2
re,  
do  
≠  
• ≠  
c  
re
c  ≠  re
c  ≠,,d
s4

Ejercicios  11.4  (pág.  777)

1.  s1 3.  s1
5.  0 7.  1
9. un/1 un/0

b/0 b/0
→ s0 s1 s2
b/1

un/1

11 un/0 b/1 un/1

un/0 b/1 un/0


→ s0 s2 3
s1
b/0

b/0

13
F gramo

     
yo
S abab

s0 s0 s1 1   0  

s1 s2 s2 1   0  

s2 s2 s0 0 1
Machine Translated by Google

1008 Soluciones  a  ejercicios  impares

15.
F gramo

yo
S a b abdominales
  
s0   s0   s1   0   1  

s1   s1   s2   0   1  

s2   s2   s3   0   1  

s3 s3 s1 0 1

17.  0101 19.  001011 21.  0110 23.  0000

25
F gramo

yo
S 00 01 10 11 00 01 10 11
  
C0 C0  C0  C0  C1  0110
C1 C0  C1  C1  C1  1001

27.  11001 29.  110000

31 un/0 b/1
b/0
b/1
→ s0 s1 s2
un/0

un/0

33. un/0 un/0 un/1 un/0

b/0 b/1 b/0


→ s  
0 s1 s2 s3 b/0

35.  no 37.  si

39.  Palabras  terminadas  en  aa. 41. b/0 b/1 un/0

43.  g(s,  x)  =  0 → s  
0
un/1
s1
un/0
s2 b/0

Ejercicios  11.5  (pág.  781)

1.  no 3.  no

5.  s0  →  as1,  s0  →  bs0,  s1  →  as1,  s1  →  bs2,  s2  →  as1,  s2  →  bs3,  s3  →  as4,  s3  →  bs0,  s3  →  a,  s4  →  a,  s4  →  b
Machine Translated by Google

Capítulo  11 Lenguajes  formales  y  máquinas  de  estados  finitos 1009

7.  s0  →  as0,  s0  →  bs1,  s1  →  as0,  s1  →  bs2,  s2  →  as0,  s2  →  bs2,  s1  →  b,
s2  →  segundo

9.  s0  →  as0,  s0  →  bs1,  s1  →  as1,  s1  →  bs2,  s2  →  as2,  s2  →  bs3,  s3  →  as3,  s3  →  bs3,  
s2  →  b,  s3  →  a,  s3  →  b

11.  s0  →  as1,  s0  →  bs0,  s1  →  as1,  s1  →  bs2,  s2  →  as1,  s2  →  bs0,  s1  →  b

13.  s0  →  as1,  s0  →  bs2,  s1  →  as1,  s1  →  bs3,  s2  →  as2,  s2  →  bs2,  s3  →  as1,
s3  →  bs3,  s1  →  segundo ,  s3  →  segundo

15.  s0  →  as1,  s0  →  bs0,  s1  →  as2,  s1  →  bs1,  s2  →  as2,  s2  →  bs2,  s1  →  a,
s2  →  segundo

17.  s0  →  as2,  s0  →  bs1,  s1  →  as1,  s1  →  bs1,  s2  →  as3,  s2  →  bs1,  s3  →  as3,  s3  →  bs3,  
s2  →  a,  s3  →  a,  s3  →  b

19.  s0  →  as1,  s0  →  bs0,  s1  →  as1,  s1  →  bs2,  s2  →  as3,  s2  →  bs0,  s3  →  as3,  s3  →  bs3,  
s2  →  a,  s3  →  a,  s3  →  b

21.  s0  →  as1,  s0  →  bs2,  s1  →  as4,  s1  →  bs3,  s2  →  as3,  s2  →  bs4,  s3  →  as3,  s3  →  bs3,  
s4  →  as4,  s4  →  bs4,  s4  →  a,  s4  →  b

Ejercicios  11.6  (pág.  785)

1. 3.
a a a,  b a
a b
→ s0 s1 s2 → a a
s0 s1 s2
b

b
s3 ba,b

5. a a,  b 7.
yo
S a b
a a
→ s0 s1 s2
s0 {s0,  s1}   {s2}  
s1 {s1}   {s2}  
b b
s2 {s2} {s2}
s4 s3
a,  b
Machine Translated by Google

1010 Soluciones  a  ejercicios  impares

9. yo
11.  sí;  σ­ABBF  13.  
S a b
   sí;  σ­AAAF  15.  sí;  
s0   {s0,  s1}   {s5}  
s1   {s1,  s2}   {s5}   s0­s1­s1­s2
s2   {s2}  {s3}   {s3}   17.  no
s3   {s4}  {s5} {s4}  
s4   {s4}   19.  no
s5 {s5}
21.  sí;  s0­s1­s2­s3­s4

23 a b

a b
→ σ A B
a
F

25 b a a,  b
b
a b a
→  σBCD  _ A
a
a
F
b

27 b b
29 a,  b
a,  b
a a a
a →  s0 s2
→  s0 s1 s2 s3
b

a cama  y  desayuno

a,  b
s1

31 a,  b a,  b
a

a b a
→  s0 s1 s2 s3

33.
s1
a a,  b
b
a,  b

→  s0 s3 s4

a
b b
s2
Machine Translated by Google

Capítulo  11 Lenguajes  formales  y  máquinas  de  estados  finitos 1011

35. a a

→  sábado a b
0 s1 s2 s3
b
a,  b
s5 a,  b

Ejercicios  11.7  (pág.  792)

1. b a
a
→ s0  { } { } 1
b
a

{s0,  s2}

3. b a
a
a b
→  { }  s0 {s  0,  s1} {s1,  s3 } {s1,  s2,  s3}
b
b a b
a,  b a

{}  s3 {s0,  s1,  s2}

5.
a,  b

{}  s2
b

→ b
{ }  s0

a
a
{s0,  s1}

7.
a,  b
φ
b a,  b
a b
→ { }  σ {A} {B}

a b
a

{A,  F}
Machine Translated by Google

1012 Soluciones  a  ejercicios  impares

9. b

a a →
s0 s1

s2
b
b

11
b a

a b b →
s0 s1

s2 s3
a
a
b

13 15.
b a,  b b a a,  b
a → a b a →
s0 s1

s0 s1

s2 s3

17.  Palabras  que  comienzan  con  aa. 19.  Palabras  que  comienzan  con  bba.
21.  Palabras  con  al  menos  una  a. 23.  Palabras  que  contienen  aba  como  subcadena.

25
a,  b

φ
b a
b
b
a a b
→ {s  2} {s1 } {s0} {s0,  s2}
a

27
a,  b

φ
a b
a a,  b

b b
→ {s  3} {s1} {s0,  s1,  s2,  s3}
a

{s2 }
Machine Translated by Google

Capítulo  11 Lenguajes  formales  y  máquinas  de  estados  finitos 1013

29
b a,  b
a
→ {s1} {s1,  s2}

31 a

{s0,  s3}
a
b a,  b
b
b a
→ {s3} {s2,  s3} {s0,  s1,  s2,  s3}

Ejercicios  de  repaso  (pág.  794)

1.  {a,  aa,  ab,  aab,  bca,  bcab}  
3.  {λ,  a,  aa,  bc,  abc,  bca,  bcbc},  {λ,  a,  aa,  bc,  abc,  bca,  bcbc,  a3,  abca,  bca2,  
bcbca,  a2bc,  abcbc,  bcabc,  bcbcbc}

5.  a,  b,  ac 7.  ab,  abab,  ababab 9.  si


11 frase

notario  público
verbo notario  público

Arte adj  sustantivo  come Arte sustantivo

el  lobo  discreto la repollo

13.  no 15.  si
17 σ 19 σ

b σ a A

a A b σ

a B a A
a
b

21.  σ  →  segundoσ,  σ  →  segundo 23.  σ  →  bσb,  σ  →  b
Machine Translated by Google

1014 Soluciones  a  ejercicios  impares

25 mientras  ejecuta

tiempo expr hacer sentencia

vble optar vble asmt  stmt

X y vble expr

X vble signo vble

y z

27
a,  b

a b b
→  s0 s1 s2 s3

b a a

s4 a,  b

29 yo
S a b
  
s0   s1   s0  
s1   s2   s0  
s2 s2 s2

31.  Las  palabras  que  comienzan  con  abb. 33.  Palabras  que  contienen  aa.

35.
a,  b
a a a
→  s0 s1 s2 s3

b  bb

s4
a,  b

37. un/1 un/1


b/0 un/0 b/0
→ s0 s1 s2 s3
b/0 b/0

un/1
Machine Translated by Google

Capítulo  11 Lenguajes  formales  y  máquinas  de  estados  finitos 1015

39.
F gramo

yo
S abab
  
s0   s0   s1   0   1  

s1   s2   s1   0   1  

s2 s2 s2 1 0

41.  0010111 43.  010110

45. un/1

un/0 b/0 b/1


→  s0 s1 s2 s3b /1

b/0 un/0  un/0

b/0 s4 un/0

47.
b/0   un/1
un/0 b/0 b/1
→ s0 s1 s2 s3
un/0
b/1

49.  σ  →  aA,  σ  →  bC,  A  →  bB,  A  →  aC,  B  
→  aC,  C  →  aC,  C  →  bC,  B  →  bD,  D  
→  aD,  D  →  bD ,  D  →  a,  D  →  b

51.
b b a,  b
a
→ s0 s1 s2
b

53.
S      yo
a b

s0 {s1}   {s2}  
s1 {s0,  s1}   {s1}  
s2 {s1}  {s3} {s3}  
s3 {s2,  s2}

55.  si 57.  no

59.  si 61.  si
Machine Translated by Google

1016 Soluciones  a  ejercicios  impares

63.
b b a,  b
a a
→  σ A B
b
a b
F

sesenta  y  cinco.

b a,  b
a a
→ {s0} {s  1} {s2 }

b a b

{s1,  s2}

67.  Palabras  que  contienen  exactamente  uno  a.

69.  Por  el  ejercicio  68  de  la  sección  11.1,  (A     B )     =  (A     B)     =  (A      B)   .

Ejercicios  complementarios  (pág.  798)

1. 3.
b b
→ (0,  0) (0,  1) → (0,  0) (0,  1)
b b

aa  aa Automóvil  club  británico
Automóvil  club  británico

b b
(1,  0) (1,  1) (1,  0) (1,  1)
b b

5.
, d d , d
d d  Ed  s3  s4
→ s0 s1 s2 s5

do  ≠  re, c  ≠  d,e
c  ≠  re
c  ≠ ,,d
c  ≠ ,,d c  ≠  re

s2

7.  σ  →  aA,  σ  →  bC,  A  →  aA,  A  →  bB,  B  →  aA,  B  →  bB,  C  →  aC,
C  →  bC,  B  →  b

9.  wfnp::=  λ|wfnp
Machine Translated by Google

Capítulo  12 Álgebra  Booleana  y  Circuitos  Combinatorios 1017

11 13
0 0
0 gramo

1 1 entrada  f
→ s0 s1 s2

1 1 s 0 1

0  1 1 s0 s0 s1 1  

s1 s2 s0 0  

s2 s2 s1 1

s3  
1
17.  101110
15.  1101

19.  Supongamos  que  x  =  x1x2 ...xn     L.  Entonces  hay  un  DFSA  A  que  exceptúa  x,  
entonces  hay  un  camino  que  comienza  en  s0  y  termina  en  un  estado  de  aceptación  sn.
Al  invertir  el  camino,  se  obtiene  la  cadena  xnxn−1 ...x1.  Entonces,  hacer  sn  el  
estado  inicial  y  s0  el  estado  de  aceptación  produce  un  NDFSA  A  que  acepta  
LR.  (Si  A  contiene  más  de  un  estado  de  aceptación,  introduzca  un  nuevo  
estado  s;  luego,  correspondiente  a  cada  arista  entrante  a  un  estado  de  
aceptación,  agregue  una  arista  que  termine  en  s).  Ahora  el  resultado  se  sigue  
de  los  teoremas  11.3  y  11.4.

Capítulo  12 Álgebra  Booleana  y  Circuitos  Combinatorios

Ejercicios  12.1  (pág.  812)

1.  30 3.  30 5.  1 7.  5 9.  6


11.  5 13.  35 15.  2 17.  70 19.  10

21.  5  =  70/5  =  14      23.  5  (5     7)  =  5  35  =  5


(5 )  =  14  =  70/14  =  5
25.  2n 27.  si 29.  no 31.  si 33.  si

35.
+  0 1ab  _ ∙  0  1  ab

1111  aa  1  a1  
  1a  b
b  
  b
0  1
0  b
1  1 0  0000  1  0  1  aba  0  aa  
0  b  0  b  0  b

0  =  1,  1  =  0,  a  =  b,  b  =  a.

37.  x  +  xy  =  x 39.  (x  +  y)  =  xy
Machine Translated by Google

1018 Soluciones  a  ejercicios  impares

41.  xx  =  xx  +  0  =  xx  +  xx   43.  Por  la  ley  del  complemento,  0  +  0  =  1.
=  x(x  +  x )  =  x1  =  x   Pero  por  ley  de  identidad,  0  +  0  =  
45.  x(x+y)  =  xx+xy  =  x+xy  =  x  49.   0 .     0  =  1.
(x  +  y)z  =  z  (x  +  y)  =  zx  +  zy  =  xz   47.  (x+y)  =  xy .     [(x+y) ]  =  (xy )  es  
+  yz  51.  (x+z)( y+z)  =  x( y+z) decir,  x  +  y  =  (xy )
+z( y+z)  =  xy  +  xz  +  zy  +  zz  =  xy  +  xz  
+  zy  +  z  =  xy  +  (z  +  zx)  +  zy  =  xy  
+  (z  +  zy)  =  xy  +  z

Ejercicios  12.2  (pág.  821)

1.  1,  0 3.  2 5.  si 7.  si


9.
x  y X yx+yx+  y (x  +  y )(x  +  y)
0  0  11  1  0  1  10  0  1  0  01  1  1   1   1  
1  00  1 1   0  
0   0  
1 1

11
xyz  xy  yyzz  yz  xy  +  yz  +  yz
000  01010  0  001  01100  1  010  00011  1  011  
00000  0  100  01010  0  101  01100  1  110  00011  
1  111  10000  1

13
xyz  y  x+y  +  z  xy  z  (x  +  y  +  z)  (xy  z)
000  11  0  001  11  0  010  00  0  011   0  

01  0  100  11  0  101  11  1  110  01  0   0  

111  01  0 0  

0  

0  

1  

0  

0
Machine Translated by Google

Capítulo  12 Álgebra  Booleana  y  Circuitos  Combinatorios 1019

15.
xyz  yz  xyz  (yz)  x(yz)  xyz  +  x(yz)
000  00  1  001  00  1  010  00  1   0   0  

011  10  0  100  00  1  101  00  1   0   0  

110  00  1  111  11  0 0   0  

0   0  

1   1  

1   1  

1   1  

0 1

17
x  y xy x  +  xy
0   0   0   0  

0   1   0   0  


1   0   0   1  

1 1 1 1

↑ idéntico ↑

19
xy  x+y  (x  +  y)  xyxy
0  0  0  0  1   1  11  1  0  10  0  0  
1  1  0  1  1   01  0  0  00  0
1  1

↑ idéntico  ↑

21
xy  x+y  (x  +  y)  xyx  +  y
0  0  0   0   1  11  1  0  10  1  0  01  1  0  
1  1  0   1   00  0
1  1 1  

↑ no  idéntico  ↑

23.  si 25.  xyz,  xyz ,  xy  z,  xy  z , x  yz,  x  yz ,  xy  z,  xyz


27.  xy 29.  xy  +  xy
31.  xyz  +  xy  z 33.  xy  z  +  x  yz  +  xyz
35.  xy  +  xy  +  xy 37.  xy
Machine Translated by Google

1020 Soluciones  a  ejercicios  impares

39.  xyz  +  xyz  +  x  yz 41.  x+y  =  x1+1y  =  x( y+y )+(x+x )y  =  xy  
+  xy  +  xy  +  xy  =  (xy  +  xy)  +  xy  
43.  (x  +  y)xy  =  xxy  +  yxy  =  
+  xy  =  xy  +  xy  +  xy
xy  +  x(yy )  =  
xy  +  x0  =  xy
45.  y(x  +  z)  =  yx  +  yz  =  xy  +  yz  
47.  0 =  xy(z  +  z )  +  (x  +  x )yz  
=  xyz  +  xyz  +  xyz  +  x  yz  
49.  1 =  xyz  +  xyz  +  x  yz
51.  0
53. 55.
xy  x↑y  (x↑y)↑(x↑y) xy  x↑y  (x↑y)↓(x↑y)

0  0  0   1   0   0  0  1   1   0  


1  1  0   1   0   0  1  0   1   0  
1  1 1   0   1  1 1   0  
0 1 0 1

57.
X y x  ↑  x y  ↑  y (x  ↑  x)  ↓  (y  ↑  y)

0   0  11  0  1  10  0  0  01  0  1  00  1
0  
1  

59.  xy  +  xy  +  xy

61.
xy  x+yx  ↑  xy  ↑  y  (x  ↑  x)  ↑  (y  ↑  y)

0  0  011  0  0  1  110  1  1  0  101  1  1  1  100  1

↑ idéntico ↑

63.
X y  x+yx  ↓  y (x  ↓  y)  ↓  (x  ↓  y)

0   0  01  0  1  10  1  0  10  1  1  10  1
0  
1  

↑ idéntico ↑

65.  (0,1,1)
Machine Translated by Google

Capítulo  12 Álgebra  Booleana  y  Circuitos  Combinatorios 1021

67.  Como  {↑}  es  funcionalmente  completo,  x  =  x  ↑  x  y  x  +  y  =  (x  ↑  
x)  ↑  ( y  ↑  y),  {+, }  también  es  funcionalmente  completo.

69.  1 71.  0
73.  0 75.  no
77. 79.
X x     x xy  x     yy     x

0   0   0  0  0   0   0  

1 0 1  1  0   1   1  

1  1 1   1  

0 0

↑ ↑

81.
xyz  y     z x     ( y     z)  x     y (x     y)     z

000  00  00  001  11  01  010  11  11  011  00  10  100  01  11  101  10  
10  110  10  00  111  01  01

↑ idéntico ↑
83.  (x  +  y)(x  +  y )  
85.  (x  +  y  +  z)(x  +  y  +  z )(x  +  y  +  z )  
87.  (x+y+z)(x+y+z )(x+y  +z)(x+y  +z )(x  +y+z)(x  +y+z )(x  +y  +z)

Ejercicios  12.3  (pág.  829)

1.  0 3.  1 5.  1 7.  1 9.  0

11 13.  xy  +  xy  +  xy
xy  x  ↑  y
15.  xyz  +  x  yz  +  xyz  +  x  yz  +  xyz
0   0   1  

0   1   1  

1   0   1  

1 1 0
Machine Translated by Google

1022 Soluciones  a  ejercicios  impares

17 19.  cierto
NAND  xyz
21.  falso
000  1  001  1  
010  1  011  1  
100  1  101  1  
110  1  111  0

Ejercicios  12.4  (pág.  837)

1.  x  =  0 3.  x  =  y  =  1 5.  xy 7.  (x  +  y)  y


9.  x  +  ( yz) 11.  (x+y)z  +xz 13.  (xy  +  yz+  zx)xy  z
15.
xy  x+y  (x  +  y) (x  +  y)  y

0   0  01  0  1  10  0  0  10  0  1  10  0
0  
1  

17
xyz  yz  (yz) x  +  (yz)

000  01  1  001  01  1  010  01  1  011  
10  0  100  01  1  101  01  1  110  01  1  
111  10  1

19 X

y x( y′zyz′)

z
Machine Translated by Google

Capítulo  12 Álgebra  Booleana  y  Circuitos  Combinatorios 1023

21 X

23 25
X X'
X
xy

y 29.  1001 31.  1011

27
X
xy

33.  s  =  (x  +  y)(xy)  =  xy  +  xy
  si  =  sci  +  s  ci  =  (xy  +  x  y)c i +  (xy  +  x  y)  ci
+  
+  ((x  
=  xy  c  yo +  x  yc  i   +  
xx  +y  )(x  
xy  +  xyy  
 )ci  
+  
=  xy  c  i xy  +  0)ci  yy )ci  
+  x  yc  i  =  xy  c    (i0  
+   x   
y c  
i   
+   =+
  x  yci  
xy  ++    
xy  c  +  x  yc  i  +  xy  ci  i

35.
X

y s(xy)  (xy)′

cxy

Ejercicios  12.5  (pág.  849)

1.  x 3.  xy 5.  x

7.  xy 9.  xy  +  yz  +  zx  +  xyz 11.  xy


13.  xyz 15.  x(wy  +  wy ) 17.  x
Machine Translated by Google

1024 Soluciones  a  ejercicios  impares

19.  xy  +  xy yy

23.  x 21 x1  _

X 1
25.  xyz  +  xyz  +  xy  z  +  xyz

27.  xyz  +  xy  z  +  x  yz  +  xyz 29.  xy 31.  y

33.  xy  +  yz 35.  z

37.  wxyz  +  wxy  z  +  wx  yz  +  wx  yz  +  wx  yz  +  wxyz  +  w  xyz  +  w  xy  z

39.  wxyz  +  wx  yz  +  wxyz  +  wxyz  +  w  xy  z  +  w  xy  z

yz  yz  yzyz yz  yz  yzyz

wx 1 wx 1

41. wx 43. wx 1

wx wx 1

wx  1 ancho  x 1

45.  z 47.  wyz  +  wy

49.  yz 51.  wx  +  yz

Ejercicios  12.6  (pág.  856)

1.  x  +  yz 3.  y  +  wy 5.  wx  +  xyz  +  wxy

7.  x  +  y 9.  z 11.  xyz  +  wxyz

13.  wx  +  xyz 15.  xyz  +xy  z+xyz  +  xyz

17.  wxyz  +  wx  yz  +  wxyz  +  wxyz  +  wx  yz  +  wx  yz

19.  wxyz  +  wxyz

21.  wxyz  +  wx  yz  +  wx  yz  +  wx  yz

23.  yz  +  yz  +  wx  +  wx

25.  yz  +  wxy  +  xyz  +  wxy  z

27.  xy  z  +  xyz  +  wx  +  yz

Ejercicios  de  repaso  (pág.  859)

1.  si 3.  no 5.  Cuando  n  es  producto  de  primos  distintos.


7.  2 9.  30 11.  (x  +  y)(x  +  y)  =  y
Machine Translated by Google

Capítulo  12 Álgebra  Booleana  y  Circuitos  Combinatorios 1025

13
xyz  x  +  yx  +  y  +  zxyxyzxyzf

000  0  001  0  010   0  11  1  1  11  1  1  10  0   1   1   0  


1  011  1  100  1  101   1  10  0  1  01  0  1  01  0   0   0   0  
1  110  1  111  1 1  00  0  1  00  0 1   0   0  
0   0   0  
1   0   0  
0   0   0  
1   0   0  
0 0 0

15.
xyz  yz  xxyzx  +  x  yz  x  +  yz

000  01  0  001  01  0  010  01  0  011   0   0  

11  1  100  00  0  101  00  0  110  00  0   0   0  

111  10  0 0   0  

1   1  

1   1  

1   1  

1   1  

1 1

↑_  idéntico  _↑

17.  x  yz  +  xy  z  +  xyz 19.  xyz  +  xyz  +  xy  z  +  x  yz
21.  xyz  +  xyz  +  xy  z  +  x  yz 23.  0

25
xy  x  ↓  x  y↓  y (x↓  x)↑  (y↓  y)

0   0  11  0  1  10  1  0  01  1  1  00  1
0  
1  
1

27.  xy  +  xy  +  xy  +  xy 29.  xy  z  +  x  yz  +  xyz
31 X 33.  1110

z xyzx′y′z′
Machine Translated by Google

1026 Soluciones  a  ejercicios  impares

yz  yz  yzyz 37.  x  +  y  39.  wy  +  wy  41.  y
wx  1 1
43.  wx  +  wyz  +  xyz  +  yz
35. wx 1 1
45.  x  +  z  47.  wy  +  xy  z  +  wxyz  49.  wy  +  yz  +  
wx 1 1
xyz  +  xyz  51.  wxyz  +  w  xy  z  +  wx  yz
wx  1 1

53.  wx  yz  +  wxyz  +  wxyz  +  wxyz  +  wxyz  +  wx  yz

Ejercicios  complementarios  (pág.  862)

1.  sí  3.  

prueba  (por  PMI):  Claramente,  x(1)  =  x .  Supongamos  que  x(k)  es  igual  a  x  si  k  es  par
y  x  si  k  es  impar,  donde  k  ≥  1.  Entonces:

X si  k  es  par  
x(k+1)  =  x(k)  =
(x )  de  lo  contrario

x  si  k  es  par
=
x  de  lo  contrario

Así,  el  resultado  se  sigue  por  inducción.

5.  demostración  (por  inducción  fuerte):  el  resultado  es  claramente  verdadero  cuando  n  =  1.
Así  que  suponga  que  es  cierto  para  cualquier  expresión  booleana  con  2,  3,  
4, ...  o  k  variables.  Entonces  (x1x2 ...xkxk+1)  =  [(x1x2 ...xk)xk+1]  =  [(x1x2 ...xk)  
+  xk+1]  =  (x  
1  A
2+  sí  
x   el  
+  x  +
+∙∙∙+  x  ∙∙∙+  xx    +  x  kxk  
k)  +   =  
  k++1.2
1  1
resultado  sigue  por  PMI.

7.  xy  +  (x  +  y)  +  xy 9.  [ xy  +  (x  +  y )  +  x  ( y ) ]

11.  {xy  +  [( y )  1 ]+[x  +  (0 ) ]} 13.  si 15.  no

Apéndice  A

Ejercicios  A.2  (pág.  873)

1.  6 3.  a2  +  b2 5.  0


Machine Translated by Google

Apéndice  A 1027

7.  ab+bc+ca−a2−b2−c2 9.  (a  +  b  +  c)(ab  +  bc  +  ca  −  a2  −  b2  −  c2)

11.  (ad  −  bc)(eh  −  fg) 13.  |AB|  =  (ad  −  bc)(eh  −  fg)  =  |A|∙|B|

15.  |A|∙|A|=|A|∙|AT|=|A  ∙  AT|=|In|  =  1,  entonces  |A|=±1.
x  −   x  −  un
segundo  17.  f(x)  =  f(a)  +  un  −  segundo f(b)  
segundo  ­  un

Ejercicios  A.3  (pág.  881)

1.  f(x  +  y)  =  hacha  +  y  =  hacha  ∙  ay  =  f(x)  ∙  f(y)

3.  Sea  f(x)  =  f(y).  Entonces  ax  =  ay,  entonces  x  =  y.  Así  f(x)  =  f(y)  implica  x  =  y;  entonces  f  
es  inyectiva.

5.  Suponga  que  log  2  =  a/b,  donde  a  y  b  son  números  enteros  primos  relativos  y  a<b.  
Entonces  2  =  10a/b;  es  decir,  2b  =  10a.  Entonces  5a  =  2b−a.  Esto  implica  2|5,  lo  cual  
es  una  contradicción.  Así  log  2  es  un  número  irracional.  7.  pH  =  −  log(3.  76  ×  10−8)  ≈  

7.  4248
9.  0  decibales 11.  70  decibales

13.  10.84006 15.  2,67917  parsecs

Ejercicios  A.4  (pág.  887)

1.  2134 3.  4213 5.  21354 7.  23145

9.  231,  312,  321 11.  2341,  2413,  2431

13.  3421,  4123,  4132 15.  23541,  24135,  24153

17.  12,  21 19.  134 21.  1345 23.  3456

25.  12,  13,  23 27.  12,  13,  14,  23,  24,  34

29.  Siguiente  byte  del  algoritmo  (a1a2  ∙∙∙  a8)

(*  Este  algoritmo  encuentra  el  byte  que  sigue  al  byte  a1a2 ...  a8  en  orden  
lexicográfico.  *)
Empezar  (*algoritmo*)  i  ←  
8
while  bi  =  1  do  (*  continuar  hasta  bi  =  0  *)  comenzar  (*  
while  *)  bi  ←  0  i  ←  i­1  endwhile  bi  ←  0  (*  actualizar  bi  *)

Fin  (*algoritmo*)

31.  Ø,  {1} 33.  Ø,  {1},  {2},  {3},  {1,  2},  {1,  3},  {2,  3},  {1,  2,  3}
Machine Translated by Google

1028 Soluciones  a  ejercicios  impares

Ejercicios  A.5  (pág.  892)

1.  30 3.  210 5.  1680 7.  12,600


9.  30 11.  −  2 13.  168

15.  x2  +  y2  +  z2  +  2xy  −  2yz  −  2zx  
17.  x3  −  y3  +  z3  −  3x2y  +  3x2z  +  3xy2  +  3y2z  +  3xz2  −  3yz2  −  3xyz  
19.  x3  +  8y3  +  z3  +  6x2y  +  3x2z  +  12xy2  +  12y2z  +  3xz2  +  6yz2  +  6xyz
21.  12 23.  2520 25.  3.326.400 27.  kn

29.  12.080.096.000 31.  56 33.  10


35.  455 37.  210

39.  x3  +  8y3  +  z3  −  6x2y  +  3x2z  +  12xy2  −  12y2z  +  3xz2  −  6yz2  −  6xyz
41.  10 43.  165 45.  1.801.800
Machine Translated by Google

Créditos

Imagen  de  portada  (cabeza  de  girasol)  cortesía  de  Runk,  Schoenberger/Grant  
Heilman  Photography,  Inc.

Texto  de  The  Divine  Proportion  de  HE  Huntley  (Mineola,  Nueva  York:  Dover  
Publications,  1970)  reimpreso  por  cortesía  del  editor.

Ilustración  de  una  cafetera  (Figura  1.7)  por  Michael  Crawford,  Nueva  York.

Todas  las  ilustraciones  de  retratos  biográficos  de  Nancy  Crumpton.

1029
Machine Translated by Google

Esta  página  se  ha  dejado  en  blanco  intencionalmente
Machine Translated by Google

Índice

A búsqueda  de  aliento  primero   merge  sort,  313–314,  328–
ABRACADABRA,  378–379,  387– (BFS),  621–622  tipo  de   329  minmax,  252  
388 burbuja,  230–233,  248–249,  317–318   multiplication,  225  next­
Valor  absoluto,  33,  126 complejidades  de,  247–252,  319– combination,  887  next­
Leyes  de  absorción,  83 333,  420–422  conectividad,  471–475,   permutation,  884–885  next­
Cadena  aceptada,  764 481  corrección  de,  224  –237   subset,  96–97  bases  no  decimales  
Estado  de  aceptación,  autómata,  760,   corrección  de  recursivo,  316–319   (base­b  expansion),  197–207  
761,  783 definido,  96  búsqueda  en  profundidad   origen  de  la  palabra ,  96
Ackermann,  Wilhelm,  278  función,   (DFS),  618–621
278
Gráficos  acíclicos,  610–611 Prim's,  628–633  
Véase  también  Dags
producto,  171  
Sumador   recursivo,  307–333  
Dijkstra,  717–723   clasificación  por  selección,  
lleno,  836–837  
forma  normal  disyuntiva  (DNF),  819   233–234  complejidad  
mitad,  835–836
divide  y  vencerás,  228,  327–329   espacial,  247  árbol  de  
Suma  en  
división,  185–191  relación  de   expansión,  616–617  
base  b,  199–201  de   equivalencia,  490 subconjuntos,  102  complejidad  
funciones,  123  de  
temporal,  247–248  clasificación  
funciones  generadoras,  300–
topológica,  501  torre,  309
301
Euclidiano,  191–197,  322–323 Warshall,  477–481
de  matrices,  166  
circuito  euleriano,  561 Alfabeto,  75
regla  (principio),  43,  99,  344–345,  415
Gráfico  euleriano,  559–560   Griego,  
exponenciación,  318–319   entrada  894,  772
Aristas  de  
factorial,  226 Carácter  alfanumérico,  348
adyacencia  y,  520   Fibonacci,  310   Gramática  ambigua,  753–755
gráficos  y,  520   findmax,  251 antepasados,  636
representación  de  lista,  452–453,   floyd,  725 Y
538  matriz,  444–445,  520–522,  
Gilbert's,  728   Booleano,  438  
528,  716 máximo  común  divisor  (mcd),  311   conjunciones  y  uso  de,  5
codicioso,  633  apretón  de  manos,   Y  puerta,  825–826
Cuadrados  adyacentes,  en  mapas  de   308–309 Soportes  angulares,  744
Karnaugh,  843 Relaciones  antisimétricas,  456–458
Álgebra de  Horner,  336 Apelación,  Kenneth,  589
Véase  también  Álgebra  de  Boole   Huffman,  671–675   Argumentos,  38  
origen  de  la  palabra,  96 identificador,  767–768   válidos  e  inválidos,  39–49
ALGOL  (ALGOrítmico clasificación  por  inserción,  236–237 Aristóteles,  1,  2
Idioma),  748 Kruskal's,  616–617,  626–628   Aritmética,  teorema  fundamental  de,  218–
Adición  de   secuencia  legalmente  emparejada,   221
algoritmos,  199–200   106–107  búsqueda  lineal,  227– Media  aritmética,  54
multiplicación  binaria,  203   228,  249–250,  316–317,  319–320,   Secuencia  aritmética,  180
búsqueda  binaria,  228–230,  250– 421–422  fusión,  312–313 Rompecabezas  de  La  vuelta  al  
251,  311–312,  327–328  árbol  de   mundo,  564–565
búsqueda  binaria,  665 Matriz,  predecesora,  720

1031
Machine Translated by Google

1032 Índice

ASCII,  670   Números  binarios,  394–395 Variables  vinculadas,  33


conjunto  de  caracteres,   Operadores  binarios,  12,  31,  806 Algoritmo  de  búsqueda  de  
867  Operador  de  asignación,  7   Predicado  binario,  33–34 respiración  primero  (BFS),  621–622
Instrucción  de  asignación,  7   Relación  binaria,  443–444 puente,  686
Relaciones  de  recurrencia  homogéneas   Algoritmo  de  búsqueda  binaria,   Briggs,  Enrique,  879
lineales  asociadas  con  coeficientes   228–230,  250–251,  311–312,  327–328 Logaritmos  Briggsianos,  879
constantes  (ALHRRWCC),  287,   Algoritmo  de  clasificación  de  burbujas,  230–
293–296  Ley  asociativa  Álgebra   Algoritmo  de  árbol  de  búsqueda  binaria,  665 233,  248–249,  317–318
booleana,  806  de  lógica,  22  de   Árboles  de  búsqueda  binarios,  664–668 Topología  de  bus,  529
conjuntos,  83  Propiedades  asociativas,   Resta  binaria,  203–205 Bytes,  348–349
470 ,  804,  805  Autómatas.  Ver   Árboles  binarios,  639,  646–663
Autómatas  de  estado  finito  (FSA) Números  catalanes  y,  660   C
triangulaciones  entre  paréntesis  y,  659– Rompecabezas  de  repollo,  cabra  y  
660 lobo,  552–554
Binet,  Jacques  Philippe  Marie,  277 Cantor,  Georg,  67,  68  proceso  
Forma  de  Binet  del  número  de  Fibonacci,   de  diagonalización,  141
276–277,  290,  304–305 Trato  de  cartas,  132–133
Complejidad  de  tiempo  de  caso  promedio,   Coeficiente  binomial,  366,  386,  889 Cardinalidad  de  conjuntos,  98–102,  140
248,  420–422  Axiomas  definidos,   Expansión  binomial,  388 Carrol,  Lewis,  44,  45
185  dual,  807 Probabilidad  binomial,  423 llevar,  199
Teorema  del  binomio,  386–399,  888– plano  cartesiano,  88
889 Productos  cartesianos,  87–89
grafos  bipartitos,  526 Catalan,  Eugene  Charles,  108  números,  
B   Paradoja  del  cumpleaños,  182,  412. 108,  388–390,  393,  395–396,  660  
Bachmann,  Paul  Gustav  Heinrich,   problema  de  paréntesis,  394
237,  238  Retroceso,  618   Operaciones  de  bits,  
Backus,  John  W.,  748  Forma   cadena  438,  95 Cayley,  Arthur,  164,  519,  609  fórmula,  
Backus­Naur,  748  Forma  Backus­ Boole,  Jorge,  1,  4,  803 626
Normal  (BNF),  748–751  Árboles   Aplicaciones  del   Función  de  techo,  126–130
equilibrados,  640–643  Paradoja  de   álgebra  booleana,  803   Centro,  árbol,  614
Barber,  42–43  Base  ­b   complemento,  442,  806   Ecuación  característica,  287–288
expansión,  197–207  Cláusula  base,   definido,  806–811  ejemplos,   Función  característica,  131–132
105,  262,  814  Paso  base,  209  Begin,  18   804–806  expresiones,  7,  814– Raíces  características,  288
Bell,  Eric  T.,  397  Números  de  Bell,  397,   817  expresión,  autodual,  863   Rompecabezas  de  tablero  
492  Bernoulli,  Jacob  I.,  212,  422   matrices,  438–443  operadores,   de  ajedrez,  rompecabezas  de  ocho  
desigualdad,  212  ensayos,  422  –425   12,  806  potencia,  441  reglas  de   reinas,  620  rompecabezas  de  cuatro  
Complejidad  de  tiempo  en  el  mejor  de   precedencia,  807  producto,  439– reinas,  619–620  problema  del  recorrido  del  
los  casos,  247  Declaraciones   440,  806  suma,  806  elemento   caballo,  558–560,  567–571
bicondicionales,  14–15  Estimación  Big­ unidad,  806  variables,  2,  813   rompecabezas  de  dos  reinas,  133–134
Oh,  618  Notación  Big­Oh,  237–239   elemento  cero,  806 Niño,  636  
Notación  Big­Omega,  243–245   izquierda  y  derecha,  646
Notación  Big­theta,  245  Función  de   Chomsky,  (Avram)  Noam,  751,  752
biyección,  137–138  Binario  alfabeto,  75  
Expansión  decimal  codificada  en  binario   jerarquía  de  la  gramática,  751
(BCD),  852–854  Dígitos  binarios,  95  Árboles   Funciones  booleanas,  134   Número  cromático,  588
de  expresión  binaria,  655  Multiplicación   forma  normal  conjuntiva  (CNF),  824 Juego  de  tirada  a  la  suerte,  423–424
binaria,  201–203 Problema  circular,  370–372
definido,  813–814   Circuito,  combinatorio,  830–840  minimización  
forma  normal  disyuntiva  (DNF),  817–820   de,  840–850
igualdad,  815–817  integridad  funcional,   Circuitos,  gráfico,  548
820  mintérmino,  817–820 Euleriano,  556
Circuitos,  secuenciales,  830
Clase,  equivalencia,  486–490
NAND  y  NOR,  820–821  definición   intervalo  cerrado,  73
recursiva,  814 Intervalo  cerrado­abierto,  73
Machine Translated by Google

Índice 1033

camino  cerrado,  548 Gráfico  completo  de  n­partitas,  605 Corrección  de  algoritmos  


Cierre Complejidades  de  los  algoritmos,   recursivos,  316–319
Kleene,  739–741   247–252 Programa  correcto,  224–225
reflexivo,  482   Complejidades  de  los  gráficos,  622–623 Correspondencia,  uno  a  uno,  137–
simétrico,  482   Complejidades  de  los  algoritmos   138
transitivo,  475–482 recursivos,  319–333 Conjuntos  contables,  140–141
Problema  de  los  cocos  y  los  monos,  437,   Componentes,  5 Contraejemplo,  53
493 Números  compuestos,  189–190 Principios  de  conteo,  344–351
Sistema  de  código  de  barras,  493 Función  de   Ciclo  
Códigos,  gris,  446–447 composición,  150–153   dígrafos,  698  
Esquema  de  codificación,  Huffman,  670 de  relaciones,  463–464 gráficos,  526
Codominio  de  funciones,  118 Proposiciones  compuestas,  5 hamiltoniano,  565–566  de  un  
Coeficientes Operaciones  informáticas,  con  conjuntos,   camino,  446,  548
binomial,  366,  386,  889   94–98 Permutaciones  cíclicas,  355–356
multinomial,  891 Representaciones  por  computadora  
cofactor,  868 de  gráficos,  538–539  de   D
Colisión,  138–139 relaciones,  449–454 Dags  (gráficos  acíclicos  dirigidos),  707–
Colorear,  gráfico,  586–598 Concatenación,  76  de   715
Columna  vectorial,  matriz,  165 idiomas,  736–739 Bases  de  datos,  464–466  
Combinaciones,  365–375   Conclusión,  9 Campo  de  datos,  451  de  
generación,  383–384,  885–887  con   líneas  concurrentes,  266 Bruijn,  Nicolaas  G.,  702  gráfico,  
repeticiones,  379–381 Probabilidad  condicional,  417–419 703  secuencia,  702–704
Circuitos  combinatorios,  830–840   Declaraciones  condicionales,  
minimización  de,  840–850 definidas,  9 Curvas  de  decaimiento,  875
Teorema  del   Congruencia   Expansión  decimal,  197
binomio  de  combinatoria,  386–399   con  b  módulo  m,  484   Árboles  de  decisión,  676–680
combinaciones,  365–375  definidas,   relación,  484–486 Grado  de  una  región,  581–582
343  desarreglos,  360–365  principios   Regla  de  la  conjunción,  43 Grado  de  vértice,  520,  694–698
fundamentales  de  conteo,  344–351   Conjunciones,  5–7   De  Moivre,  Abraham,  298,  299
principio  generalizado  de  inclusión/ símbolo  de,  5 De  Morgan,  Augusto,  23,  207
exclusión,  399–409   Forma  normal  conjuntiva  (CNF),  824 Descripción  de  la  ley  de  la  lógica  
permutaciones,  351–360  repeticiones,   de  De  Morgan,  22–24  para  
375–386 Dígrafos  conectados   cuantificadores  negadores,  35–36
fuertemente,  698–700   Ley  de  los  conjuntos  de  
débilmente,  705 De  Morgan,  83
Diferencia  común,  180 Gráficos  conectados,  549–554 Eventos  dependientes,  419–420
factor  común,  191 Conectivos,  5 Algoritmo  de  búsqueda  primero  en  
Razón  común,  180 Relación  de  conectividad,  471–475,  481 profundidad  (DFS),  618–621
Enlace  de  comunicación,  2  etapas,  550 Trastornos,  360–365  contando,  
Ley  conmutativa Prueba  de  existencia  constructiva,  52– 404–408
álgebra  booleana,  806  de   53 Derivación,  lenguaje  y,  746–747
lógica,  22  de  conjuntos,  83 Gramática  libre  de  contexto,  751
Lenguaje  independiente  del  contexto,  751–753 Árboles  de  derivación,  743–744
Propiedades  conmutativas,  804,  805 Gramática  sensible  al  contexto,  751 Descartes,  René,  87
Elementos  comparables,  495 Lenguaje  sensible  al  contexto,  751– Desprendimiento,  ley  de,  43
complementos 753 Determinantes,  867–874
Booleano,  442,  806  de   Contingencias,  16 Autómatas  deterministas  de  
grafos,  536  propiedades,   Contradicciones,  16   estado  finito  (DFSA),  
805,  806  relación,  469  de   prueba  por,  51 779–782
conjuntos,  81 Contrapositivo,  11–12  prueba   Diagonal,  matriz,  166
de,  51 diagramas
Grafos  bipartitos  completos,  527 Ley  contrapositiva,  22,  43 Hasse,  496–499  
Gráficos  completos,  524–526 Converso,  11–12 transición,  760,  772  árbol,  
Integridad,   Corrección  de  algoritmos,  224–237 6
funcional,  820 Venn,  72­73
Machine Translated by Google

1034 Índice

Diferencia   Propiedades  de  divisibilidad,  189–197 de  funciones  generadoras,  299–


entre  conjuntos,  80   Algoritmo  de  división,  185–191 300  de  matrices,  166  de  
simétrica,  82 Divisor,  186   relaciones,  455  de  conjuntos,  70  
Diferencial  e  Integral máximo  común  divisor  (mcd),  191 de  palabras,  734–735
Cálculo,  El  (De  Morgan),  23
Doctrina  de  las  posibilidades,  La
Dígrafos  (gráficos  dirigidos)  cerrados,   (De  Moivre),  299 Clase  de  equivalencia,  486–490
698  dags,  707–715  secuencia  de   Dominio   Relaciones  de  equivalencia,  482–484
Bruijn,  702–704  grado  de  vértice,   de  funciones,  118  de   Declaración  de  equivalencia,  483
694–698  características  de,  691–692   relaciones,  443 Circuitos  combinatorios  equivalentes,  840–850
abiertos,  698  relaciones  y,  443–447 Ley  de  dominación  de  
la  lógica,  21  de   Autómatas  de  estado  finito  equivalentes,  764–
conjuntos,  83 768

No  me  importan  las  condiciones,  851–856 Autómatas  de  estado  finito  no  
Números  de  Fibonacci  y,  692– producto  escalar,  493 deterministas  equivalentes,  784–
694  Ganancia  de  n  etapas,   785
ley  de  doble  complementación,  83;
698  rutas,  accesibilidad  y  ciclos,   Ley  de  la  doble  negación,  22 Redes  de  conmutación  equivalentes  25–26
698 Axiomas  duales,  807
Dualidad,  principio  de,  807 Erdös,  Paul,  147  
fuertemente  conectado,  698–700   Dudeney,  Enrique  Ernesto,  40–41 teorema,  146–147
problema  de  la  teleimpresora,  700–702   Duffiniano,  257 Euclides,  191
débilmente  conectado,  705  ponderado,   Estado  de  volcado,  767 Algoritmo  euclidiano,  191–195,  322–323
715–725  Dijkstra,  Edsger  Wybe,  717  
algoritmo,  717–723  Dirac,  Gabriel  Andrew,   Euler,  Leonhard,  53,  72,  393,  516–517  
mi
571  teorema,  570  gráficos  acíclicos   circuito,  556  fórmula,  578–581  
Domingo  de  Pascua,  135,  195
dirigidos.  Consulte  Dags  Bordes  dirigidos,  692   gráficos,  556–564  ruta,  556  función  
Excentricidad,  árbol,  614
Gráficos  dirigidos.  Ver  Digraphs  Directly   phi,  196  teorema,  512
Bordes,  gráfico,  445,  517  
derivable,  746  Pruebas  directas,  50  Dirichlet,  
adyacente,  520  dirigido,  692  
Peter  Gustav  Lejeune,  144,  145  Principio  de  
paralelo,  518
caja  de  Dirichlet.  Consulte  el  principio  del  
casillero  Gráficos  desconectados,  549  
Probabilidad  discreta,  409–427  Conjuntos   Puzzle  ocho  monedas,  678
paridad  par,  765
disjuntos,  71  Disyunciones  definidas,  7–8   Rompecabezas  de  ocho  reinas,  620 Evento(s)  
exclusivo,  8,  18  inclusive,  8  Símbolo   Operaciones  elementales,  gráfico,  583 definido(s),  410  
para,  7  Forma  normal  disyuntiva  (DNF),  817– dependiente  e  independiente,  419–420  
Elementos   mutuamente  excluyentes,  414–
820  Ley  distributiva  Álgebra  booleana,  
806  de  lógica,  22  de  conjuntos,  83  Propiedades   comparables,  495   415  probabilidad  de,  410–414
distributivas,  804,  805  Función  div,  132–134   mayor  y  menor,  500  matriz,  165  
Divide  y  vencerás  algoritmo,  228,  327–329   máximos,  499  mínimos,  499  no   Evas,  Howard,  67,  391
Dividendo,  186 comparables,  495  de  conjuntos,   Medio  excluido,  ley  de,  28
68 Disyunción  exclusiva,  8,  18
Prueba  de  existencia,  52–53
Cuantificadores  existenciales,  32
Elementos  (Euclides),  192 Expansión  
Puntos  suspensivos,  73 base­b,  197–207  
Lengua  vacía,  735 decimal  codificado  en  binario  (BCD),  
Relación  vacía,  455 852–854
Juegos  vacíos,  70 binomial,  388  
Palabra  vacía,  75 decimal,  197
Puntos  finales,  ruta,  546 Valor  esperado,  419–420
Mientras  tanto,  18 Experimento,  409
Igualdad  de   Función  exponencial,  125,  874–877
expresiones  booleanas,  815–
817  de  funciones,  123 Expresiones,
Booleano,  7,  814–817,  863
Machine Translated by Google

Índice 1035

F bien  formado,  107   propiedades  de,  136–143  
Factoriales,  108,  226,  308 Zeller's,  492   cuadrático,  125  rango  de,  
Falacia,  39 FORTRAN,  748   120–121  definición  recursiva  
árbol  genealógico,  611 Problema  de  cuatro  colores,  589– de,  262–278  suma  de,  123  sobreyección,  
Fermat,  Pierre­Simon  de,  4,  5 595  Rompecabezas  de  cuatro  reinas,   137  transición,  761,  772,  783
Último  teorema,  4,  145,  511   619–620  Variables  libres,  33  Viernes  
número,  338  primos,  53 13,  485–486  Friedberg,  SH,  325  
Sumador  completo,  836–  837  Árbol   Teorema  fundamental  de  la  
Fibonacci,  Leonardo,  268,  269 completo,  639  Completitud  funcional,   aritmética,  218–221
números  de  Fibonacci,  269–271   820  Funciones  valor  absoluto,  126   Decisiones  confusas,  28–29
algoritmo,  310 Ackermann,  278  aplicaciones,  117– Lógica  difusa,  26–28
Binet  forma  de,  276–277,  290,  305– 118  biyección,  137–138  Boolean,   Conjuntos  borrosos,  91–92
305  complejidad  de  la  versión   134,  813–824  cardinalidad,  140   Subconjuntos  borrosos,  91–92
iterativa,  320–321 techo,  126–130  característica,  131–
132  codominio  de,  118  composición,   G  
dígrafos  y,  692–694  parafinas   150–153  definida,  118–120,  448   Game  tree,  637  
y,  523–524  permutaciones   div,  132–134  dominio  de,  118   Gates  AND,  825–
y,  356–358  recursivo,  310 igualdad  de,  123  phi  de  Euler,  196   826  definido,  824  
exponencial,  125,  874–877  piso,   logic,  824–829  
Árboles  de  Fibonacci,  646–647 126–130  generación,  298–307,   NAND,  828  NOR,  
Estado  final,  autómata,  761,  783 383–  384  gráficos,  122–123  mayor   828–829  NOT,  
Sucesiones  finitas,  158 entero,  127  crecimiento  de,  237– 827  OR,  826  XOR,  
Conjuntos  finitos,  73– 247  hashing,  138–139  identidad,   18  Gauss,  Karl  
74  número  de  particiones  de,  490 136  inyección,  136–137  inversa,   Friedrich,  145,  
Aplicaciones  de  autómatas  de   153–155  menor  entero,  127  lineal,   222,  484  Inclusión/
estado  finito  (FSA),  759–761   125  logarítmico,  125–126,  877–879 exclusión  generalizada  (GIEP),  399–409  
definidas,  761–764  deterministas,   Principio  del  casillero  
779–782  equivalentes,  764–768,   generalizado,  147–149  Combinaciones  
784–785  lenguajes  aceptados,  787– generadoras,  383–384,  885–887  
792  no  deterministas,  782–787   Funciones  generadoras,  298–307,  383–
Máquinas  de  estados  finitos  (FSM),   384  Permutaciones  generadoras,  
771  aplicaciones,  733  características,   352–358,  883  Media  geométrica,  54  
772–778  Mealy,  772  Moore,   Secuencia  geométrica,  180  Gilbert,  
799–800  simplemente  mínimo,  800   EN,  728  algoritmo,  728  Goldbach,  
retardo  de  unidad,  774  Código  de   Christian,  4,  5  conjetura,  4  
longitud  fija,  670  Función  de  piso,   Proporción  áurea,  179,  276  Gráfico  
126–130  Floyd,  Robert  W.,  725,  726   elegante,  604  Árbol  elegante,  686  
algoritmo,  725  Fútbol  pools,  426–427   Gramáticas,  743–759
Forest,  611  For  loop,  233,  381–382  
Lenguajes  formales.  Ver  Idiomas  Serie  
de  potencias  formales,  299  Fórmulas  
de  Cayley,  626  de  Euler,  578–581  punto   McCarthy's  91,  275  mod,  
medio,  145  molecular,  519   132–134  siguiente  
recursividad,  262  estructural,  519–520 estado,  761,  783  uno  a  
uno,  136–137  
correspondencia  uno  a  uno,  137–
138
sobre,  137   ambiguo,  753–755
orden,  más  común,  239–241  salida,   Jerarquía  de  Chomsky,  751  
772  definición  por  partes,  121–123   sensible  al  contexto,  independiente  del  
principio  del  casillero  y,  144–150   contexto  y  regular,  751
polinomio,  125  producto  de,  123 Escriba  0,  751
Tipo  1,  751
Tipo  2,  751
Tipo  3,  751
Machine Translated by Google

1036 Índice

Funciones  gráficas,  122–123 H   Propiedades  de  identidad,  805  


gráficos Haken,  Wolfang,  589  Half­ Instrucción  si­y­solo­si,  14–15  Instrucción  
Véase  también  Dígrafos  (gráficos   adder,  835–836  Half­life,   si,  Expresiones  booleanas  en,  7  
dirigidos)  acíclico,  610–611   881  HAM  (Hampton  Court)   Composición  de  instrucciones  si­
adyacencia  e  incidencia,  520  bipartito,   game,  572–573  Hamilton,  William  Rowan,   entonces­si  no,  152  Definición  por  partes,  
526  colorante,  586–598  complemento   564,  565  Around  the  World   122  Instrucción  si­entonces,  753–754  
de,  536  completo,  524–526   puzzle,  564–565  graph,  565–567  Hamming ,   Imagen,  función,  119  Predecesor  
bipartito  completo,  527  n­partito   Richard  W.,  180  distancia,  179,   inmediato,  692  Sucesor  inmediato,  692  
completo,  605  complejidad,  622–623   446  Problema  de  apretón  de  manos,   Implicación  ley  de  conversión,  22  
computadora  representaciones  de,   264–265,  280,  308–309,  431,  524–
Implicaciones,  9–11  recíproco,  inverso  y  
538–539 526,  533 contrapositivo  de,  11–12  Incidencia,  
gráfico,  520  Principio  de  inclusión/
exclusión,  99–101,  345,  414–415  
generalizado,  399–409  Disyunción  
inclusiva,  8  Ingrado,  vértice,  694  
Eventos  independientes,  419–420  Índice,  
conectado,  549–554  ciclo,   Media  armónica,  351  Número   suma,  158,  160–161  Conjuntos  indexados,  
526  de  Bruijn,  703  grado   armónico,  332  Función  hash,   86  Pruebas  indirectas,  51  
de  vértice,  520   138–139  Hasse,  Helmut,  496,  
Inducción.  Véase  Inducción  matemática  
desconectado,  549  bordes,   497  diagramas,  496–499  Montón,  
Paso  de  inducción,  209  Definiciones  
445,  517 669  Heapsort,  669  Heawood,  
inductivas,  263  Hipótesis  inductiva,  209  
Percy  John,  589  Altura,  árbol,  
Desigualdad,  inducción  matemática  y,  212  
Eulerian,  556–564   637  Árboles  heterogéneos,  655  
Reglas  de  inferencia,  43  Forma  inferencial,  
agraciado,  604 Número  hexagonal,  284  Hilbert,  
38  Secuencias  infinitas,  158  Conjuntos  
hamiltoniano,  565–567   David,  74  hotel  paradoxes,  74–75  
infinitos,  73–74  Símbolo  de  infinito,  74  
homeomorfo,  583  isomorfo,   Hoare,  C.  Anthony  R.,  316  Gráficos   Notación  infija,  653  Estado  inicial  autómata  
541–544  nodos,  445,  517  no   homeomorfos,  583  Árboles   de  estado  finito,  760,  761,  783  
plano,  582  caminos,  446–447,   homogéneos,  664  Horner,  William  
máquina  de  estado  finito,  772  Vértice  inicial,  
546–547 G.,  207  algoritmo,  336  método,  
692  Función  de  inyección,  136–137  Recorrido  
207  Huffman,  David  Albert,  671  
en  orden,  árbol,  647–648  Alfabeto  de  
Petersen,  530–531  plana,   algoritmo,  671–675  esquema  de  
entrada,  772  Símbolos  de  entrada,  761,  783  
576–586  representación   codificación,  670  árboles,  670–675  
Algoritmo  de  clasificación  por  inserción,  
plana,  576  regular,  536   Huygens,  Christian,  419  Hipótesis,  9  
236–  237  Puntos  interiores,  477  
autocomplementaria,  605   inductivo,  209
Vértice  interno,  636  Número  de  libro  estándar  
simple,  518–520  cuadrada,  
internacional  (ISBN),  509
575  sub,  522–523  subyacente,  
702,  705  no  dirigida,  517  unión  
de,  532–533  vértice ,  445,  517  
ponderado,  527–528  rueda,  526,  
623

yo

Ibn  Ezra,  rabino,  366
gris,  franco,  447 Cálculo  icosiano,  565
Códigos  grises,  446–447,  566 Ley  idempotente  de  
Máximo  común  divisor  (mcd),  191  recursivo,   la  lógica,  21  de  
311 conjuntos,  83
Función  de  identidad,  136
Mayor  elemento,  500 ley  de  identidad
mayor  función  entera,  127 álgebra  booleana,  806  de  
Algoritmo  codicioso,  633 lógica,  21  de  conjuntos,  83
Curvas  de  crecimiento,  874
Guthrie,  Francisco,  589 Matrices  de  identidad,  166
Machine Translated by Google

Índice 1037

Relaciones  de   Kronecker,  Leopoldo,  68,  162 Leyes,  de  


intersecciones,  462–463   delta  de  Kronecker,  162 absorción  de  
de  conjuntos,  78–80 Kruskal,  José  Bernardo,  616 conjuntos,  83  
Intervalos,  73 algoritmo  para  árbol  de  expansión   asociativa,  83  conmutativa,  83
Argumentos  inválidos,  39–49 mínimo,  626–628  algoritmo  para   De  Morgan,  83  
Isomorfismo   árbol  de  expansión,  616–617 distributivo,  83  
invariante(s),  542–544   dominación,  83  doble  
bucles,  225–227 Kuratowski,  Kazimierz,  584  teorema,   complementación,  83  idempotente,  
Inversa/invertible,  11–12   583 83  identidad,  83  inversa,  83
funciones,  153–155  de  una  
matriz,  174  relación,  469 L  
Lagrange,  Joseph  Louis,  159   hoja,  636
ley  inversa Lambda,  uso  de,  75  Lamé,  Gabriel,   año  bisiesto,  130
de  lógica,  21   325  teorema,  323–324  Landau,   Elemento  mínimo,  500
de  conjuntos,  83 Edmund,  237,  238  símbolo,  237   función  de  mínimo  entero,  127
Descapotable,  153 Reconocedores  de  lenguaje.  
Hijo  izquierdo,  646
Investigación  de  las  Leyes  de Consulte  Lenguajes  de  autómatas  
Subárbol  izquierdo,  646
Pensamiento,  An  (Boole),  1,  4,   de  estado  finito  aceptados­
Leibniz,  Gottfried  Wilhelm,  1,  3,  72,  867
803 reconocidos,  764,  787–792  
Relación  irreflexiva,  461  Isaka,   Forma  Backus­Normal  (BNF),  748–
Longitud  
Satoru,  26,  27  Ejemplo  de   751  concatenación  de,  736–739  
de  un  camino,  446,  546  
Island  of  Knights  and  Knaves,  12–14,   sensible  al  contexto,  
de  una  palabra,  75
39–40  Vértice  aislado,  520   independiente  del  contexto  y  regular,  
751–753  derivación  y,  746  –746   Nivel,  árbol,  637,  638
Gráficos  isomorfos,  541–544  Árboles  
Orden  lexicográfico,  495–496,  883–
isomorfos,  645  Isomorfismo  definido,   vacío,  735  igualdad  de  palabras,  734–
885
541  invariantes,  542–544  Método  de   735  gramáticas,  743–759  gramáticas,  
Combinación  lineal,  194
iteración ,  226,  279–282,  320–321   ambiguo,  753–755  cierre  de  
función  lineal,  125
Iverson,  Kenneth  E.,  127 Kleene,  739–741  reconocido  por  
Relaciones  de  recurrencia  homogénea  
autómatas  deterministas  de  estado  
lineal  con  coeficientes  constantes  
finito,  779–782  de  conjuntos,  75  
(LHRRWCC),  287–293
Laplace,  Pierre­Simon ,  410  Lattice­
walking,  377–378  Laws,  of  logic  
Conjuntos  linealmente  ordenados,  495
asociative,  22  conmutative,  22  
j contrapositive,  22,  43  De  Morgan's,  22– Relaciones  de  recurrencia  
lineales  no  homogéneas  con  
Java,  735 24,  35–36  desapego,  43  distributive,  
22  domination,  21  double  negation,   coeficientes  constantes
Unirse,  booleano,  439
22  ejemplos  usando,  24–25  medio   (LNHRRWCC),  293–298
k excluido,  28  idempotente,  21  identidad,  21   Palpación  lineal,  139
Kaplansky,  Irving,  285,  290 conversión  de  implicación,  22  inversa,  21   Algoritmo  de  búsqueda  lineal,  227–228,  
Karnaguth,  Mauricio,  842,  843 reductio  ad  absurdum,  22 249–250,  316–317,  319–320,  421
mapas,  842–850
Keno,  420   campo  vinculado,  451
al­Khowarizmi,  Abu­Abdalá Listas  vinculadas,  449–450
Muhammed  ibn  Musa,  96 Método  de  listado,  68–69
Kirchoff,  Gustav  Robert,  609,  610 literal,  817
Kleene,  Stephen  Cole,  cierre  739,   Redes  de  área  local  (LAN),  529
operador  739–741,  739 Función  logarítmica,  125–126,  877–879

Problema  de  la  gira  de  Knight,  558–560,   logaritmos
567–570 Briggsian,  879  
Knuth,  Donald  Ervin,  244 común,  879  
Rompecabezas  del  puente  de   natural,  879
Königsberg,  516–517 Lógica
Kosko,  Bart,  26,  27 Véase  también  Leyes,  de  
Kowa,  Seki,  867 argumentos  lógicos,  38–49.
Machine Translated by Google

1038 Índice

Lógica  (continuación)   multiplicación,  167–172   principio,  345–347,  418–419  escalar,  


comparada  con  operaciones  de   negativa,  167  accesibilidad,   167–170  desplazamiento  y  binario,  
conjuntos,  82–86  definido,  1   699  submatrices  de,  391–393   201–203
puertas,  824–829  predicado,  36   resta,  167  suma,  166  simétrica,   Eventos  mutuamente  
métodos  de  prueba,  49–56  proposicional,   174  telecomunicaciones/redes   excluyentes,  414–415  
36  proposiciones,  2–20  cuantificadores,   y,  528–532  transposición,  175   tareas,  344
32–38  tablas,  813–814,  825–  827   triangular  superior,  874  
variables,  2 adyacencia  ponderada,  528,  716  cero,   norte

166  Maurocylus,  Francesco,  207   NAND  (no  y),  30,  820–821
Elemento  máximo,  499  Maxterm,  824   Puerta  NAND,  828
McCarthy,  John,  275  91  función,  275   Napier,  John,  877  
McCulloch,  Warren  S.,  733  Mealy,   predicado  n­ario,  33
Equivalencia  lógica,  20–32   George  H.,  772  máquinas,  772  Media   Naur,  Peter,  748  n­
definida,  20  redes  de   aritmética,  54  geométrica,  54  armónica,   cubo,  566
conmutación  equivalentes,  25–26 351  Meet,  Booleano,  439  Miembros  de   Negación,  8–9
conjuntos,  68  Ruedas  de  memoria,  701– Negativo  de  matrices,  167
decisiones  confusas,  28–29   702  Algoritmo  de  fusión,  312–313   Multiplicación  anidada,  207
lógica  confusa,  26–28 Algoritmo  de  clasificación  de  fusión,   Redes,  528–530
Lógica  del  azar,  The  (Venn),  72 313–314,  328–329  Mersenne,  Marin,  56   identidad  de  Newton,  398;
Bucles   primos,  56  Fórmula  de  punto  medio,  145   Función  de  estado  siguiente,  761,  783
dígrafos/gráficos,  446   Elemento  mínimo,  499  Árbol  de   Algoritmo  del  siguiente  subconjunto,  96–97
invariantes,  225–227 expansión  mínimo ,  626–634  Minor,  868   puntero  nulo,  451
Límite  inferior,  sumatoria,  158 Minterm,  817–820  m­nary  tree,  639– Nodos,  445,  450–451,  517,  692
Lucas,  François­Edouard­Anatole,  273,  274   640  complete,  645  Mod  function,  132– Elementos  no  comparables,  495
número,  273,  524,  623  teorema,  512 134  Mod  operator,  485  Modulus,  484   Prueba  de  existencia  no  constructiva,  52,  
Molecular  formula,  519  Moore,  Edward   53
Forrest,  799,  800  machine,  799–800   Bases  no  decimales,  197–207
Lukasiewicz,  enero,  62,  653,  654,  659 Coeficiente  multinomial,  891  Teorema   Triángulo  de  Pascal  y,  390–391
multinomial,  890–892  Multiplicación  en   Autómatas  de  estado  finito  no  
base  b,  201  de  funciones  generatrices,   deterministas  (NDFSA),  782–
300–301  de  matrices,  1  67–172   787
Máquinas  M.  Ver  Máquinas  de   anidado,  207 Símbolo  no  terminal,  744,  746
estados  finitos  Constante   NOR  (no  o),  30,  820–821
mágica,  222  Cuadrado  mágico,   Puerta  NOR,  828–829
222  Mapas,  Karnaguth,  842–850   Notaciones  
Juego  Master  Mind,  430  Análisis   big­oh,  237–239  big­
matemático  de  la  lógica  (Boole),  4   omega,  243–245  big­
Inducción  matemática theta,  245  infix,  653

Polaco,  653  
definido,  207   posfijo,  653  
descripción  de,  207–224   prefijo,  653  
versión  fuerte  de,  218  versión   polaco  invertido,  653  
débil  de,  208–209 generador  de  conjuntos,  
Sistema  matemático,  804 69  suma,  158–161
Adición  de  matriz   NOT  gate,  827  n­
(matrices),  166   stage  win,  698  n­
adyacencia,  444–445,  520–522 tuple,  86
Booleano,  438–443   Conjuntos  nulos,  70
definido,  165   Palabra  nula,  75
descubrimiento  de,   Números
164  igualdad  de,  166   Bell,  397,  492  
identidad,  166  inversa,   binario,  394–395
174  invertible,  174 Catalán,  108,  388–390,  393,  395–
396,  660
Machine Translated by Google

Índice 1039

cromático,  588   Resultado,  409,  410 Petersen,  Julius,  530,  532  gráficos,  


compuesto,  189–190 Grado  exterior,  vértice,  694 530–531
Fermat,  338 Función  de  salida,  772 Gramática  de  estructura  de  frase,  746  
Fibonacci,  269–271,  276–277, Símbolos  de  salida,  772 Definición  por  partes,  121–123  Descripción  
290,  304–305,  310,  356–358,  523– del  principio  de  casillero  de,  144–150  
524,  692–694  juego,  420  armónico,   PAG
generalizado,  147–149  Pitts,  Walter,  
332  hexagonal,  284 Palíndromo,  108,  741 733  Problema  de  pizza,  368  Gráficos  
Paradoja,  4   planos,  576–586  Representación  plana,  
cumpleaños,  182,  412 576  Campo  de  puntero,  451  Notación  
Lucas,  273,  524,  623   Russell,  42–43,  69 polaca ,  653  invertido,  653  Polya,  G.,  378  
perfecta,  124  pentagonal,   Bordes  paralelos,  518 Número  poligonal,  216–218  Polígonos,  
284  poligonal,  216–218   Padre,  636 triangulación  de  convexo,  275
prima,  189–190  piramidal   Paréntesis,  emparejados  legalmente,  
cuadrada,  285 106–107
problema  de  paréntesis,
Stirling,  278,  375,  490   catalán,  394
tetraédrico,  285,  382   Triangulaciones  entre  paréntesis,  659–
triangular,  216,  285,  382 660
Paridad,  433   Función  polinomial,  125  Poset.  
O ­máquina  de  cheques,  765   Consulte  Conjuntos  parcialmente  ordenados  
paridad  impar,  765 pares,  765  impares,  765 Notación  de  posfijo,  653  Recorrido  de  
complemento  a  uno,  203 posorden,  árbol,  648  Potencia  booleana,  
Función  de  correspondencia  uno  a   árboles  de  análisis,  745 441  Cadena  de  orden,  569  Ciclo  de  orden,  
uno,  137–138 análisis,  745 576  Conjuntos,  72–73  Relación  de  
Función  uno  a  uno,  136–137 Regla  de  descomposición  en  fracciones   precedencia,  459  Reglas  de  precedencia,  
sobre  la  función,  137 parciales,  301–306 807  Matriz  de  predecesores,  720  Función  
Intervalo  abierto­cerrado,  73 Conjuntos  parcialmente  ordenados   de  predecesor  (PRED) ,  142  Lógica  de  
intervalo  abierto,  73 (poset),  494–503 predicados,  36  Predicados,  33–34  Prefijo,  
camino  abierto,  548 Pedidos  parciales,  493–506 735  Código  de  prefijo,  674  Notación  de  
Operadores Particiones  de  conjuntos  finitos,  490 prefijo,  653  Premisa,  9  Preorden  transversal,  
Booleano,  12,  806 Particiones  de  conjuntos,  89–91 árbol,  647  Prim,  Robert  Clay,  628,  629  
Kleene,  739   Pascal,  Blaise,  370,  371   algoritmo,  628–633  Primer  círculo  de  orden,  
relacional,  658–659 identidad,  370  triángulo,  386– 551  –552  Números  primos  divisibilidad  y,  189–
O 391 190  Fermat,  53  Mersenne,  56  gemelos,  195  
Booleano,  438   Pascal  (lenguaje  informático),  71 Wilson,  512  Principia  Mathematica  (Russell  y  
exclusivo,  8   Rutas   Whitehead),  42  Principio  de  dualidad,  807  
inclusive,  8   cerradas,  548   Principio  de  inclusión/exclusión,  99–101,  345  
símbolo  de,  7 en  dígrafos,  698  en   Principio  de  matemática  inducción  (PMI).  Ver  
ORDEN,  136 dígrafos  y  relaciones,  446–447 inducción  matemática
par  ordenado,  86
Arboles  enraizados  ordenados,  638 puntos  finales,  546
Conjuntos  ordenados,   Euleriano,  556
86  linealmente,   Hamiltoniano,  565–566  puntos  
495  parciales  y  totales,  493–506 interiores,  477  de  longitud,  
Funciones  de  orden,  más  comunes,  239– 446,  546  abiertos,  548  más  
241 cortos,  717–723  simples,  546,  
Ordenamientos   548
lexicográficos,  495–496,  883–885  lineal,  
495  parcial  y  total,  493–506 número  pentagonal,  284
Pentagramas,  519
orden  de  precedencia,  15 número  perfecto,  124
número  ordinal,  121 Permutaciones,  351–360  
Mineral,  Oystein,  571,  589   cíclicas,  355–356  
teorema,  570 generadoras,  883  con  
O  puerta,  826 repeticiones,  376–379
Machine Translated by Google

1040 Índice

Principios  de  contar,  344–351 Rango   equivalencia,  482–493  


principios  de  la  lógica  empírica, de  funciones,  120–121  de   intersección,  462–463  
El  (Venn),  72 relación,  443  r­combinación,   inversa,  469  irreflexiva,  461  
Producto 366–367 operaciones  sobre,  461–
Booleano,  439–440,  806 Accesibilidad,  698   471  órdenes  parciales  y  
Cartesiano,  87–89   matriz,  699 totales,  493–506  caminos,  446–447  
punto,  493  de   Cadena  reconocida,  764 precedencia,  459  propiedades  
funciones,  123  de   Relación  de  recurrencia,  262   de,  454–461  recursiva,  466–468  
matrices,  170  símbolo,   resolución,  278–298 reflexiva ,  455–456  simétrico,  456–
163  de  dos  funciones,   Algoritmos  recursivos   459  transitivo,  459  cierre  transitivo,  
242–243 (recursivos),  307–333   475–482  unión,  462–463
Regla  de  producción,  744–745,  746 cláusula,  105,  262,  814  
Programa,  correcto,  224–225 definidos,  261  definición  
Prueba  por  casos,  52; de  expresiones  booleanas,  
Pruebas 814  definición  de  
por  casos,  52   funciones,  262–278  definición  de  
por  contradicción,  51  por   conjuntos,  104–109  desarreglos,  362–365   Complemento  relativo  de  conjuntos,  80
contrapositivo,  51   fórmula,  262  funciones  generadoras,  298– Relativamente  primo,  194­195
contraejemplo,  53  directo,   307  relaciones  de  recurrencia  lineales   resto,  186
50  existencia,  52–53   homogéneas  con  coeficientes  constantes,   Notación  polaca  invertida  (RPN),  653
indirecto,  51  trivial,  50   287–293  relaciones  de  recurrencia  
vacío,  49–50  Factor  propio,   lineales  no  homogéneas  con   hijo  derecho,  646
124,  189  Subconjunto   coeficientes  constantes,  293–298   Subárbol  derecho,  646
propio,  70  Lógica   relaciones,  466–468  resolución  por   Topología  en  anillo,  529
proposicional,  36  Proposiciones,   iteración,  278–286 Raíz,  dag,  731
2–32  formas  compuestas,  5   Árboles  enraizados,  635–646  
simples,  5  proposicionales. ordenados,  638
Problema  del  tambor  giratorio,  701
Torneos  de  todos  contra  todos,  526
Ley  de  reducción  al  absurdo,  22 Vector  fila,  matriz,  165  
Cierre  reflexivo,  482 permutación  r,  352
Ver  formularios  de  estado  de  cuenta Propiedad  reflexiva,  143. cubo  de  rubik,  882
Relación  reflexiva,  455–456 Russell,  Bertrand,  42,  68  
q Región,  grado  de  a,  581–582 paradoja,  42–43,  69
función  cuadrática,  125 Gramática  
Cuantificadores   regular,  751   S
definidos,  32   gráfico,  536   Espacio  muestral,  409,  410
existencial,  32   lenguaje,  751–753 Multiplicación  escalar,  167–170
cómo  corregir  simbólicamente,  32–33   Cadena  rechazada,  764 Algoritmos  de  búsqueda,  227–230
negación  de,  35–36  valores  de  verdad  de,   Operadores  relacionales,  658–659 Selecciones,  379
33–35  universal,  32 Relaciones   Clasificación  por  selección,  233–234
antisimétricas,  456–458   Gráfico  autocomplementario,  605
Quarternions,  sistema  de,  565 aplicaciones,  437  binarias,  443– Expresiones  booleanas  autoduales,  863
Tablero  de  ajedrez  reina   444 Semántica,  743
rompecabezas  ocho,  620   Matrices  booleanas,  438–443   Secuencias  
cuatro,  619–620  dos,  133–134 cierre,  475–482  complemento,  469   definidas,  157  
composición,  463–464   finito  e  infinito,  158  notación  
cola,  621 representaciones  informáticas  de,   de  suma  y,  158–161  términos  de,  
BÁSICO  RÁPIDO,  737 449–454  conectividad,  471–475   157
Clasificación  rápida,  316 congruencia,  484–486  dígrafos  
cociente,  186 y,  443–449  vacío,  455  igualdad,  455 Circuitos  secuenciales,  830
Notación  de  constructor  de  conjuntos,  69
R Conjuntos

Conejos,  calculando  tasas  reproductivas   Véase  también  Leyes,  de  conjuntos;  
de,  268–269,  286 Cardinalidad  de  subconjuntos,  98–104,  140
Machine Translated by Google

Índice 1041

Productos  cartesianos,  87–89   Símbolo  de  inicio,  744,  746   T
comparados  con  operaciones  lógicas,  82– Declaraciones,  definidas,   Mesas
86  conceptos,  67–78  contables,  140– autómata  de  2–3  estados,   Véase  también  Lógica  de  
141  contablemente  infinitos,  140–141   760,  761,  767,  783  para   tablas  de  verdad,  813–814,  825–
definidos,  68–69  disjuntos,  71  vacíos,  70   máquina  de  estados  finitos,  772  Stein,   827  transición,  761,  772
iguales,  70  finitos  e  infinitos,  73–  74  difusa,   Sherma  K.,  700  Stemple,  J.,  589  Stifel,   función  tau,  350
91–92  indexada,  86  ordenada  linealmente,   Michel,  366  Stirling,  James,  278  números   Tautología,  16
495  nula,  70  operaciones,  78–94   del  segundo  tipo,  278,  375,  490  Cadenas   Problema  de  teleimpresora,  700–702
operaciones  (computadora),  94–98   aceptadas/reconocidas,  764  rechazadas,   Término,  secuencia,  157
ordenada,  86  parcialmente  ordenada,  494   764  Dígrafos  fuertemente  conectados,  698– Cláusula  terminal,  105,  262
particiones,  89–91  potencia,  72–73  definida   700  Fuertemente  orientables,  731   Símbolo  de  terminal,  743,  746
recursivamente,  104  –109  incontable,  141   Orientación  fuerte,  731  Fórmula   Vértice  terminal,  636,  692
universal,  70–71  Teoría  de  conjuntos,  67,   estructural,  519–520  Subgrafos,  522–523   árbol  ternario,  639
68  Shannon,  Claude  Elwood,  803,  804   Submatrices  de  amatrix,  391–393   Palabra  ternaria,  78,  349
Sheffer,  Henry  M.,  31  Trazo  de  Sheffer,  31– Subconjuntos  definidos,  69  difusa,  91– Números  tetraédricos,  285,  382
32  Desplazamiento  y  multiplicación  binaria,   92  propia,  70  Regla  de  sustitución,  744– Teorema,  49
201–203  Camino  más  corto,  717–  723   745  Resta  binaria,  203–205  de   Véase  también  bajo  el  nombre  de
Hermanos,  636  SIM,  531–532  Simmons,   matrices,  167  Subárboles,  636  izquierda  y   Théorie  Analytique  des
Gustavus  J.,  531  Gráficos  simples,  518– derecha,  646  Elevación  sucesiva  al   Probabilités  (Laplace),  410
520  Unión  de,  532–533  Camino  simple,   cuadrado,  331  Función  sucesora  (SUCC ),   Rompecabezas  Three  Houses­Utilities,  
546,  548  Proposiciones  simples,  5  Regla   577–578
142  Sufijo,  735  Suma,  806  de  funciones,  
de  simplificación,  43  Simplemente  mínimo,   Lógica  de  tres  valores,  62
123  de  matrices,  166  de  dos  funciones,  241–
800  Sink,  694  Smullyan ,  Raymond,  39–40   242  Notación  de  suma,  158–161  Función  de   Complejidad  del  tiempo,  247–248
Clasificación  de  algoritmos,  230–234   índice  topológico,  523
sobreyección,  137  Sobreyecciones,  
montón,  669  topológico,  500–503  torneo,  669   Clasificación  topológica,  500–503
conteo,  402–404  Redes  de  conmutación,  
Fuente,  694  Complejidad  espacial,  247  Árboles   Bus  de  
16–17  equivalente,  25  –26  Sylvester,  
de  expansión,  614–634  Gráfico  cuadrado,  575   topología,  
James  Joseph,  164  Lógica  simbólica  (Carroll),  
Número  piramidal  cuadrado,  285  Estrella  de   anillo  529,  
44–45  Lógica  simbólica  (Venn),  72  Cierre  
David,  549  Topología  de  anillo  en  estrella,  529– estrella  529,  
simétrico,  482  Diferencia  simétrica,  82  
530  Topología  de  estrella,  528 anillo  en  estrella  528,  529–530
Simétrica  de  una  matriz,  174  Relaciones  
Toro,  847
simétricas,  456–459  Estructuras  sintácticas  
Pedidos  totales,  493–506
(Chomsky),  752  Sintaxis,  743,  745
Clasificación  de  torneo,  669
Torneos,  todos  contra  todos,  526
Torre  de  Brahma,  265,  281–282,  309–310,  
321
Torre  de  Hanoi,  265
senderos,  422
Traité  de  Méchanique  Céleste
(Laplace),  410
Diagrama  de  transición,  760,  772
Función  de  transición,  761,  772,  783
Mesa  de  transición,  761,  772
Relación  de  clausura  transitiva,  475–
482
Propiedad  transitiva,  55,  78
Relación  transitiva,  459
Transpuesta  de  una  matriz,  174
Estado  trampa,  767
Problema  del  vendedor  ambulante,  571–
573
Recorridos,  árbol,  647–653  en  
orden,  647–648  en  orden  
posterior,  648  en  orden  previo,  
647
Machine Translated by Google

1042 Índice

Tratado  de  ecuaciones  diferenciales para  conjunciones,  6–7   interior,  478  


(Boole),  4 para  disyunciones  (inclusive),  8  para   interno,  636  
Tratado  sobre  el  cálculo  de  lo  finito implicaciones,  10–11  para  Island  of   aislado,  520  
Diferencias  (Boole),  4 Knights  y terminal,  636,  692
Árboles   Ejemplo  de  Knaves,  12–14   Von  Ettinghausen,  Andreas,  366
gráficos  acíclicos,  610–611   para  negación,  9 Von  Segner,  Johan  Andreas,  393
ancestros,  636  aplicaciones,   Valores  de  verdad,  3–5
609  equilibrado,  640–643   Doce  Días  de  Navidad,  221,  373,  431 W
binario,  639,  646–663,  659– Warshall,  Stephen,  algoritmo  
661  expresión  binaria,  655  búsqueda   primos  gemelos,  195 477,  477–481
binaria,  664–668  centro,  614  niño,  636,   Rompecabezas  de  dos  reinas,  133–134 Dígrafos  débilmente  conectados,  705
646  decisión,  676  –680  derivación,  743– complemento  a  dos,  203 Sitios  web,  895–898
744  descendientes,  636  diagrama,  6   Tipo  0  gramática,  751 Weierstrass,  Karl,  68
excentricidad,  614  requisitos  de  borde,   Gramática  tipo  1,  751 Peso,  de  árboles  de  expansión,  626
612–613  familia,  611 Gramática  tipo  2,  751 Matriz  de  adyacencia  ponderada,  528,  716
Gramática  tipo  3,  751
Dígrafos  ponderados,  715–725  
tu
Gráficos  ponderados,  527–527  
Operador  unario,  12 Fórmula  bien  formada,  107  Principio  
predicado  unario,  33 de  buena  ordenación,  186  Gráficos  
Conjunto  incontable,  141 de  ruedas,  526,  623  Ciclos  while,  
Fibonacci,  646–647   Gráfico  subyacente,  702,  705 618,  622  Expresiones  booleanas  en,  
completo,  639  juego,  637   Grafo  no  dirigido,  517 7  Corrección,  225  Sentencias  
Unión  
elegante,  686  altura  y   while,  795  Whitehead,  Alfred  North,  
nivel,  637–638   de  grafos,  532–533   42  Wiles,  Andrew  J.,  4  Williams,  Ben  
heterogéneo,  655  homogéneo,   relaciones,  462–463  de   Ames,  493  Wilson,  John,  512  prima,  
664 conjuntos,  78 512  teorema,  512  Palabra(s)  vacía(s),  
Unidad  de  máquinas  de  retardo,  774 75  igualdad  de,  734–735  longitud,  75  
Huffman,  670–675   elemento  unidad,  806 nulo,  75  ternario,  78 ,  349
vértice  interno,  636   Cuantificadores  universales,  32
isomorfo,  645  hoja,  636   Conjuntos  universales,  70–71
expansión  mínima,  626– Universo  del  discurso  (UD),  33
634  m­nario,  639–640  m­nario,   Límite  superior,  sumatoria,  158
completo,  645  raíz  ordenada,  638   Matriz  triangular  superior,  874
padre,  636  análisis,  745  requisitos   V
de  ruta,  611–  612  arraigado,  635–
Vacuamente  cierto,  11
646  hermanos,  636  expansión,  614–
Pruebas  vacías,  49–50
634  sub,  636,  646  vértice  terminal,   Complejidad  temporal  en  el  peor  de  los  casos,  248
Argumentos  válidos,  39–49
636  ternario,  639  transversales,  647– Valor
653 X
absoluto,  33,  126  
XOR,  18  
esperado,  419–420  
plano  xy,  88
verdad,  3–5
Vandermonde,  Alejandro
Y  
Théophile,  568,  569  
Yager,  Ronald  R.,  28  
identidad,  398–399
Variables método,  28–29  Yashima,  
Números  triangulares,  216,  285,  382 530–531
Booleano,  2,  813  
lógica,  2
Triangulación  de  polígonos  convexos,  275 Z  
Venn,  John,  72  
Zadeh,  Lotfi  A.,  91,  92  Zeller,  
diagramas,  72–73
Teorema  del  trinomio,  889–890 Vértice  (vértices)   Christian  Julius  Johannes,  492  fórmula,  492  
Pruebas  triviales,  50 Elemento  cero,  806  Matrices  cero,  
grado  de,  520,  694–698  
Tablas  de   166
dígrafo,  445,  692  gráfico,  445,  
verdad  para  enunciados  
517  inicial,  692
bicondicionales,  14–15
Machine Translated by Google

Lista  de  biográficos
bocetos

Nombre  y  fechas Página

1.  Aristóteles  (384–322  a.  C.) (2)  
2.  Leibniz,  barón  Gottfried  Wilhelm  (1646­1716) (3)  
3.  Boole,  Jorge  (1815–1864) (4)  
4.  Fermat,  Pierre­Simon  de  (1601–1665) (5)  
5.  Goldbach,  cristiano  (1690­1764) (5)  
6.  De  Morgan,  Augusto  (1806–1871) (23)  
7.  Kosko,  Bart  8.   (27)  
Isaka,  Satoru  9.   (27)  
Dudeney,  Henry  Ernest  (1857–1930) (40)  
10.  Russell,  Bertrand  Arthur  William  (1872–1970) (42)  
11.  Carroll,  Lewis  (1832–1898) (45)  
12.  Cantor,  Jorge  (1845­1918) (68)  
13.  Venn,  Juan  (1834­1923) (72)  
14.  Hilbert,  David  (1862­1943) (74)  
15.  Descartes,  René  (1596­1650) (87)  
16.  Zadeh,  Lotfi,  A.  (1921–) (92)  
17.  Iverson,  Kenneth  E.  (1920–) (127) )  
18.  Dirichlet,  Gustav  Peter  Lejeune  (1805–1859) (145)  
19.  Erdös,  Paul  (1913–1996) (147)  
20.  Lagrange,  José  Luis  (1736–1813) (159)  
21.  Cayley,  Arturo  (1821–1895) (164)  
22.  Silvestre,  James  Joseph  (1814–1897) (164)  
23.  Hamming,  Richard  Wesley  (1915–1998) (180)  
24.  Euclides   (192)  
25.  Bernoulli,  Jacob  I.  (1654–1705) (212)  
26.  Bachmann,  Paul  Gustav  Henrich  (1837­1920) (238)  
27.  Landau,  Edmundo  (1877­1938) (238)  
28.  Knuth,  Donald  Ervin  (1938–) (244)  
29.  Fibonacci,  Leonardo  (¿1170?­1250?) (269)  
30.  Lucas,  François­Edouard­Anatole  (1842–1891) (274)
Machine Translated by Google

Nombre  y  fechas Página

31.  McCarthy,  Juan  (1927–) (275)  
32.  Binet,  Jacques  Philippe  Marie  (1786–1865) (277)  
33.  De  Moivre,  Abraham  (1667–1754) (299)  
34.  Lamé,  Gabriel  (1795–1870) (325)  
35.  Pascual,  Blaise  (1623­1662) (371)  
36.  Laplace,  Pierre­Simon  (1749–1827) (411)  
37.  Gauss,  Karl  Friedrich  (1777–1855) (484)  
38.  Hasse,  Helmut  (1898­1979) (497)  
39.  Euler,  Leonard  (1707­1783) (516)  
40.  Petersen,  Julio  (1839­1910) (532)  
41.  Hamilton,  Guillermo  Rowan  (1805–1865) (565)  
42.  Vandermonde,  Alexandre­Théophile  (1735–1796) (569)  
43.  Dirac,  Gabriel  Andrés  (1925­1984) (571)  
44.  Mineral,  Oystein  (1899–1968) (571)  
45.  Kuratowski,  Kazimierz  (1896­1980) (584)  
46.  Guthrie,  Francisco  (1831–1899) (589)  
47.  Kirchoff,  Gustav  Robert  (1824–1887) (610) )  
48.  Kruskal,  Joseph  Bernard  (1928–) (616)  
49.  Prim,  Robert  Clay  (1921–) (629)  
50.  Lukasiewicz,  enero  (1878–1956) (654)  
51.  Huffman,  David  Albert  (1925–1999) (671)  
52.  Dijkstra,  Edsger  Wybe  (1930­2002) (717)  
53.  Floyd,  Robert  W.  (1936–) (726)  
54.  Kleene,  Stephen  Cole  (1909–1994) (739)  
55.  Backus,  John  W.  (1924–) (748)  
56.  Naur,  Peter  (1928–) (748)  
57.  Chomsky,  (Avram)  Noam  (1928–) (752)  
58.  Moore,  Edward  Forrest  (1925–) (800)  
59.  Shannon,  Claude  Elwood  (1916­2001) (804)  
60.  Karnaugh,  Mauricio  (1924–) (843)
Machine Translated by Google

Índice  de  aplicaciones

Artículo Elemento  de  página Página

Cajero  automático  Números  de   768 Caminar  enrejado 377  


placas  de  automóviles  Cheque  bancario   347  Paréntesis  emparejados  legalmente 106  
Número  de  identificación  Un  rompecabezas   493  Luminarias  en  pasillos  198 834  
binario  La  paradoja  del  cumpleaños  El   Redes  de  área  local 529  
rompecabezas  de  la  col,  la  cabra  y  el  lobo   182,  412  Rompecabezas  de  lógica 39–42  
Trato  de  cartas  Juegos  de  casino  Cocos  y   552  Números  de  Lucas  132   273  
monos  Sistema  de  código  de  barra  El   Números  dgráfico  
e  Lucas  y  
de   el  
la  
rueda  132  Cuadrados  mágicos   623  
problema  del  círculo  Disyuntores   493  Lotería  estatal  de  Massachusetts  493  Liga  Nacional   222  
Programación  libre  de  conflictos  Trastornos   de  Fútbol  Americano  370  Liga  Nacional  de  Hockey  829   411  
de  conteo  Conteo  de  sobreyecciones   Cubo  n  591  Un  rompecabezas  no  decimal  404  Número   91  
Cicloparafinas  Bases  de  datos  Día  de  la   de  años  bisiestos  402  Número  de  particiones  524   164,  612  
semana  Secuencias  de  De  Bruijn   Parafinas  464  Máquina  de  verificación  de  paridad 566  
Trastornos  Pantallas  digitales  Domingo  de   205  
Pascua  El  rompecabezas  de  las  ocho   130  
monedas  EQUIVALENCIA  en  FORTRAN   490,  492  
Números  de  Fibonacci  Árboles  de  Fibonacci   523  
Viernes  trece  El  rompecabezas  de  las   765  
cuatro  reinas  Lógica  difusa  en  la  toma  de   135,  492  El  problema  de  la  pizza 368  
decisiones  El  juego  de  Yashima  Tipos  de   702 Función  de  oficina  de  correos   128  
genes  El  dorado  relación  Códigos  Gray  El   360  números  romanos 798  
problema  del  apretón  de  manos  Código   854  Torneo  de  todos  contra  todos 526  
Hashing  Huffman  Moléculas  de  hidrocarburo   135,  195  SIM 531  
Requisitos  de  insulina  Libro  estándar   678  semillas  de  girasol 271  
internacional  No. 483  Análisis  de  encuestas 100  
269  Circuitos  de  conmutación 16  
646  Lógica  simbólica  en  la  toma  de  decisiones 39  
485  tareas  en  la  construcción  de  una  casa 494  
619  Telecomunicaciones 528  
28  El  problema  del  teletipo 700  
530  El  rompecabezas  de  las  tres  casas­utilidades 577  
697  tres  en  raya 637  
276  Tractor­segadora 825  
446  patrones  de  semáforos 592  
264,  533  Torre  de  Brahma  138 265  
Triangulaciones  de  un  n­ágono 275,  393  
674  Torniquetes 762  
519  Doce  días  de  Navidad 221,  373  
173  Rompecabezas  de  dos  reinas 133  
509  Máquina  de  retardo  unitario 774  
Caballeros  ona3H3  tablero  de  ajedrez   558  Servicio  de  paquetería  unido 492  
Knight's  tour  Puntos  de  celosía 567  Factura  de  agua   121  
145 Fórmulas  bien  formadas 107

1043
Machine Translated by Google

Esta  página  se  ha  dejado  en  blanco  intencionalmente
Machine Translated by Google

Índice  de  algoritmos

Algoritmo Página Algoritmo Página

Algoritmo  del  siguiente  subconjunto 96   Evaluación  de  polinomios 336  


Subconjuntos  de  un  conjunto 102   Relación  de  conectividad 474  
Secuencia  emparejada  legalmente 106   Algoritmo  de  Warshall 481  
producto  matriz 171   Relación  de  equivalencia 490  
Algoritmo  de  números  primos 190   clasificación  topológica 501  
Algoritmo  de  Euclides 193   Algoritmo  de  Horner 336  
Representación  en  base  b 198   gráfico  euleriano 559  
Adición  en  base  b 199   circuito  de  eulerain 561  
multiplicación  binaria 203   Árbol  de  expansión  mínimo 616  
multiplicación  de  enteros 225   Búsqueda  en  profundidad 618  
Algoritmo  factorial 226   Búsqueda  en  amplitud 621  
búsqueda  lineal 227,  316   Algoritmo  de  Kruskal 628  
Búsqueda  binaria 229,  301   algoritmo  de  Prim 631  
Ordenamiento  de  burbuja 232,  317   Recorrido  en  orden 647  
Clasificación  de  selección 233,  321   Recorrido  de  pedido  anticipado 647  
Tipo  de  inserción 236   Recorrido  posterior  al  pedido 648  
Valor  máximo  en  una  lista 251   Árbol  de  búsqueda  binario 665  
Mínimo  y  máximo  en  una  lista 252   Algoritmo  de  Huffman 671  
apretones  de  manos 308   Algoritmo  de  Dijkstra 721  
Torre  de  Brahma 309   Identificador  válido 767  
Fibonacci 310,  320   Forma  normal  disyuntiva 819  
Ordenar  por  fusión 314   Siguiente  permutación 884  
exponenciación 318 Próxima  combinación 887

1044
Machine Translated by Google

Lista  de  Símbolos

Tema Símbolo Sentido Página

LÓGICA    negación  de  p   8  

pp     q disyunción  de  p  y  q  


p      conjunción  de  p  y  q  exclusiva   7  

qp  XOR  q o  de  p  y  qp  implica  qp  si  y   5  

pag  →  q sólo  si  qp  es  lógicamente  
pq equivalente  a  q  contradicción   18  

p  ≡  q   tautología  no  p  y  q  no  p  o  q 9  


FT  p   14  
NAND   20  

qp  NOR  qp  |   21  

q  P(x1,  x2, ... ,   21  

xn)  ( x)P(x)   Función   30  

( x)P(x) proposicional  de  trazo  de  Sheffer 30  

Cuantificación  existencial  P(x) 31  

Cuantificación  universal  P(x)  por  lo   33  32  32
tanto 38

CONJUNTOS {x1,  x2, ... ,  xn}  {x  |   conjunto  con  elementos  x1,  x2, ...  y  xn  notación   68


P(x)}  Ø  UZ de  creación  de  conjuntos 69
conjunto   70
vacío  conjunto   70  
universal  conjunto   73  
N,  Z+ de  enteros  conjunto  de   73  
R   enteros  positivos  conjunto  de   73  
[a,  b]   números  reales  intervalo   73  
[a,  b)   cerrado  intervalo  cerrado­ 73  
(a,  b]   abierto  intervalo  abierto­ 73  
(a,  b) cerrado  intervalo  abierto   73  
UN  =   igualdad  de  conjuntos  A  y  B  x   70  
segundo  x   es  un  elemento  de  A  x  no  es   68  
  UN  x   /   un  elemento  de  AA  es  un   68  
AA      subconjunto  de  B  conjunto   69  
segundo  PAG  (A) potencia  de  A  cardinalidad  de   72  
|A|   A  par  ordenado  conjunto   98  
(a,  b)   ordenado  unión  de  A  y  B   86  

(a1,  a2, ... ,  an) intersección  de  A  y  B  diferencia   86  


A      de  A  y  B  complemento  de  A   78  
BA  ∩  B diferencia  simétrica  de  A  y  B   78  
A  ­  B unión  de  conjuntos  Ai,i  =  1,   80  
A 2, ... , 81  
A     B 82
norte

Ai norte 86
  i=1
norte

∩   Ai intersección  de  conjuntos  Ai,i  =  1,  2, ... , norte 86


yo=1

FUNCIONES f(x)   valor  de  la  función  f  en  x   119  


f :  A  →  B  f  +   función  f  de  A  a  B  suma  de   118  
g  fg funciones  f  y  g  producto  de   123  
funciones  f  y  g 123
Machine Translated by Google

Tema Símbolo Sentido Página

I  a función  de  identidad  en  A   136


−1   153  
ff  ◦   inversa  de  f  composición  de  
ga   g  y  f  resto  cuando  a  se   151  
mod  ba   divide  por  b  cociente  cuando  a  se  divide   132  
div  b por  b  piso  de  x  techo  de  x  n­ésimo  término   132
X la  secuencia  {sn} 126
X 127
sn 157
norte

am  +  am+1  +∙∙∙+  un 158
yo  =  m
ai   suma  los  términos  ai,  donde  i     S 160
yo     S
norte

ai amam+1  ∙∙∙  un 163
yo  =  
m  n! n  factorial   241
f(x)  =  O(g(x))  f(x)   f(x)  es  gran­oh  de  g(x)   237
=  (g(x))  f(x)  =   f(x)  es  gran­omega  de  g(x)   243  
(g(x))  mín  {x,  y}   f(x)  es  gran­theta  de  g(x)   245  
máx  {x,  y} mínimo  de  x  e  y  máximo  de  x   252  
y  y 251

MATRICES 165  
(aij)m×n matriz  m  ×  n  con  entradas  aij  suma  
A  +  B de  A  y  B  diferencia  de  A  y  B   166  
A  −  B   multiplicación  escalar  de  A  por  k   167  
kA producto  de  A  y  B  matriz  identidad  de   167  
AB orden  n  traspuesta  de  A 170  
En 166
A 174

ENTEROS un  |   a  divide  a  ba   189  


ba  b   no  es  factor  de  b  máximo   189  
mcd  {a,  b}   común  divisor  a  y  b  mínimo  común  divisor   191
lcm  {a,  b}   de  a  y  b  representación  en  base  b
(a1a2  ∙∙∙  an)b 197

COMBINADORES  P(n,  r) número  de  permutaciones  r  de  un  conjunto  con  n  elementos   352


Dn número  de  desarreglos  de  n  elementos  número  de  combinaciones   361
C(n,  r) r  de  un  conjunto  con  n  elementos 366
norte

366
norte

coeficiente  binomial:  coeficiente  de  xn  en  desarrollo  de  (1  +  x)
r
400
N(Pi1Pi2 ...Pin )
norte

número  de  elementos  que  tienen  cada  una  de  las  propiedades  Pij ,  j  =  1,  2, ... ,  
N(P  P  ∙∙∙P )  i1  i2  en norte 400
número  de  elementos  que  no  tienen  ninguna  de  las  propiedades  Pij ,  j  =  1,  2, ... ,
PROBABILIDAD EDUCACIÓN  FÍSICA) probabilidad  del  evento  E   410  
P(E|F) probabilidad  condicional  de  E  dado  F 417

BOOLEANO A     B unión  de  A  y  B   439  


MATRICES A     B encuentro  de  A  y  B   439  
AB producto  booleano  de  A  y  B  n­ 439
Un] ésima  potencia  booleana  de  A 441

continuado
Machine Translated by Google

Tema Símbolo Sentido Página

RELACIONES S  ◦  R compuesto  de  relaciones  S  y  R  n­ 463  


Rn ésima  potencia  de  la  relación  R   466
R−1   relación  inversa  a  es  congruente  con   469  
a  ≡  b(mod  m)  a   b  módulo  ma  no  es  congruente  con  b   484  
≡  b(mod  m)  [a]R   módulo  m  clase  de  equivalencia  de  a  con   484  
a     bababa  b respecto  a  R  a  es  menor  que  ba  es  menor  o  igual   486  
que  ba  es  mayor  que  ba  es  mayor  o  igual  que  b 495  
495

GRÁFICOS {u,  v}  o  {u  −  v} gráfico  de  aristas   517  


G  =  (V,  E)   no  dirigido  con  conjunto  de  vértices  V  y  conjunto   517  
grados  (v) de  aristas  E  grado  del  vértice  v  gráfico  completo   520  
kn con  n  vértices  ciclo  con  n  vértices  rueda  con  n  +   524
cn 1  vértices  gráfico  bipartito  completo  n­cubo   526
Wn camino  de  a  a  b  unión  de  G1  y  G2  grado  de  la   526
km,  n región  R 527  
Qn   566  
a­v1­  ∙∙∙  ­  vn−1­b 546  
G1     G2   532  
grado(R) 581

BOOLEANO B {0,  1}   807  


ÁLGEBRA x,  y variables  booleanas   813
X complemento  de  x  
x  +  yx   suma  booleana  de  x  e  y   814  
∙  y  (o  xy)  x  ↑   producto  booleano  de  x  e  yx   814  
yx  ↓  y NAND  yx  NOR  y  inversor 820  
820  

X X 827

X
x+y O  puerta 826
y

X
xy Y  puerta 825
y

IDIOMAS xy   concatenación  de  palabras  x  e  y   76  


λ  || cadena  vacía  longitud  de  la  cadena  x   75  
x||   inversión  de  w  concatenación  de   75
wR idiomas  A  y  B 113  
AB 736  
A Alfabeto  de  cierre   739  
Kleene 75

ESTADO  FINITO σ iniciar  la   744


MÁQUINAS w  →  w producción  de   744
w     w símbolos  w  es  directamente  derivable  de  w 746
Un  símbolo  no  terminal 748
A::=  B|  X Forma  de  Backus­Naur 748
(S,  I,  O,  f,  g,  s0)  máquina  de  estados  finitos  con  salida  s0 772  
estado  de  inicio 760  
(S,  I,  f,  s0,  F) máquina  de  estados  finitos  sin  salida 761

También podría gustarte